You are on page 1of 136

178

Solutions ManualFluid Mechanics, Seventh Edition

Chapter 6 Viscous Flow in Ducts


P6.1AnengineerclaimsthatflowofSAE30Woil,at20C,througha5cmdiametersmooth
pipeat1millionN/h,islaminar.Doyouagree?Amillionnewtonsisalot,sothissoundslike
anawfullyhighflowrate.

Solution: ForSAE30Woilat20C(TableA.3),take =891kg/m3 and =0.29kg/ms.


ConverttheweightflowratetovolumeflowrateinSIunits:
Q

(1E 6 N / h)(1 / 3600 h / s )


w
m3

0
.
0318

(0.05m) 2 V , solve V 16.2


3
2
g
s
4
s
(891kg / m )(9.81m / s )

Calculate

Re D

VD
(891kg / m 3 )(16.2m / s )(0.05m)

0.29 kg / m s

2500

(transitional )

Thisisnothigh,butnotlaminar.Ans.Withcarefulinletdesign,lowdisturbances,andavery
smoothwall,itmightstillbelaminar,butNo,thisistransitional,notdefinitelylaminar.

P6.2
The present pumping rate of North Slope crude oil through the Alaska
Pipeline (see the chapter-opener photo) is about 600,000 barrels per day (1 barrel = 42
U.S. gallons). What would be the maximum rate if the flow were constrained to be
laminar? Assume that Alaskan crude oil fits Fig. A.1 of the Appendix at 60C.
Solution: From Fig. A.1 for crude at 60C, = 0.86(1000) = 860 kg/m3 and = 0.0040
kg/m-s. From Eq. (6.2), the maximum laminar Reynolds number is about 2300. Convert
the pipe diameter from 48 inches to 1.22 m. Solve for velocity:
Re D 2300

(860kg / m3 ) V (1.22m)
VD
m

; Solve for V 0.00877

0.0040 kg / m s
s

Q VA (0.00877 m / s ) ( / 4)(1.22m) 2 0.01025 m3 / s 3600 24

886 m3 / day 5600 barrels/day Ans.

179

Solutions ManualFluid Mechanics, Seventh Edition

P6.3
Following up Prob. P6.2, suppose the Alaska Pipeline were carrying 30
million U.S. gallons per day of SAE 30W oil at 20C? Estimate the Reynolds number. Is the
flow laminar?
Solution: For SAE 30W oil at 20C, Table A.3, = 891 kg/m3, and = 0.29 kg/m-s.
Convert the flow rate into cubic meters per second and then find the Reynolds number:
gal
gal
m3
m
24 3600 347
0.0037854 1.314
(or V 1.13 )
day
s
s
s
4Q
4(891)(1.314)

4200 Ans. (Transitional , not laminar)


D (0.29)(1.22m)

Q 30E6
Re D

6.4Forflowof SAE30Woil througha5cmdiameterpipe,fromFig.A.1,forwhat


3
flowrateinm /hwouldweexpecttransitiontoturbulenceat(a)20Cand(b)100C?
Solution:ForSAE30Woiltake 891kg/m 3 andtake0.29kg/msat20C(Table
A.3)and0.01kg/msat100C(FigA.1).WritethecriticalReynoldsnumberintermsof
flowrateQ:
VD 4Q
4(891kg/m 3 )Q
(a) Re crit 2300

D (0.29kg/ms)(0.05m )
m3
m3
solve Q 0.0293
106
Ans.(a)
s
h
VD 4Q
4(891kg/m 3 )Q
(b) Re crit 2300

D (0.010kg/ms)(0.05m)
solve Q 0.00101

m3
m3
3.6
s
h

Ans.(b)

180

Chapter 6Viscous Flow in Ducts

6.5Inflowpastabodyorwall,earlytransitiontoturbulencecanbeinducedbyplacing
atripwireonthewallacrossthe flow,asinFig.P6.5.IfthetripwireinFig.P6.5 is
placedwherethelocalvelocityisU,itwilltriggerturbulenceifUd/850,wheredisthe
wirediameter[Ref.3ofCh.6].Ifthespherediameteris20cmandtransitionisobservedat
ReD90,000,whatisthediameterofthetripwireinmm?

Fig.P6.5

Solution:ForthesameUand,
Ud
UD
Re d
850; Re D
90000,

Re
850
or d D d (200mm)
1.9 mm
90000

Re D

P6.6 Forflowofauniformstreamparalleltoasharpflatplate,transitiontoaturbulent
boundarylayerontheplatemayoccuratRex=Ux/1E6,whereUistheapproachvelocity
andxisdistancealongtheplate.IfU=2.5m/s,determinethedistancexforthefollowingfluids
at20Cand1atm:(a)hydrogen;(b)air;(c)gasoline;(d)water;(e)mercury;and(f)glycerin.
Solution:Wearetocalculatex=(Rex)()/(U)=(1E6)()/[(2.5m/s)].Makeatable:
FLUID

kg/m3

kg/ms

xmeters

Hydrogen

0.00839

9.05E5

43.

Air

1.205

1.80E5

6.0

Gasoline

680

2.92E4

0.17

Water

998

0.0010

0.40

Mercury

13,550

1.56E3

0.046

181

Solutions ManualFluid Mechanics, Seventh Edition

Glycerin

1260

1.49

470.

Clearlytherearevastdifferencesbetweenfluidpropertiesandtheireffectsonflows.

6.7Cola,approximatedaspurewaterat20C,istofillan8ozcontainer(1U.S.gal
128floz)througha5mmdiametertube.Estimatetheminimumfillingtimeifthetube
flowistoremainlaminar.Forwhatcola(water)temperaturewouldthisminimumtime
be1min?
3

Solution:Forcolawater,take998kg/m and0.001kg/ms.Convert8fluid
3
3
ounces(8/128)(231in )2.37E4m .Then,ifweassumetransitionatRe2300,
Re crit 2300

VD 4 Q
2300 (0.001)(0.005)
m3

, or: Q crit
9.05E6

D
4(998)
s

Thentfill/Q2.37E4/9.05E626sAns.(a)
3

(b)WefillinexactlyoneminuteifQcrit2.37E4/603.94E6m /s.Then
Q crit 3.94E6

m 3 2300 D

s
4

if water 4.36E7m 2 /s

FromTableA1,thiskinematicviscosityoccursatT66CAns.(b)
3

6.8Whenwaterat20C( 998kg/m , 0.001kg/ms)flowsthroughan8cm


diameterpipe,thewallshearstressis72Pa.Whatistheaxialpressuregradient(p/x)
ifthepipeis(a)horizontal;and(b)verticalwiththeflowup?
Solution:Equation(6.9b)appliesinbothcases,notingthatwisnegative:
(a) Horizontal:
(b) Vertical,up:

dp 2 w 2(72Pa )
Pa

3600
dx
R
0.04m
m

Ans.(a)

dp 2 w
dz 1
Pa

g
3600 998(9.81) 13, 400
dx
R
dx
m

Ans.(b)

182

Chapter 6Viscous Flow in Ducts

6.9Alightliquid( 950kg/m )flowsatanaveragevelocityof10m/sthrougha


horizontalsmoothtubeofdiameter5cm.Thefluidpressureismeasuredat1mintervals
alongthepipe,asfollows:
x,m:
p,kPa:

0
304

1
273

2
255

3
240

4
226

5
213

6
200

Estimate(a)thetotalheadloss,inmeters;(b)thewallshearstressinthefullydeveloped
sectionofthepipe;and(c)theoverallfrictionfactor.
Solution:AssketchedinFig.6.6ofthetext,thepressuredropsfastintheentrance
region (31 kPa in the first meter) and levels off to a linear decrease in the fully
developedregion(13kPa/mforthisdata).
(a)Theoverallheadloss,forz0,isdefinedbyEq.(6.8)ofthetext:
hf

p 304,000 200,000Pa

11.2m
g (950kg/m3 )(9.81m/s2 )

Ans.(a)

(b)ThewallshearstressinthefullydevelopedregionisdefinedbyEq.(6.9b):
p
fully developed 13000Pa 4 w 4 w , solvefor w 163 Pa
L
1m
d
0.05m

Ans.(b)

(c)TheoverallfrictionfactorisdefinedbyEq.(6.10)ofthetext:
foverall h f ,overall

2
d 2g
0.05m 2(9.81m/s )

(11.2
m
)
0.0182

L V2
(10m/s)2
6m

Ans.(c)

NOTE:Thefullydevelopedfrictionfactorisonly0.0137.

6.10Waterat20C(998kg/m )flowsthroughaninclined8cmdiameterpipe.At
sectionsAandB,pA186kPa,VA3.2m/s,zA24.5m,whilepB260kPa,VB3.2
m/s,andzB9.1m.Whichwayistheflowgoing?Whatistheheadloss?

183

Solutions ManualFluid Mechanics, Seventh Edition

Solution:Guess that the flow is from A to B and write the steady flow energy
equation:
pA VA2
pB VB2
186000
260000

zA

zB h f , or:
24.5
9.1 h f ,
g 2g
g 2 g
9790
9790
or: 43.50 35.66 h f , solve: h f 7.84 m Yes, flow is from Ato B. Ans.(a,b)

6.11Waterat20Cflowsupwardat4m/sina6cmdiameterpipe.Thepipelength
between points 1 and 2 is 5 m, and point 2 is 3 m higher. A mercury manometer,
connectedbetween1and2,hasareadingh135mm,withp1higher.(a)Whatisthe
pressurechange(p1p2)?(b)Whatistheheadloss,inmeters?(c)Isthemanometerreading
proportionaltoheadloss?Explain.(d)What
isthefrictionfactoroftheflow?
Solution:A sketch of this situation is
shown at right. By moving through the
manometer,weobtainthepressurechange
betweenpoints1and2,whichwecompare
withEq.(6.9b):
p1 w h m h w z p2 ,
N

N
or: p1 p2 133100 9790 3 (0.135m) 9790 3 (3m)

m
m
16650 29370 46,000Pa
FromEq.(6.9b), h f
The friction factoris

Ans.(a)

p
46000Pa
z
3m 4.7 3.0 1.7m
w
9790N /m3

f hf

Ans.(b)

2
d 2g
0.06m 2(9.81m/s )

(1.7
m
)
0.025 Ans.(d)

L V2
5m
(4m/s)2

Bycomparingthemanometerrelationtotheheadlossrelationabove,wefindthat:
hf

( m w )
h
w

andthusheadlossis proportionaltomanometerreading. Ans.(c)

NOTE:INPROBLEMS6.12TO6.99,MINORLOSSESARENEGLECTED.

184

Chapter 6Viscous Flow in Ducts

6.12A5mmdiametercapillarytubeisusedasaviscometerforoils.Whentheflow
3
rateis0.071m h,themeasuredpressuredropperunitlengthis375kPam.Estimatethe
viscosityofthefluid.Istheflowlaminar?Canyoualsoestimatethedensityofthefluid?
Solution:Assumelaminarflowandusethepressuredropformula(6.12):
p ? 8Q
Pa ? 8(0.071/3600)
kg

, or: 375000

, solve 0.292
Ans.
4
4
L R
m
ms
(0.0025)
kg
Guessing oil 900 3 ,
m
4 Q 4(900)(0.071/3600)
check Re

16 OK,laminar Ans.
d
(0.292)(0.005)
Itisnotpossibletofinddensityfromthisdata,laminarpipeflowisindependentofdensity.

6.13A soda straw is 20 cm long and 2 mm in diameter. It delivers cold cola,


3
approximatedaswaterat10C,atarateof3cm s.(a)Whatistheheadlossthroughthe
straw? What is the axial pressure gradient px if the flow is (b) vertically up or
(c)horizontal?Canthehumanlungdeliverthismuchflow?
3

Solution:Forwaterat10C,take1000kgm and1.307E3kgms.CheckRe:
Re

4 Q 4(1000)(3E6m 3 /s)

1460 (OK,laminarflow)
d (1.307E3)(0.002)

Then,fromEq.(6.12), h f

128 LQ 128(1.307E3)(0.2)(3E6)

0.204m
gd 4
(1000)(9.81)(0.002)4

Ans.(a)

Ifthestrawishorizontal,thenthepressuregradientissimplyduetotheheadloss:
p
horiz gh f 1000(9.81)(0.204m) 9980 Pa
L
L
0.2m
m

Ans.(c)

Ifthestrawisvertical,withflowup,theheadlossandelevationchangeaddtogether:
p
vertical g(h f z) 1000(9.81)(0.204 0.2) 19800 Pa
L
L
0.2
m

Ans.(b)

Thehumanlungcancertainlydelivercase(c)andstronglungscandevelopcase(b)also.

185

Solutions ManualFluid Mechanics, Seventh Edition

6.14Water at 20C is to be siphoned


through a tube 1 m long and 2 mm in
diameter, as in Fig. P6.14. Is there any
height H forwhichtheflowmightnotbe
laminar?WhatistheflowrateifH50cm?
Neglectthetubecurvature.

Fig.P6.14

Solution:Forwaterat20C,take998kgm and0.001kgms.Writethesteady
flowenergyequationbetweenpoints1and2above:
patm 02
p
V2
V2
32 L

z1 atm tube z 2 h f , or: H


hf
V
g 2g
g
2g
2g
gd 2

(1)

V2
32(0.001)(1.0)V
m
EnterdatainEq.(1): 0.5

, solve V 0.590
2
2(9.81) (998)(9.81)(0.002)
s
Equation(1)isquadraticinVandhasonlyonepositiveroot.Thesiphonflowrateis

m3
m3
(0.002)2 (0.590) 1.85E6
0.0067
if H 50 cm Ans.
4
s
h
Check Re (998)(0.590)(0.002) /(0.001) 1180(OK,laminarflow)

Q H=50cm

ItispossibletoapproachRe2000(possibletransitiontoturbulentflow)forH1m,
forthecaseofthesiphonbentovernearlyvertical.WeobtainRe2000atH0.87m.
6.15ProfessorGordonHollowayandhisstudentsattheUniversityofNewBrunswick
3
wenttoafastfoodemporium andtriedtodrinkchocolate shakes( 1200kg/m ,
6kg/ms)throughfatstraws8mmindiameterand30cmlong.(a)Verifythattheir
humanlungs,whichcandevelopapproximately3000Paofvacuumpressure,wouldbe
unabletodrinkthemilkshakethroughtheverticalstraw.(b)Astudentcut15cmfrom
hisstrawandproceededtodrinkhappily.Whatrateofmilkshakeflowwasproducedby
thisstrategy?
Solution:(a)Assumethestrawisbarelyinsertedintothemilkshake.Thentheenergy
equationpredicts
2
p1 V1
p2 V22

z hf
g 2 g 1 g 2 g 2
000

(3000Pa )
V 2tube

0.3m h f
(1200kg/m3 )(9.81m/s2 ) 2 g

Solve for h f 0.255 m 0.3 m

V2tube
0 whichisimpossible
2g

Ans.(a)

186

Chapter 6Viscous Flow in Ducts

(b)Bycuttingoff15cmofverticallengthandassuminglaminarflow,weobtainanew
energyequation
h f 0.255 0.15

V 2 32 LV
V2
32(6.0)(0.15)V

0.105
m

38.23V
2
2g
2(9.81) (1200)(9.81)(0.008)2
gd

Solvefor V 0.00275m/s, Q AV ( /4)(0.008)2 (0.00275)


Q 1.4 E7

m3
cm 3
0.14
s
s

Ans.(b)

Check the Reynolds number:Red Vd/ (1200)(0.00275)(0.008)/(6) 0.0044


(Laminar).

P6.16

Fluid flows steadily, at volume rate Q, through a large horizontal pipe and then
divides into two small pipes, the larger of which has an inside diameter of 25 mm and carries
three times the flow of the smaller pipe. Both small pipes have the same length and pressure
drop. If all flows are laminar, estimate the diameter of the smaller pipe.
Solution: For laminar flow in a horizontal pipe, the volume flow is a simple formula,
Eq. (6.12):
d 4 p
Qlaminar
( )
128 L
Since p, L, and are the same in the two small pipes, it follows that the flows simply vary as
the 4th power of their diameters. Let pipe 1 have the 25-mm diameter. Then we compute
Q1 ( const ) ( d14 ) 3Q2 3(const )(d 24 )
Thus

d2

d1
31/ 4

25 mm

1.316

19.0 mm

Ans.

187

Solutions ManualFluid Mechanics, Seventh Edition

6.17A capillaryviscometer measuresthetimerequiredforaspecifiedvolume of


liquidtoflowthroughasmallboreglasstube,asinFig.P6.17.Thistransittimeisthen
correlatedwithfluidviscosity.Forthesystemshown,(a)deriveanapproximateformula
forthetimerequired,assuminglaminarflowwithnoentranceandexitlosses.(b)IfL
3
12cm,l2cm,8cm ,andthefluidiswaterat20C,whatcapillarydiameterDwill
resultinatransittimetof6seconds?

Fig.P6.17

Solution:(a)Assumenopressuredropandneglectvelocityheads.Theenergyequation
reducesto:
p1 V12
p2 V22

z1 0 0 ( L l )

z2 h f 0 0 0 h f , or: h f L l
g 2g
g 2g
Forlaminar flow, h f
Solve for

128 LQ
gd 4
t

and, foruniformdraining, Q

128 L
gd 4 ( L l )

Ans.(a)

(b)Applyto t 6s.Forwater,take 998kg/m and 0.001kg/ms.Formula(a)


predicts:
t 6s

128(0.001kg/ms)(0.12m)(8 E 6m 3 )
,
(998kg/m3 )(9.81m/s2 )d 4 (0.12 0.02m)

Solvefor d 0.0015 m

Ans.(b)

Chapter 6Viscous Flow in Ducts

188

6.18Todeterminetheviscosityofaliquidofspecificgravity0.95,youfill,toadepth
of12cm,alargecontainerwhichdrainsthrougha30cmlongverticaltubeattachedto
3
thebottom.Thetubediameteris2mm,andtherateofdrainingisfoundtobe1.9cm s.
Whatisyourestimateofthefluidviscosity?Isthetubeflowlaminar?

Fig.P6.18

Solution:Theknownflowrateanddiameterenableustofindthevelocityinthetube:
V

Q
1.9 E6m 3 /s
m

0.605
2
A ( /4)(0.002m)
s
3

Evaluate liquid0.95(998)948kgm .Writetheenergyequationbetweenthetopsurface


andthetubeexit:
2
pa Vtop
p
V2

ztop a
0 hf ,
g 2 g
g 2 g

or: 0.42

V 2 32 LV (0.605)2
32 (0.3)(0.605)

2
2g
2(9.81) 948(9.81)(0.002)2
gd

NotethatLinthisexpressionisthetubelengthonly(L30cm).
kg
(laminar flow) Ans.
ms
Vd 948(0.605)(0.002)
Red

446(laminar )

0.00257

Solvefor 0.00257

189

Solutions ManualFluid Mechanics, Seventh Edition

6.19Anoil(SG0.9)issuesfromthepipeinFig.P6.19atQ35ft /h.Whatisthe
3
kinematicviscosityoftheoilinft /s?Istheflowlaminar?
Solution:Applysteadyflowenergy:
patm 02
p
V2

z1 atm 2 z 2 h f ,
g 2g
g 2g

Fig.P6.19

where V2

Q
35/3600
ft

7.13
2
A (0.25 /12)
s
V22
(7.13)2
Solve h f z1 z2
10
9.21ft
2g
2(32.2)

Assuminglaminarpipeflow,useEq.(6.12)torelateheadlosstoviscosity:
128 LQ 128(6)(35/3600)

ft 2
h f 9.21ft

, solve 3.76E4

s
gd 4
(32.2)(0.5/12)4

Ans.

CheckRe 4Q/( d) 4(35/3600)/[ (3.76E4)(0.5/12)] 790(OK,laminar)

P6.20TheoiltanksinTinylandareonly160cmhigh,andtheydischargetotheTinylandoil
truckthroughasmoothtube4mmindiameterand55cmlong. Thetubeexitisopentothe
atmosphereand145cmbelowthetanksurface.Thefluidismediumfueloil,=850kg/m3and

=0.11kg/ms.Estimatetheoilflowrateincm3/h.

Solution:Thesteadyflowenergyequation,with1atthetanksurfaceand2theexit,gives

z1 z 2

V 2
LV2
V2
64 0.55m
850V (0.004)
f
, or : z 1.45m
(2.0
) , Re d
2g
d 2g
2g
Re d 0.004m
0.11

190

Chapter 6Viscous Flow in Ducts

Wehavetakentheenergycorrectionfactor=2.0forlaminarpipeflow.
SolveforV=0.10m/s,Red=3.1(laminar),Q=1.26E6m3/s4500cm3/h.Ans.
TheexitjetenergyV2/2gisproperlyincludedbutisverysmall(0.001m).

6.21InTinyland,housesarelessthana
foot high! The rainfall is laminar! The
drainpipe in Fig. P6.21 is only 2 mm in
diameter.(a)Whenthegutterisfull,what
is the rate of draining? (b) The gutter is
designedforasuddenrainstormofupto
5mmperhour.Forthiscondition,whatis
themaximumroofareathatcanbedrained
successfully?(c)WhatisRed?

Fig.P6.21

Solution:If the velocity at the gutter


surface is neglected, the energy equation
reducesto
V2
32 LV
z
h f , where h f ,laminar
2g
gd 2
3
Forwater,take 998kg/m and 0.001kg/ms.(a)With z known,this isa
quadraticequationforthepipevelocityV:
V2
32(0.001kg/ms)(0.2m)V
0.2m

,
2
2(9.81m/s ) (998kg/m 3 )(9.81m/s2 )(0.002m)2
m
or: 0.051V 2 0.1634V 0.2 0, Solve for V 0.945 ,
s
3
3

m
m
m

Q (0.002m)2 0.945 2.97E6


0.0107
Ans.(a)

4
s
s
h
3

(b)Theroofareaneededformaximumrainfallis0.0107m /h0.005m/h2.14m .Ans.


(b)
(c) The Reynolds number of the gutter is Re d (998)(0.945)(0.002)/(0.001) 1890
laminar.Ans.(c)

191

Solutions ManualFluid Mechanics, Seventh Edition

6.22AsteadypushonthepistoninFig.P6.22causesaflowrate Q 0.15cm /s
3
throughtheneedle.Thefluidhas 900kg/m and0.002kg/(ms).WhatforceFis
requiredtomaintaintheflow?

Fig.P6.22

Solution:Determinethevelocityofexitfromtheneedleandthenapplythesteady
flowenergyequation:
V1

Q
0.15

306cm/s
A ( /4)(0.025)2

p2 V22
p1 V12
Energy:

z2

z h h , with z1 z 2 ,V2 0,h f2 0


g 2g
g 2g 1 f1 f2
Assumelaminarflowfortheheadlossandcomputethepressuredifferenceonthepiston:
p2 p1
V12 32(0.002)(0.015)(3.06) (3.06)2
h f1

5.79m
g
2g (900)(9.81)(0.00025)2 2(9.81)

Then F pA piston (900)(9.81)(5.79) (0.01) 2 4.0N


4

Ans.

6.23SAE10oilat20Cflowsinaverticalpipeofdiameter2.5cm.Itisfoundthatthe
3
pressureisconstantthroughoutthefluid.Whatistheoilflowrateinm /h?Istheflowup
ordown?
3

Solution:ForSAE10oil,take870kg/m and0.104kg/ms.Writetheenergy
equationbetweenpoint1upstreamandpoint2downstream:
p1 V12
p
V2

z1 2 2 z 2 h f , with p1 p 2 and V1 V2
g 2g
g 2g
Thus h f z1 z 2 0bydefinition.Therefore,flowis down. Ans.
Whileflowingdown,thepressuredropduetofrictionexactlybalancesthepressurerise
duetogravity.

192

Chapter 6Viscous Flow in Ducts

AssuminglaminarflowandnotingthatzL,thepipelength,weget
hf

128 LQ
z L,
gd 4

(8.70)(9.81)(0.025)4
m3
m3
or: Q
7.87E4
2.83
128(0.104)
s
h

Ans.

6.24Twotanksofwaterat20Careconnectedbyacapillarytube4mmindiameter
and 3.5 m long. The surface of tank 1 is 30 cm higher than the surface of tank 2.
3
(a)Estimatetheflowrateinm /h.Istheflowlaminar?(b)Forwhattubediameterwill
Redbe500?
3
Solution:Forwater,take998kg/m and0.001kg/ms.(a)Bothtanksurfaces
areatatmospheric pressureandhavenegligible velocity. Theenergyequation,when
neglectingminorlosses,reducesto:
z 0.3m h f

128 LQ
128(0.001kg/ms)(3.5m)Q

4
gd
(998kg/m3 )(9.81m/s 2 )(0.004m)4

m3
m3
Solvefor Q 5.3E6
0.019
Ans.(a)
s
h
Check Re d 4Q/( d ) 4(998)(5.3E6)/[ (0.001)(0.004)]
Re d 1675 laminar. Ans.(a)
(b)IfRed5004Q/(d)andzhf,wecansolveforbothQandd:
Re d 500
h f 0.3m

4(998kg/m3 )Q
, or Q 0.000394 d
(0.001kg/ms)d

128(0.001kg/ms)(3.5m)Q
, or Q 20600 d 4
3
2
4
(998kg/m )(9.81m/s )d

Combinethesetwotosolve for Q 1.05E6m 3 /s and d 2.67 mm

Ans.(b)

193

Solutions ManualFluid Mechanics, Seventh Edition

6.25FortheconfigurationshowninFig.P6.25,thefluidisethylalcoholat20C,and
3
thetanksareverywide.Findtheflowratethatoccurs,inm /h.Istheflowlaminar?
3

Solution:Forethanol,take789kg/m and0.0012kg/ms.Writetheenergy
equationfromupperfreesurface(1)tolowerfreesurface(2):

Fig.P6.25

V12

V22

p1
p

z1 2
z 2 h f , with p1 p2 andV1 V2 0
g 2g
g 2g
Then h f z1 z 2 0.9m

128 LQ 128(0.0012)(1.2m)Q

gd 4
(789)(9.81)(0.002)4

Solvefor Q 1.90E6m 3 /s 0.00684 m 3 /h.

Ans.

ChecktheReynoldsnumberRe4Q/(d)795OK,laminarflow.

P6.26Twooiltanksareconnectedby

za=22m

zb=
15m

two9mlongpipes,asinFig.P6.26.
Pipe1is5cmindiameterandis6m
higherthanpipe2.Itisfoundthatthe
flowrateinpipe2istwiceaslargeas
theflowinpipe1.(a)Whatisthediameter

D1=5cm
SAE30W

D2

oilat
20C

L=9m

Fig.P6.26

6m

194

Chapter 6Viscous Flow in Ducts

ofpipe2?(b)Arebothpipeflowslaminar?
(c)Whatistheflowrateinpipe2(m3/s)?
Neglectminorlosses.

Solution:(a)Ifweknowtheflowsarelaminar,and(L,,)areconstant,thenQD4:
From Eq. (6.12),

Q2
D
2.0 ( 2 ) 4 , hence D2 (5 cm)(2.0)1 / 4 5.95 cm
Q1
D1

Ans.(a)

Wewillchecklaterinpart(b)tobesuretheflowsarelaminar.[Placingpipe1sixmeters
higherwasmeanttobeaconfusingtrick,sincebothpipeshaveexactlythesameheadloss
andz.](c)FindtheflowratefirstandthenbacktracktotheReynoldsnumbers.ForSAE
30Woilat20C(TableA.3),take =891kg/m3 and =0.29kg/ms. Fromtheenergy
equation,withV1=V2=0,andEq.(6.12)forthelaminarheadloss,

z 22 15 7 m h f

128LQ

gD24

Solve for

128(0.29kg / m s )(9m) Q2

(891kg / m 3 )(9.81m / s 2 )(0.0595m) 4


Q2 0.0072 m 3 /s

Ans.(c )

In a similar manner, insert D1 = 0.05m and compute Q1 = 0.0036 m3/s = (1/2)Q1.


(b)NowgobackandcomputetheReynoldsnumbers:

Re1

4 Q1
4 Q2
4(891)(0.0036)
4(891)(0.0072)

281 ; Re 2

473 Ans.(b)
D1
(0.29)(0.050)
D2
(0.29)(0.0595)

Both flows are laminar, which verifies our flashy calculation in part (a).

195

Solutions ManualFluid Mechanics, Seventh Edition

6.27LetusattackProb.6.25insymbolicfashion,usingFig.P6.27.Allparameters are
constantexcepttheuppertankdepthZ(t).Findanexpressionfortheflowrate Q(t)asa
functionofZ(t).Setupadifferentialequation,andsolveforthetimet0todraintheupper
tankcompletely.Assumequasisteadylaminarflow.
Solution:TheenergyequationofProb.6.25, using symbols only, is combined with a
controlvolumemassbalanceforthetanktogivethebasicdifferentialequationforZ(t):

Fig.P6.27

energy: h f

32 LV
d 2

h Z; massbalance:
D Z d 2 L Q d 2 V,
2

dt 4
4
4
gd
or:

2 dZ

gd 2
D
d 2 V, where V
(h Z)
4
dt
4
32 L

Separatethevariablesandintegrate,combiningalltheconstantsintoasingleC:
Z

dZ
gd 4
Ct

C
dt,
or:
Z

(h

Z
)e

h
,
where
C

o
hZ

32 LD2
Zo
0
TankdrainscompletelywhenZ 0, att 0

Z
1
ln 1 o
C
h

Ans.

Ans.

196

Chapter 6Viscous Flow in Ducts

6.28Forstraighteningandsmoothingan
airflowina50cmdiameterduct,theduct
ispackedwithahoneycombofthinstraws
oflength30cmanddiameter4mm,asin
Fig.P6.28.Theinletflowisairat110kPa
and20C,movingatanaveragevelocityof
6 m/s. Estimate the pressure drop across
thehoneycomb.

eachonewouldseetheaveragevelocityof
6m/s.Thus

Solution:For air at 20C, take


3
1.8E5kg/msand 1.31kg/m .There
wouldbeapproximately12000straws,but
Fig.P6.28

p laminar

32 LV 32(1.8E5)(0.3)(6.0)

65Pa
d2
(0.004)2

Ans.

CheckReVd/(1.31)(6.0)(0.004)/(1.8E5)1750OK,laminarflow.

P6.29
SAE 30W oil at 20C flows through a straight pipe 25 m long, with
diameter 4 cm. The average velocity is 2 m/s. (a) Is the flow laminar? Calculate (b) the
pressure drop; and (c) the power required. (d) If the pipe diameter is doubled, for the same
average velocity, by what percent does the required power increase?

Solution: For SAE 30W oil at 20C, Table A.3, = 891 kg/m3, and = 0.29 kg/m-s. (a)
We have enough information to calculate the Reynolds number:
Re D

VD
(891)(2.0)(0.04)

246 2300

0.29

Yes, laminar flow Ans.(a )

(b, c) The pressure drop and power follow from the laminar formulas of Eq. (6.12):

197

Solutions ManualFluid Mechanics, Seventh Edition

32 LV
D

32(0.29)(25)(2.0)
(0.04) 2

290, 000 Pa

Ans.(b)

m3
D V (0.04)2 (2.0) 0.00251
4
4
s
3
m
Power Q p (0.00251 )(290, 000 Pa) 729 W Ans.(c)
2
(d) If D doubles to 8 cm and V remains the same at 2.0 m/s, the new pressure drop will be
72,500 Pa, and the new flow rate will be Q = 0.01005 m3/s, hence the new power will be
P = Q p = (0.01005)(72,500) = 729 W
Zero percent change!
Q

This is because D2 cancels in the product P = Q p = 8 L V2.


NOTE: The flow is still laminar, ReD = 492.

6.30SAE10oilat20Cflowsthrough
the 4cmdiameter vertical pipe of
Fig. P6.30. For the mercury manometer
readingh42cmshown,(a)calculatethe
3
volumeflowrateinm /h,and(b)statethe
directionofflow.

Ans.(d)

Fig.P6.30

Solution:For SAE 10 oil, take


3
870 kg/m and 0.104 kg/ms. The
pressure at the lower point (1) is
considerablyhigherthanp2accordingtothe
manometerreading:
p1 p2 ( Hg oil )gh (13550 870)(9.81)(0.42) 52200Pa
p/(oil g) 52200/[870(9.81)] 6.12m
Thisismorethan3mofoil,thereforeitmustincludeafrictionloss:flowisup.Ans.(b)
Theenergyequationbetween(1)and(2),withV1V2,gives
p1 p2
128 LQ
z 2 z1 h f , or 6.12m 3m h f , or: h f 3.12m
g
gd 4
Compute Q

(6.12 3) (870)(9.81)(0.04)4
m3
m3
0.00536
19.3
128(0.104)(3.0)
s
h

Ans.(a)

CheckRe 4Q/(d) 4(870)(0.00536)/[ (0.104)(0.04)] 1430 (OK,laminarflow).

198

Chapter 6Viscous Flow in Ducts

P6.31 AlaminarflowelementorLFE(MeriamInstrumentCo.)measureslowgasflowrates
withabundleofcapillarytubespackedinsidealargeoutertube.Consideroxygenat20Cand
1atmflowingat84ft3/minina4indiameterpipe. (a)Istheflowapproachingtheelement
turbulent? (b)Ifthereare1000capillarytubes, L =4in,selectatubediametertokeepRe d
below1500andalsotokeepthetubepressuredropnogreaterthan0.5lbf/in 2.(c)Dothetubes
selectedinpart(b)fitnicelywithintheapproachpipe?

Solution:Foroxygenat20Cand1atm(TableA.4),takeR=260m2/(s2K),hence=p/RT=
(101350Pa)/[260(293K)]=1.33kg/m3=0.00258slug/ft3.Alsoread=2.0E5kg/ms=4.18E
7slug/fts.ConvertQ=84ft3/min=1.4ft3/s.ThentheentrypipeReynoldsnumberis

Re D

VD
4 Q

4(0.00258slug / ft 3 )(1.4 ft 3 / s )

( 4.18 E 7 slug / ft s )( 4 / 12 ft )

33,000

( turbulent ) Ans.( a )

(b)TokeepRedbelow1500andkeepthe(laminar)pressuredropnomorethan72psf(0.5psi),
Re d

Vd
1500 and

32 LV
d

72

lbf
ft

where V

Q / 1000
( / 4) d 2

Selectvaluesofdanditerate,oruseEES.TheupperlimitonReynoldsnumbergives
Re d 1500

if

d 0.00734 ft 0.088 in ;

p 2.74 lbf / ft 2

Ans.(b)

Thisisasatisfactoryanswer,sincethepressuredropisnoproblem,quitesmall. One
thousandofthesetubeswouldhaveanareaaboutonehalfofthepipearea,sowouldfit
nicely.Ans.(c)
Increasing the tube diameter would lower Red and have even smaller pressure drop.
Example:d=0.01ft,Red=1100,p=0.8psf.These0.01ftdiametertubeswouldjust
barelyfitintothelargerpipe.Onedisadvantage,however,isthatthesetubesareshort:

199

Solutions ManualFluid Mechanics, Seventh Edition

the entrance length is longer than the tube length, and thus p will be larger than
calculatedbyfullydevelopedformulas.

6.32SAE30oilat20Cflowsinthe3
cmdiameter pipe in Fig. P6.32, which
slopes at 37. For the pressure measure
ments shown, determine (a) whether the
flowisupordownand(b)theflowrate
3
inm /h.

Fig.P6.32

Solution:ForSAE30oil,take 891
3
kg/m and 0.29kg/ms.Evaluatethe
hydraulicgradelines:
HGL B

pB
180000
500000
zB
15 35.6m; HGL A
0 57.2m
g
891(9.81)
891(9.81)
Since HGL A HGL B theflowisup

Ans.(a)

Theheadlossisthedifferencebetweenhydraulicgradelevels:
h f 57.2 35.6 21.6m

128 LQ
128(0.29)(25)Q

gd 4
(891)(9.81)(0.03)4

Solvefor Q 0.000518m 3 /s 1.86 m 3 /h

Ans.(b)

Finally,checkRe4Q/(d)68(OK,laminarflow).

P6.33
Water at 20C is pumped from a reservoir through a vertical tube 10 ft
th
long and 1/16 inch in diameter. The pump provides a pressure rise of 11 lbf/in2 to the
flow. Neglect entrance losses. (a) Calculate the exit velocity. (b) Approximately how
high will the exit water jet rise? (c) Verify that the flow is laminar.
Solution: For water at 20C, Table A.3, = 998 kg/m3 = 1.94 slug/ft3, and = 0.001
kg/m-s = 2.09E-5 slug/ft-s. The energy equation, with 1 at the bottom and 2 at the top of
the tube, is:

200

Chapter 6Viscous Flow in Ducts

p1
V12
p2
V22
V22
32 LV2
11(144)

z1
00

z2 h f 0
10
g
2g
1.94(32.2)
g
2g
2g
gD 2
2
2
Vexit
Vexit
32(0.0000209)(10)V
or : 25.4
10
; or : 15.4 ft
3.94 Vexit
2(32.2)
64.4
(1.94)(32.2)(0.00521) 2

(a, c) The velocity head is very small (<1 ft), so the dominant term is 3.94 Vexit. One can
easily iterate, or simply use EES to find the result:
Vexit 3.84

ft
s

Ans.(a ) ; Re D

VD (1.94)(3.84)(0.00521)

1860 laminar Ans.(c)

0.0000209

(b) Assuming frictionless flow outside the tube, the jet would rise due to the velocity
head:
H rise

2
Vexit
(3.84 ft / s ) 2

0.229 ft 2.75 inches


2g
2(32.2 ft / s 2 )

Ans.(b)

6.34Derivethetimeaveraged xmomentumequation(6.21)bydirectsubstitutionof
Eqs.(6.19)intothemomentumequation(6.14).Itisconvenienttowritetheconvective
accelerationas
du 2

(u )
(uv) (uw)
dt x
y
z
whichisvalidbecauseofthecontinuityrelation,Eq.(6.14).

Solution:Intothexmomentumeqn.substituteuuu,vvv,etc.,toobtain
2

(u 2uu u2 )
(vu vu vu vu) (wu wu wu wu)
y
z
x

(p p) g x [ 2 (u u)]
x

201

Solutions ManualFluid Mechanics, Seventh Edition

NowtakethetimeaverageoftheentireequationtoobtainEq.(6.21)ofthetext:
du

u2
uv
uw
g x 2 u
y
z
x
dt x

Ans.

P6.35
In the overlap layer of Fig. 6.9a, turbulent shear is large. If we neglect
viscosity, we can replace Eq. (6.24) by the approximate velocity-gradient function
du
fcn( y , w , )
dy
Show that, by dimensional analysis, this leads to the logarithmic overlap relation (6.28).
Solution: There are four variables, and we may list their dimensions in the (MLT)
system:
du
y
w

dy
{T 1}

{L}

{ML1T 2 }

{ML3 }

These can be formed into a single pi group that is therefore equal to a dimensionless
constant:
du
du y
1 y a wb c
yields 1
(
) constant C1
dy
dy w
Rearrange this into a differential equation and then integrate:

1
du
v*
C1 w
C1
; Integrate : u C1 v *ln( y ) C2
dy
y
y

Ans.

We recognize the square-root term as the friction velocity v* from Eq. (6.25). If the
constants are rearranged so that the logarithm has a dimensionless argument, we would
obtain Eq. (6.28):
yv*
u
1

ln(
) B
v*

202

Chapter 6Viscous Flow in Ducts

6.36Thefollowingturbulentflowvelocitydatau(y),forairat75Fand1atmneara
smoothflatwall,weretakenintheUniversityofRhodeIslandwindtunnel:
y,in:

0.025

0.035

0.047

0.055

0.065

u,ft/s:

51.2

54.2

56.8

57.6

59.1

Estimate(a)thewallshearstressand(b)thevelocityuaty0.22in.

Solution:Forairat75Fand1atm,take0.00230slug/ft and3.80E7slug/fts.
Wefiteachdatapointtothelogarithmicoverlaplaw,Eq.(6.28):
0.0023u*y
u 1 u*y
1
ln
B
ln
5.0, u* w /
u*

0.41 3.80E7
Entereachvalueofuandyfromthedataandestimatethefrictionvelocityu*:
y,in:
u*,ft/s:
yu*/(approx):

0.025
3.58
45

0.035
3.58
63

0.047
3.59
85

0.055
3.56
99

0.065
3.56
117

Eachpointgivesagoodestimateofu*,becauseeachpointiswithinthelogarithmiclayer
inFig.6.10ofthetext.Theoverallaveragefrictionvelocityis
u*avg 3.57

ft
lbf
1%, w,avg u*2 (0.0023)(3.57)2 0.0293 2
s
ft

Ans.(a)

(b)Outaty0.22inches,wemayestimatethattheloglawstillholds:

u*y 0.0023(3.57)(0.22/12)
1

396, u u*
ln(396) 5.0

3.80E 7
0.41

ft
or: u (3.57)(19.59) 70
Ans.(b)
s

Figure6.10showsthatthispoint(y 396)seemsalsotobewithinthelogarithmiclayer.

203

Solutions ManualFluid Mechanics, Seventh Edition

6.37Twoinfiniteplatesadistancehapartareparalleltothexzplanewiththeupper
platemovingatspeedV,asinFig.P6.37.Thereisafluidofviscosityandconstant
pressurebetweentheplates.Neglectinggravityandassumingincompressibleturbulent
flowu(y)betweentheplates,usethelogarithmiclawandappropriate
boundary conditions to derive a formula for dimensionless wall shear stress versus
dimensionlessplatevelocity.Sketchatypicalshapeoftheprofileu(y).

Fig.P6.37

Solution:The shear stress between parallel plates is constant, so the centerline


velocitymustbeexactlyuV/2atyh/2.Antisymmetricloglawsform,onewith
increasingvelocityfor0yh/2,andareversemirrorimageforh/2yh,asshown
below:

Thematchpointatthecentergivesusaloglawestimateoftheshearstress:

204

Chapter 6Viscous Flow in Ducts

V
1 hu*
ln
B, 0.41, B 5.0, u* ( w )12

2u* 2

Ans.

Thisisoneformofdimensionlessshearstress.Themorenormalformisfriction
coefficientversusReynoldsnumber.CalculationsfromtheloglawfitaPowerlaw
curvefitexpressionintherange2000Reh1E5:
Cf

w
0.018
0.018

(1/2)V 2 (Vh/ )1/4 Re1h4

Ans.

6.38SupposeinFig.P6.37thath3cm,thefluidiswaterat20C(998kg/m ,
0.001kg/ms),andtheflowisturbulent,sothatthelogarithmiclawisvalid.Ifthe
shearstressinthefluidis15Pa,estimateVinm/s.
Solution:JustasinProb.6.37,applytheloglawatthecenterbetweenthewall,that
is,yh/2,uV/2.Withwknown,wecanevaluateu*immediately:
u*
or:

w
15
m V /2 1 u * h/2

0.123 ,
ln
B,

998
s
u*

0.123(0.03/2)
V /2
1
m

ln
5.0 23.3, Solve for V 5.72

0.123 0.41 0.001/998


s

Ans.

6.39By analogy with laminar shear, du/dy. T. V. Boussinesq in 1877


postulatedthatturbulentshearcouldalsoberelatedtothemeanvelocitygradient turb
du/dy,where is called the eddy viscosity and is much larger than . Ifthe
logarithmicoverlaplaw,Eq.(6.28),isvalidwithw,showthatu*y.
Solution:Differentiate the loglaw, Eq. (6.28), to find dudy, then introduce the
eddyviscosityintotheturbulentstressrelation:
If

u
1 yu
du u*
ln

B, then
u*
dy y

Then, if w u *2

du
u*

, solve for u * y
dy
y

Notethat/=y+,whichismuchlargerthanunityintheoverlapregion.

Ans.

205

Solutions ManualFluid Mechanics, Seventh Edition

6.40Theodore von Krmn in 1930 theorized that turbulent shear could be


2 2
representedby turbdudywhere y du/dy iscalledthemixinglengtheddy
viscosity and 0.41 is Krmns dimensionless mixinglength constant [2,3].
Assumingthat turb wnearthewall,showthatthisexpressioncanbeintegratedto
yieldthelogarithmicoverlaplaw,Eq.(6.28).
Solution:Thisisaccomplishedbystraightsubstitution:

turb w u*2
Integrate:

u*

du
du du
du u*
2 y 2
, solvefor

dy
dy dy
dy y

du

dy
u*
, or: u ln(y) constant
y

Ans.

ToconvertthistotheexactformofEq.(6.28)requiresfittingtoexperimentaldata.

P6.41Tworeservoirs,whichdifferinsurfaceelevationby40m,areconnectedby350mof
newpipeofdiameter8cm.Ifthedesiredflowrateisatleast130N/sofwaterat20 C,may
thepipematerialbe(a)galvanizediron,(b)commercialsteel,or(c)castiron?Neglectminor
losses.

Solution:ApplyingtheextendedBernoulliequationbetweenreservoirsurfacesyields

40 m

L V2
D 2g

f (

350 m
V2
)
0.08 m 2(9.81 m / s 2 )

wherefandVarerelatedbythefrictionfactorrelation:
1
f

2.0 log10 (

/D
2.51

)
3.7
Re D f

where

Re D

VD

206

Chapter 6Viscous Flow in Ducts

WhenVisfound,theweightflowrateisgivenbyw=gQwhereQ=AV=(D2/4)V.
Forwaterat20C,take=998kg/m3and=0.001kg/ms.Giventhedesiredw=130N/s,
solvethissystemofequationsbyEEStoyieldtheallowedwallroughness.Theresultsare:
f=0.0257;V=2.64m/s;ReD=211,000;max=0.000203m=0.203mm
Any less roughness is OK. From Table 6-1, the three pipe materials have
(a)galvanized:=0.15mm;(b)commercialsteel:=0.046mm;castiron:=0.26mm
Galvanizedandsteelarefine,butcastironistoorough..Ans.Actualflowratesare
(a)galvanized:135N/s;(b)steel:152N/s;(c)castiron:126N/s(notenough)

P6.42

Fluid flows steadily, at volume rate Q, through a large horizontal pipe and
then divides into two small smooth pipes, the larger of which has an inside diameter of 25
mm and carries three times the flow of the smaller pipe. Both small pipes have the same
length and pressure drop. If all flows are turbulent, at Red near 104, estimate the diameter
of the smaller pipe.
Solution: For turbulent flow, the formulas are algebraically complicated, such as Eq.
(6.38). However, in the low Reynolds number region, the Blasius power-law
approximation, Eq. (6.39), applies, leading to a simple approximate formula for pressure
drop, Eq. (6.41):
p 0.241 L 3/ 4 1/ 4 d 4.75 Q1.75
Since p, L, , and are the same for both pipes, it follows that
Q1.75 d 4.75 , or : Qlow turbulent d 4.75 /1.75 d 19 / 7
Thus, Q1 const (d1 )19 / 7 3Q2 3(const )(d 2 )19 / 7 ,
or : d 2

d1
37 /19

25 mm
16.7 mm
1.499

Ans.

This is slightly smaller than the laminar-flow estimate of Prob. P6.29, where d2 19 mm.

207

Solutions ManualFluid Mechanics, Seventh Edition

P6.43Areservoirsupplieswaterthrough

z1=35m

100mof30cmdiametercastironpipetoa
turbinethatextracts80hpfromtheflow.

water
at20C
z2=5m

turbine

Thewaterthenexhauststotheatmosphere.

Fig.P6.43

Neglectminorlosses.(a)Assumingthat
f0.019,findtheflowrate(thereisacubic
polynomial).Explainwhytherearetwosolutions.
(b)Forextracredit,solvefortheflowrateusingtheactualfrictionfactors.

Solution:Forwaterat20C,take=998kg/m3and=0.001kg/ms.Theenergy
equationyieldsarelationbetweenelevation,friction,andturbinepower:
p1
V2
p
V2
1 z1 2 2 z 2 hturb h f
g
2g
g
2g
z1 z 2 35 5m 30m hturb h f
30 m

(80 hp)(745.7W / hp )
(9790 N / m 3 )( / 4)(0.3m) 2 V2

Power
L V22
2
(1 f
)
, Q
D V2
gQ
D 2g
4

[1 (0.019)

V22
100m
]
]
0.3 m 2(9.81m / s 2 )

Cleanthisupintoacubicpolynomial:
86.2
0.373V 2 ,
or : V 3 80.3V 231 0
V
Three roots :
V 3.34 m / s ; 6.81 m / s ; 10.15 m / s
30

208

Chapter 6Viscous Flow in Ducts

The third (negative) root is meaningless. The other two are correct. Either
Q=0.481m3/s,hturbine=12.7m,hf=17.3m
Q=0.236m3/s,hturbine=25.8m,hf=4.2mAns.(a)
Both solutions are valid. The higher flow rate wastes a lot of water and creates 17 meters of
friction loss. The lower rate uses 51% less water and has proportionately much less friction.
(b) The actual friction factors are very close to the problems Guess. Thus we obtain
Re=2.04E6,f=0.0191;Q=0.479m3/s,hturbine=12.7m,hf=17.3m
Re=1.01E6,f=0.0193;Q=0.237m 3/s,hturbine=25.7m,h f=4.3m
Ans.(b)
The same remarks apply: The lower flow rate is better, less friction, less water used.

6.44Mercuryat20Cflowsthrough4metersof7mmdiameterglasstubingatan
averagevelocityof5m/s.Estimatetheheadlossinmetersandthepressuredropin
kPa.
3

Solution:Formercuryat20C,take 13550kg/m and 0.00156kg/ms.


Glasstubingisconsideredhydraulicallysmooth, /d 0.ComputetheReynolds
number:
Red

Vd 13550(5)(0.007)

304,000; Moodychartsmooth: f 0.0143

0.00156
hf f

L V2
52
4.0
0.0143
10.4 m

d 2g
0.007 2(9.81)

Ans.(a)

p gh f (13550)(9.81)(10.4) 1,380,000Pa 1380 kPa


2

Ans.(b)

6.45Oil,SG 0.88and 4E5m /s,flowsat400gal/min througha6inch


asphaltedcastironpipe.Thepipeis0.5mileslong(2640ft)andslopesupwardat8
intheflowdirection.Computetheheadlossinfeetandthepressurechange.

209

Solutions ManualFluid Mechanics, Seventh Edition

Solution:First convert 400 gal/min 0.891 ft /s and 0.000431 ft /s. For


asphaltedcastiron,0.0004ft,hence/d0.0004/0.50.0008.ComputeV,Red,
andf:
V

0.891
ft
4.54(0.5)

4.54
;
Re

5271; calculate
d
s
0.000431
(0.25)2
then h f f

2
L V2
2640 (4.54)
0.0377
63.8ft
0.5
2(32.2)
d 2g

f Moody 0.0377
Ans.(a)

Ifthe pipeslopes upwardat 8,thepressuredropmust balance bothfriction and


gravity:
p g(h f z ) 0.88(62.4)[63.8 2640 sin 8] 23700

lbf
ft 2

Ans.(b)

P6.46
Repeat Prob. P3.5, for the same data, by using the more exact turbulent
flow formulas to compute the volume flow rate in gallons per minute. Recall the
problem: Water at 20C flows through a 5-inch-diameter smooth pipe at a centerline
velocity of 25 ft/s. Estimate the volume flow rate in gallons per minute.
Solution: For water at 20C, Table A.3, = 998 kg/m3 = 1.94 slug/ft3, and = 0.001
kg/m-s = 2.09E-5 slug/ft-s. A bit of iteration, or EES, is needed to get the proper
Reynolds number and friction factor. Our estimate in Prob. P3.5 was Vav 21 ft/s,
whence the Reynolds number is
VD (1.94 slug / ft 3 )(21 ft / s)(5 /12 ft )
Re D

812, 000

2.09 E 5 slug / ft s
Then estimate f smooth 0.0121 from Eq.(6.48)
Eq.(6.43) : Vav umax (1 1.3 f ) 1 (25 ft / s )(0.875) 21.9 ft / s
Iterate once to obtain fsmooth = 0.0120, Vav = 21.89 ft/s, hardly any change at all.
2.5 2
ft 3
gal
s
gal
Q Vav A pipe (21.89 ft / s ) (
ft ) 2.98
7.48 3 60
1340
12
s
min
min
ft
This is about 4.6% greater than our simple power-law estimate in Prob. P3.5.

Ans.

210

Chapter 6Viscous Flow in Ducts

6.47ThegutterandsmoothdrainpipeinFig.P6.47removerainwaterfromtheroofofa
building.Thesmoothdrainpipeis7cmindiameter.(a)Whenthegutterisfull,estimate
therateofdraining.(b)Thegutterisdesignedforasuddenrainstormofupto5inches
per hour. For this condition, what is the maximum roof area that can be drained
successfully?

Solution:Ifthevelocityattheguttersurfaceisneglected,theenergyequationreducesto

Fig.P6.47

V2
L V2
2 gz
2(9.81)(4.2)
hf , hf f
, solve V 2

2g
d 2g
1 fL/d 1 f (4.2/0.07)
3

Forwater,take998kg/m and0.001kg/ms.Guessf0.02toobtainthevelocity
estimate V 6 m/s above. Then Red Vd/ (998)(6)(0.07)/(0.001) 428,000
(turbulent).Then,forasmoothpipe,f0.0135,andVischangedslightlyto6.74m/s.
Afterconvergence,weobtain
V 6.77m/s, Q V ( /4)(0.07)2 0.026m 3 /s

Ans.(a)

211

Solutions ManualFluid Mechanics, Seventh Edition

Arainfallof5in/h(5/12ft/h)(0.3048m/ft)/(3600s/h)0.0000353m/s.Therequired
roofareais
Aroof Qdrain /Vrain (0.026m 3 /s)/0.0000353m/s 740m 2 Ans.(b)
6.48ShowthatifEq.(6.33)isaccurate,thepositioninaturbulentpipeflowwhere
localvelocity uequalsaveragevelocityVoccursexactlyatr0.777R,independentof
theReynoldsnumber.

Solution:Simplyfindtheloglawpositiony whereu exactlyequalsV/u*:


3
1 Ru*
ln
B

?
1 yu*
1 y
3
u*
ln B if ln R 2
r
Since y R r,thisisequivalentto 1 e 3/2 1 0.223 0.777 Ans.
R

V u*

6.49ThetankpipesystemofFig.P6.49istodeliveratleast11m /hofwaterat20Cto
thereservoir.Whatisthemaximumroughnessheightallowableforthepipe?
3

Solution:Forwaterat20C,take998kg/m and0.001kg/ms.EvaluateVand
Refortheexpectedflowrate:

Fig.P6.49

Q
11/3600
m
Vd 998(4.32)(0.03)

4.32 ; Re

129000
2
A ( /4)(0.03)
s

0.001

Theenergyequationyieldsthevalueoftheheadloss:
patm V12
patm V22

z2 h f
g 2g 1 g 2g

(4.32)2
or h f 4
3.05m
2(9.81)

2
L V2
5.0 (4.32)
, or: 3.05 f
, solvefor f 0.0192
0.03
2(9.81)
d 2g
WithfandReknown,wecanfind/dfromtheMoodychartorfromEq.(6.48):

Butalso h f f

212

Chapter 6Viscous Flow in Ducts

/d

1
2.51

2.0 log10

, solvefor 0.000394
1/2
1/2
d
(0.0192)
3.7 129000(0.0192)
Then 0.000394(0.03) 1.2E5 m 0.012 mm (pretty smooth)

Ans.

6.50Ethanolat20Cflowsat125U.S.gal/minthroughahorizontalcastironpipewith
L12mandd5cm.Neglectingentranceeffects,estimate(a)thepressuregradient,dp/dx;
(b)thewallshearstress, w;and(c)thepercentreductioninfrictionfactorifthepipe
wallsarepolishedtoasmoothsurface.

Solution:Forethanol(TableA3)take789kg/m and0.0012kg/ms.Convert
3
2
125gal/minto0.00789m /s.EvaluateVQ/A0.00789/[ (0.05) /4]4.02m/s.
Red

Vd 789(4.02)(0.05)
0.26mm

132,000,

0.0052 Thenf Moody 0.0314

0.0012
d
50mm
f
0.0314
V 2
(789)(4.02)2 50Pa Ans.(b)
8
8
4
dp
4(50)
Pa
w
4000
Ans.(a)
dx
d
0.05
m

(b) w
(a)

(c) Re 132000,

fsmooth 0.0170, hencethereductioninfis


0.0170
46% Ans.(c)
0.0314

6.51The viscous sublayer (Fig. 6.10) is normally less than 1 percent of the pipe
diameterandthereforeverydifficulttoprobewithafinitesizedinstrument.Inaneffort
togenerateathicksublayerforprobing,PennsylvaniaStateUniversityin1964builta
pipewithaflowofglycerin.Assumeasmooth12indiameterpipewithV60ft/sand
glycerinat20C.Computethesublayerthicknessininchesandthepumpinghorsepower
requiredat75percentefficiencyifL40ft.

213

Solutions ManualFluid Mechanics, Seventh Edition

Solution:Forglycerinat20C,take2.44slug/ft and0.0311slug/fts.Then
Re

Vd 2.44(60)(1ft)

4710(barelyturbulent!) Smooth: fMoody 0.0380

0.0311
0.0380

Then u* V(f/8)1/2 60

1/2

4.13

ft
s

Thesublayerthicknessisdefinedbyy 5.0yu*/.Thus
ysublayer

5
5(0.0311)

0.0154ft 0.185inches
u* (2.44)(4.13)

Ans.

Withfknown,theheadlossandthepowerrequiredcanbecomputed:
hf f
P

2
L V2
40 (60)
(0.0380)
85ft
d 2g
1 2(32.2)

gQh f
1

(2.44)(32.2) (1)2 (60) (85) 419000 550 760hp Ans.

0.75

6.52The pipe flow in Fig. P6.52 is


drivenbypressurizedairinthetank.What
gage pressure p1 is needed to provide a
3
20CwaterflowrateQ60m /h?

Fig.P6.52

Solution:For water at 20C, take


3
998kg/m and 0.001kg/ms.GetV,
Re,f:
60/3600
m
8.49 ;
2
s
( /4)(0.05)
998(8.49)(0.05)
Re
424000; fsmooth 0.0136
0.001
V

Writetheenergyequationbetweenpoints(1)(thetank)and(2)(theopenjet):
2
p1 02
0 Vpipe
L V2
m

10

80 h f , where h f f
and Vpipe 8.49
g 2g
g 2g
d 2g
s

214

Chapter 6Viscous Flow in Ducts

(8.49)2
170
Solve p1 (998)(9.81) 80 10
1 0.0136

2(9.81)
0.05

2.38E6Pa Ans.
[Thisisagagepressure(relativetothepressuresurroundingtheopenjet.)]

P6.53
Water at 20C flows by gravity through a smooth pipe from one reservoir
to a lower one. The elevation difference is 60 m. The pipe is 360 m long, with a diameter
of 12 cm. Calculate the expected flow rate in m3/h. Neglect minor losses.
Solution: For water at 20C, Table A.3, = 998 kg/m3 and = 0.001 kg/m-s. With no
minor losses, the gravity head matches the Moody friction loss in the pipe:
z 60 m h f
where

1
f

360 m
L V2
V2

f
f(
)
, Q D2 V
2
D 2g
0.12 m 2(9.81m / s )
4

2.0 log10 (

Re D f
) ,
2.51

Re D

(998) V (0.12)
VD

0.001

The unknown is V, since f can be found as soon as the Reynolds number is known. You could
iterate your way to the answer by, say, guessing f = 0.02, getting V, repeating. Or you could
put the above four equations into EES, which will promptly return the correct answer:
V 5.61

m
m3
m3
; Re D 672, 000 ; f 0.0125 ; Q 0.0634
228
s
s
h

Ans.

6.54*AswimmingpoolWbyYbyhdeepistobeemptiedbygravitythroughthelong
pipeshowninFig.P6.54.Assuminganaveragepipefrictionfactor fav andneglecting
minorlosses,deriveaformulaforthetimetoemptythetankfromaninitiallevelho.

Fig.P6.54

215

Solutions ManualFluid Mechanics, Seventh Edition

Solution:Withnodrivingpressureandnegligibletanksurfacevelocity,theenergy
equationcanbecombinedwithacontrolvolumemassconservation:
h (t )

V2
L V2

2 gh
dh
fav
, or: Qout ApipeV D 2
WY
2g
D 2g
4
1 fav L/D
dt

Wecanseparatethevariablesandintegratefortimetodrain:

2
2g
D
4
1 fav L/D

dt WY

Cleanthisuptoobtain: t drain

ho

4WY

D2

dh
WY 0 2 ho
h
2 ho (1 fav L D )
g

1/2

Ans.

216

Chapter 6Viscous Flow in Ducts

thus h f z1 z 2 100m

6.55ThereservoirsinFig.P6.55contain
waterat20C.IfthepipeissmoothwithL
4500mandd4cm,whatwilltheflow
3
rateinm /hbeforz100m?
Solution:For water at 20C, take
3
998kg/m and0.001kg/ms.Theenergy
equationfromsurface1tosurface2gives
p1 p2

and V1 V2 ,

Fig.P6.55
2

V
4500
, or fV 2 0.01744

0.04 2(9.81)

Then100m f

Iteratewithaninitialguessoff0.02,calculatingVandReandimprovingtheguess:
0.01744
0.02

1/2

0.934

0.01744
Vbetter
0.0224

m
998(0.934)(0.04)
, Re
37300, fsmooth 0.0224
s
0.001

1/2

0.883

m
, Re better 35300, fbetter 0.0226, etc......
s

Alternately,onecould,ofcourse,useEES.Theaboveprocessconvergesto
f 0.0227, Re 35000, V 0.877m/s,Q 0.0011m 3 /s 4.0m 3 /h. Ans.

P6.56
The Alaska Pipeline in the chapter-opener photo has a design flow rate of
4.4E7 gallons per day of crude oil at 60C (see Fig. A.1). (a) Assuming a galvanized-iron
wall, estimate the total pressure drop required for the 800-mile trip. (b) If there are nine
equally spaced pumps, estimate the horsepower each pump must deliver.
Solution: From Fig. A.1 for crude oil at 60C, = 860 kg/m3 and = 0.004 kg/m-s. The
pipe diameter is 4 ft. For galvanized iron, = 0.0005 ft, hence /D = 0.0005/4 =
0.000125. Convert the data to metric: 4.4E7 gal/day = 1.93 m3/s, and D = 4 ft = 1.22 m.
Calculate Reynolds number:
V

1.93 m3 / s
Q
m
VD (860)(1.65)(1.22)

1.65
;
Re

433, 000
D
A ( / 4)(1.22m) 2
s

0.004

Calculate the friction factor from Eq. (6.48):

217

Solutions ManualFluid Mechanics, Seventh Edition

1
f

2.0 log10 (

0.000125
2.51

)
3.7
433000 f

Calculate

f 0.0149

(a) The total 800-mile pressure drop is given by the usual Darcy-Moody expression, Eq. (6.10):
p f

L 2
(800)(5280) 860
V (0.0149)[
](
)(1.65) 2 1.85E7 Pa 2680 psi
D 2
4 ft
2

Ans.(a)

(b) The power delivered by each of the 9 pumps is


Power Q

ptotal
m3 1.85E7 Pa
(1.93 )(
) 396, 000 W 746 5300 hp
9
s
9

Ans.(b)

6.57ApplytheanalysisofProb.6.54tothefollowingdata.LetW5m,Y8m,ho
2m,L15m,D5cm,and0.(a)Bylettingh1.5mand0.5masrepresentative
depths,estimatetheaveragefrictionfactor.Then(b)estimatethetimetodrainthepool.
3

Solution:Forwater,take998kg/m and0.001kg/ms.ThevelocityinProb.6.54
iscalculatedfromtheenergyequation:
V

2 gh
1 fL/D

with f fcn(Re D )smoothpipe and Re D

VD
, L/D 300

(a)Withabitofiteration fortheMoodychart,weobtainRe D 108,000and f


0.0177ath1.5m,andRe D59,000andf.0202ath0.5m;thustheaverage
valuefav0.019.Ans.(a)
ThedrainformulafromProb.6.54thenpredicts:
t drain

4WY
D2

2ho (1 fav L/D)


4(5)(8)

g
(0.05)2

33700s 9.4h

2(2)[1 0.019(300)]
9.81

Ans.(b)

P6.58
For the system in Prob. P6.53, a pump is used at night to drive water back
to the upper reservoir. If the pump delivers 15,000 W to the water, estimate the flow rate.

218

Chapter 6Viscous Flow in Ducts

Solution: For water at 20C, Table A.3, = 998 kg/m3 and = 0.001 kg/m-s. Since the
pressures and velocities cancel, the energy equation becomes
zlower zupper h f h pump , or : h p
where

1
f

2.0 log10 (

360m V 2
z h f 60m f
0.12m 2(9.81)

Re D f
(998) V (0.12)
VD

) , Re D

, Q V D2
2.51

0.001
4

Power g Q h p (998)(9.81) Q h p 15, 000 W


The unknown is V, since f can be found as soon as the Reynolds number is known. You
could iterate your way to the answer by, say, guessing f = 0.02, getting V (from a cubic
equation!), and repeating. Or put the above five equations into EES, which returns the
only positive answer:
Re D 235, 000 ; f 0.0152 ; V 1.97 m / s ; Q 0.0222 m3 / s 80

m3
Ans.
h

6.59Thefollowingdatawereobtainedforflowof20Cwaterat20m /hrthroughabadly
corroded5cmdiameterpipewhichslopesdownwardatanangleof8 :p1420kPa,
z112m,p2250kPa,z23m.Estimate(a)theroughnessratioofthepipe;and(b)
thepercentchangeinheadlossifthepipeweresmoothandtheflowratethesame.
Solution:Thepipelengthisgivenindirectlyas L z/sin (9m)/sin8 64.7 m.
Thesteadyflowenergyequationthengivestheheadloss:
p1 V12
p V2
420000
250000

z1 2 2 z2 h f , or:
12
3 hf ,
g 2g
g 2g
9790
9790
Solve h f 26.4m
NowrelatetheheadlosstotheMoodyfrictionfactor:
h f 26.4 f

L V2
64.7 (2.83)2

f
, Solve f 0.050, Re 141000, Read 0.0211
d 2g
0.05 2(9.81)
d

Theestimated(anduncertain)piperoughnessisthus0.0211d1.06mmAns.(a)
(b)AtthesameRed141000,fsmooth0.0168,or66%lessheadloss.Ans.(b)

219

Solutions ManualFluid Mechanics, Seventh Edition

P6.60InthespiritofHaalandsexplicitpipefrictionfactorapproximation,Eq.(6.49),Jeppson
[20]proposedthefollowingexplicitformula:
1
f

2.0 log10 (

/d
5.74

)
3.7
Re 0d.9

(a)IsthisidenticaltoHaalandsformulaandjustasimplerearrangement?Explain.
(b)CompareJeppsontoHaalandforafewrepresentativevaluesof(turbulent)Re dand

/dandtheirdeviationscomparedtotheColebrookformula(6.48).
Solution: (a)No,it looks like arearrangement ofHaalands formula, but itis not.
HaalandstartedwithColebrookssmoothwallformulaandaddedjustenough /deffect
foraccuracy. JeppsonstartedwiththeroughwallformulaandaddedjustenoughRed
effectforaccuracy.Bothareexcellentapproximationsoverthefull(turbulent)rangeof
Redand/d.Theirpredictedvaluesoffarenearlythesameandveryclosetotheimplicit
Colebrookformula. Hereisatableoftheirstandarddeviationsoftheirvalueswhen
subtractedfromColebrook:
1E4<Red<1e8

/d=0.03

0.01

0.001

0.0001

0.00001

Jeppsonrmserror

0.000398

0.000328

0.000195

0.000067

0.000088

Haaland rms error

0.000034

0.000043

0.000129

0.000113

0.000083

Asexpected,Jeppsonisslightlybetterforsmoothwalls,Haalandforroughwalls.Both
arewithin2%oftheColebrookformulaovertheentirerangeofRedand/d.

220

Chapter 6Viscous Flow in Ducts

6.61WhatlevelhmustbemaintainedinFig.P6.61todeliveraflowrateof0.015ft /s
throughthe 12 in commercialsteelpipe?

Fig.P6.61
3

Solution:Forwaterat20C,take 1.94slug/ft and 2.09E5slug/fts.For


commercialsteel,take0.00015ft,or/d0.00015/(0.5/12)0.0036.Compute
V
Re

Q
0.015
ft

11.0 ;
2
A ( /4)(0.5/12)
s

Vd 1.94(11.0)(0.5/12)

42500 /d 0.0036, fMoody 0.0301

2.09E5

Theenergyequation,withp1p2andV10,yieldsanexpressionforsurfaceelevation:
h hf

V2 V2

2g 2g

80
L
(11.0)2
1

1 0.0301
111ft

0.5/12

d
2(32.2)

6.62Water at 20C is to be pumped


through2000ftofpipefromreservoir1to
3
2 at a rate of 3 ft /s, as shown in Fig.
P6.62.Ifthepipeiscastironofdiameter6
in and the pump is 75 percent efficient,
whathorsepowerpumpisneeded?
Solution:For water at 20C, take
3
1.94slug/ft and2.09E5slug/fts.For
cast iron, take 0.00085 ft, or /d
0.00085/(6/12) 0.0017.ComputeV,Re,
andf:
V

Q
3
ft

15.3 ;
2
A ( /4)(6/12)
s

Fig.P6.62

Ans.

221

Solutions ManualFluid Mechanics, Seventh Edition

Re

Vd 1.94(15.3)(6/12)

709000 /d 0.0017, fMoody 0.0227

2.09E5

Theenergyequation,withp1p2andV1V20,yieldsanexpressionforpumphead:
h pump z f
Power: P

2000 (15.3)2
L V2
120ft 0.0227
120 330 450 ft

d 2g
6/12 2(32.2)

gQh p 1.94(32.2)(3.0)(450)

112200 550 204hp Ans.

0.75
3

6.63Atankcontains1m ofwaterat20Candhasadrawncapillaryoutlettubeatthe
3
bottom,asinFig.P6.63.FindtheoutletvolumefluxQinm /hatthisinstant.
3

Solution:Forwaterat20C,take 998kg/m and 0.001kg/ms.Fordrawn


tubing,take 0.0015mm,or /d 0.0015/40 0.0000375.Thesteadyflowenergy
equation,withp1p2andV10,gives

Fig.P6.63

hf f

L V2
V2
V2
0.8
35.32
z
, or:
f 1.8m,V 2
1
d 2g
2g
2g
0.04
1 20f

35.32
Guess f 0.015, V

1 20(0.015)

1/2

5.21

m
998(5.21)(0.04)
, Re
208000
s
0.001

fbetter 0.0158, Vbetter 5.18m/s, Re 207000(converged)


Thus V 5.18m/s, Q ( /4)(0.04)2 (5.18) 0.00651m 3 /s 23.4m 3 /h. Ans.

222

Chapter 6Viscous Flow in Ducts

6.64RepeatProb.6.63tofindtheflowrateifthefluidisSAE10oil.Istheflow
laminarorturbulent?
3

Solution:ForSAE10oilat20C,take870kg/m and0.104kg/ms.Fordrawn
tubing,take0.0015mm,or/d0.0015/400.0000375.Guesslaminarflow:
V 2 ? 32 LV
V2
32(0.104)(0.8)V
h f 1.8m

, or: 1.8

0.195V
2
2g
2(9.81) 870(9.81)(0.04)2
gd
Quadraticequation: V 2 3.83V 35.32 0, solve V 4.33m/s
Check Re (870)(4.33)(0.04)/(0.104) 1450 (OK,laminar)
2

Soitislaminarflow,andQ(/4)(0.04) (4.33)0.00544m /s19.6m /h.Ans.


6.65InProb.6.63theinitialflowisturbulent.Asthewaterdrainsoutofthetank,will
theflowreverttolaminarmotionasthetankbecomesnearlyempty?Ifso,atwhattank
depth?Estimatethetime,inh,todrainthetankcompletely.
3
Solution:Recallthat998kg/m ,0.001kg/ms,and/d0.0000375.LetZbe
thedepthofwaterinthetank(Z1minFig.P6.63).WhenZ0,findtheflowrate:
Z 0, h f 0.8m, V 2

2(9.81)(0.8)
convergesto f 0.0171, Re 136000
1 20f

V 3.42m/s, Q 12.2m 3 /h(Z 0)


Soevenwhenthetankisempty,theflowisstillturbulent.Ans.
Thetimetodrainthetankis

d
d
dZ
(tank ) Q (A tank Z) (1m 2 )
Q,
dt
dt
dt
0m

or t drain

1m

dZ 1

Q Q

(1m)
avg

Soallweneedistheaveragevalueof(1/Q)duringthedrainingperiod.WeknowQatZ0
3
andZ 1m,letscheckitalsoatZ 0.5m:CalculateQmidway 19.8m /h.Then
estimate,bySimpsonsRule,
1
avg 1 1 4 1 0.0544 h3 , t drain 0.0544 h 3.3min
Q
6 23.4 19.8 12.2
m

Ans.

223

Solutions ManualFluid Mechanics, Seventh Edition

6.66Ethylalcoholat20Cflowsthrougha10cmhorizontaldrawntube100mlong.
Thefullydevelopedwallshearstressis14Pa.Estimate(a)thepressuredrop,(b)the
volumeflowrate,and(c)thevelocityuatr1cm.
3

Solution:Forethylalcoholat20C,789kg/m ,0.0012kg/ms.Fordrawntubing,
take0.0015mm,or/d0.0015/1000.000015.FromEq.(6.12),
p 4 w

L
100
4(14)
56000Pa
d
0.1

Ans.(a)

Thewallshearisdirectlyrelatedtof,andwemayiteratetofindVandQ:
f
8(14)

w V 2 , or: fV2
0.142 with 0.000015
8
789
d
0.142
0.015

1/2

Guess f 0.015, V

3.08

m
789(3.08)(0.1)
, Re
202000
s
0.0012

fbetter 0.0158, Vbetter 3.00m/s, Re better 197000(converged)


2

ThenV3.00m/s,andQ( /4)(0.1) (3.00)0.0236m /s85m /h.Ans.(b)


Finally,theloglawEq.(6.28)canestimatethevelocityatr1cm,yRr4cm:
1/2

1/2


m
14
u* w
0.133 ;

789

s
u 1 u*y
1
789(0.133)(0.04)
ln
B
ln

5.0 24.9
u*
0.41
0.0012
Then u 24.9(0.133) 3.3m/s at r 1cm. Ans.(c)

6.67Astraight10cmcommercialsteelpipeis1kmlongandislaidonaconstantslope
of5.Waterat20Cflowsdownward,duetogravityonly.Estimatetheflowratein
3
m /h.Whathappensifthepipelengthis2km?
3

Solution:Forwaterat20C,take998kg/m and0.001kg/ms.Iftheflowis
duetogravityonly,thentheheadlossexactlybalancestheelevationchange:
h f z L sin f

L V2
, or fV 2 2gd sin 2(9.81)(0.1)sin 5 0.171
d 2g

ThustheflowrateisindependentofthepipelengthLiflaidonaconstantslope.Ans.
Forcommercialsteel,take0.046mm,or/d0.00046.

224

Chapter 6Viscous Flow in Ducts

Beginbyguessingfullyroughflowforthefrictionfactor,anditerateVandReandf:
0.171
0.0164

f 0.0164, V

1/2

3.23

m
998(3.23)(0.1)
, Re
322000
s
0.001

fbetter 0.0179, Vbetter 3.09m/s, Re 308000(converged)


Then Q ( /4)(0.1)2 (3.09) 0.0243m 3 /s 87m 3 /h. Ans.
*6.68TheMoodychart,Fig.6.13,isbestforfindingheadloss(or p)whenQ,V,d,
andLareknown.Itisawkwardforthe2ndtypeofproblem,findingQwhenh forp
areknown(seeEx.6.9).PrepareamodifiedMoodychartwhoseabscissaisindependent
ofQandV,using/dasaparameter,fromwhichonecanimmediatelyreadtheordinate
tofind(dimensionless)QorV.UseyourcharttosolveExample6.9.
Solution:This problem was mentioned analytically in the text as Eq. (6.51). The
properparameterwhichcontainsheadlossonly,andnotflowrate,is:

gd 3 h f
L 2

/d 1.775

3.7

Red (8 )1/2 log

Eq.(6.51)

WesimplyplotReynoldsnumberversusforvarious/d,asshownbelow:

TosolveExample6.9,a100mlong,30cmdiameterpipewithaheadlossof8mand/d
3
0.0002,weusethatdatatocompute5.3E7.Theoilpropertiesare950kg/m and

225

Solutions ManualFluid Mechanics, Seventh Edition

2E5m /s.Enterthechartabove:letsfaceit,thescaleisveryhardtoread,butweestimate,
at5.3E7,that6E4Red9E4,whichtranslatestoaflowrateof
3
3
0.28Q0.42m /s.Ans.(Example6.9gaveQ0.342m /s.)

6.69ForProb.6.62supposetheonlypumpavailablecandeliveronly80hptothe
3
fluid.Whatistheproperpipesizeininchestomaintainthe3ft /sflowrate?
3

Solution:Forwaterat20C,take1.94slug/ft and2.09E5slug/fts.Forcast
iron,take0.00085ft.Wecantspecify/dbecausewedontknowd.Theenergyanalysis
aboveiscorrectandshouldbemodifiedtoreplaceVbyQ:
h p 120 f

L (4Q/ d2 )2
2000 [4(3.0)/ d 2 ]2
f
120 f
120 453 5
d
2g
d
2(32.2)
d

Butalso h p

Power 80(550)
453f

235 120 5 , or: d5 3.94f


gQ
62.4(3.0)
d

Guessf0.02,calculated,/dandReandgetabetterfanditerate:
f 0.020, d [3.94(0.02)]1/5 0.602ft, Re
or Re 589000,

4 Q
4(1.94)(3.0)

,
d (2.09E5)(0.602)

0.00085

0.00141, Moodychart: fbetter 0.0218(repeat)


d
0.602

Wearenearlyconverged.Thefinalsolutionisf0.0217,d0.612ft7.3inAns.

P6.70
Ethyleneglycolat20Cflowsthrough80metersofcastironpipeof
diameter6cm.Themeasuredpressuredropis250kPa.Neglectminorlosses.Usinga
noniterativeformulation,estimatetheflowrateinm3/h.
Solution:Forethyleneglycolat20C,TableA.3,=1117kg/m3and=0.0214kg/m
s.Theheadlossisgiven:p/g=250,000/[1117(9.81)]=22.8m.Forcastiron,=
0.26mm,hencetheroughnessratiois/d=0.26/60=0.00433.Wecanusethedirect
approachofEq.(6.51):

226

Chapter 6Viscous Flow in Ducts

g d3 hf
/ d 1.775
9.81(0.06)3 (22.8)
Re d 8 log(

) ,

1.65E6
3.7

L 2
80(0.0214 /1117) 2
0.00433
1.775
Evaluate : Re d 8(1.65E6) log(

) 9410 (yes, turbulent)


3.7
1.65E6
With the Reynolds number known, we can find the velocity and flow rate:

Red
(0.0214)(9410)
m

m3
m3
2
V

3.00 ; Q (0.06) (3.00) 0.0085


30.6
Ans.
d
1117(0.06)
s
4
2
h

6.71ItisdesiredtosolveProb.6.62forthemosteconomicalpumpandcastironpipe
system.Ifthepumpcosts$125perhorsepowerdeliveredtothefluidandthepipecosts
$7000perinchofdiameter,whataretheminimumcostandthepipeandpumpsizeto
3
maintainthe3ft /sflowrate?Makesomesimplifyingassumptions.
3

Solution:Forwaterat20C,take1.94slug/ft and2.09E5slug/fts.Forcast
iron,take0.00085ft.Writetheenergyequation(fromProb.6.62)intermsofQandd:
Pinhp

2 2
gQ
62.4(3.0)
154.2f
2000 [4(3.0)/ d ]

(z h f )
40.84
120 f

550
550
d
2(32.2)
d5

Cost $125Php $7000d inches 125(40.84 154.2f/d 5 ) 7000(12d), withdinft.


Cleanup: Cost $5105 19278f/d 5 84000d
Regardlessofthe(unknown)valueoff,thisCostrelationdoesshowaminimum.Ifweassume
forsimplicitythatfisconstant,wemayusethedifferentialcalculus:
d(Cost)
f const 5(19278)f
84000, or d best (1.148f)1/6
6
d(d )
d
Guess f 0.02, d [1.148(0.02)]1/6 0.533ft, Re

4Q

665000,
0.00159
d
d

Then fbetter 0.0224, d better 0.543ft(converged)


Result:dbest0.543ft6.5in,Costmin$14300pump$45600pipe$60,000.Ans.

227

Solutions ManualFluid Mechanics, Seventh Edition

6.72ModifyProb.P6.57bylettingthediameterbeunknown.Findtheproperpipe
diameterforwhichthepoolwilldraininabout2hoursflat.
Solution:Recallthedata:LetW5m,Y8m,ho2m,L15m,and0,with
3
water,998kg/m and0.001kg/ms.WeapplythesametheoryasProb.6.57:
V

2 gh
4WY
, t drain
1 fL/D
D2

2ho (1 fav L/D)


,
g

fav fcn(Re D ) forasmoothpipe.

Forthepresentproblem,tdrain2hoursandDistheunknown.Useanaveragevalueh
1mtofindfav.EntertheseequationsonEES(oryoucaniteratebyhand)andthefinal
resultsare
V 2.36m/s; Re D 217,000;

fav 0.0154; D 0.092m 9.2cm

Ans.

*6.73TheMoodychart,Fig.6.13,isbestforfindingheadloss(orp)whenQ,V,d,andL
areknown.Itisawkwardforthe3rdtypeofproblem,findingdwhenhf(orp)andQ
areknown(seeEx.6.11).PrepareamodifiedMoodychartwhoseabscissaisindependent
of d, using as a parameter nondimensionalized without d, from which one can
immediatelyreadthe(dimensionless)ordinatetofindd.UseyourcharttosolveEx.6.11.

Solution:AnappropriatePigroupwhichdoesnotcontaindis(ghfQ )/(L ).
Similarly,anappropriateroughnessparameterwithout d is (/Q).Afteralotof
algebra,theColebrookfrictionfactorformula(6.48)becomes
128
3

Red5/2 2.0

1/2

Red
2.51Red3/2

(128 / 3 )1/2
14.8

log10

Aplotofthismessyrelationisgivenbelow.

228

Chapter 6Viscous Flow in Ducts

TosolveExample6.11,a100mlong,unknowndiameterpipewithaheadlossof8m,
3
flowrateof0.342m /s,and0.06mm,weusethatdatatocompute 9.8E21and
3.5E6.Theoilpropertiesare 950kg/m3and 2E5m2/s.Enterthechart
above:letsfaceit,thescaleisveryhardtoread,butweestimate,at 9.8E21and
3.5E6,that6E4Red8E4,whichtranslatestoadiameterof0.27d0.36m.
Ans.(Example6.11gaved0.3m.)

P6.74 Tworeservoirs,whichdifferinsurfaceelevationby40m,areconnectedbyanew
commercialsteelpipeofdiameter8cm.Ifthedesiredweightflowrateis200N/sofwaterat
20C,whatistheproperlengthofthepipe?Neglectminorlosses.

Solution:Forwaterat20C,take=998kg/m3and=0.001kg/ms.Forcommercialsteel,
=0.046mm,thus/d=0.046mm/80mm=0.000575.Findthevelocityandthefrictionfactor:
V

/( g )
w
( / 4) D

1
f

200 /[998(9.81)]
( / 4)(0.08)

2.0 log10 (

4.06

m
,
s

Re D

/D
2.51

)
3. 7
Re D f

VD
998(4.06)(0.08)

324,000

0.001

yields

f 0.0185

229

Solutions ManualFluid Mechanics, Seventh Edition

Thenwefindthepipelengthfromtheenergyequation,whichissimpleinthiscase:

z 40 m

L V2
L
(4.06) 2
(0.0185)
, Solve
D 2g
(0.08m) 2(9.81)

L 205 m

Ans.

230

Chapter 6Viscous Flow in Ducts

6.75Youwishtowateryourgardenwith
100ftof 85 indiameterhosewhoserough
nessis0.011in.Whatwillbethedelivery,
3
inft /s,ifthegagepressureatthefaucetis
2
60 lbf/in ? If there is no nozzle (just an
open hose exit), what is the maximum
horizontaldistancetheexitjetwillcarry?

Fig.P6.75

Solution:Forwater,take1.94slug/ft and2.09E5slug/fts.Wearegiven/d
0.011/(5/8)0.0176.Forconstantareahose,V1V2andenergyyields
pfaucet
60 144psf
L V2
100
V2
h f , or:
138ft f
f
,
g
1.94(32.2)
d 2g
(5/8)/12 2(32.2)
or fV 2 4.64. Guess f ffullyrough 0.0463, V 10.0

ft
, Re 48400
s

then fbetter 0.0472, Vfinal 9.91ft/s (converged)


3

ThehosedeliverythenisQ(/4)(5/8/12) (9.91)0.0211ft /s.Ans.(a)


Fromelementaryparticletrajectorytheory,themaximumhorizontaldistanceXtraveled
2
2
bythejetoccursat45(seefigure)andisXV /g(9.91) /(32.2)3.05ftAns.(b),
whichispitiful.Youneedanozzleonthehosetoincreasetheexitvelocity.

6.76ThesmallturbineinFig.P6.76extracts400Wofpowerfromthewaterflow.Both
3

pipesarewroughtiron.Compute theflowrate Q m /h.Whyaretheretwosolutions?


Whichisbetter?

231

Solutions ManualFluid Mechanics, Seventh Edition

Solution:Forwater,take998kg/m and0.001kg/ms.Forwroughtiron,take

0.046mm,hence /d1 0.046/60 0.000767and /d2 0.046/40 0.00115.The


energyequation,withV10andp1p2,gives
z1 z2 20m

V22
L V2
h f2 h f1 h turbine , h f1 f1 1 1
2g
d1 2g

and h f2 f2

L 2 V22
d 2 2g

P
400W

and Q d12 V1 d 22 V2
gQ 998(9.81)Q
4
4
If we rewrite the energy equation in terms of Q and multiply by Q, it is essentially a cubic
polynomial, because for these rough walls the friction factors are almost constant:
8f L Q3 8f L Q3
400
8Q3
Q hturbine
20Q 12 1 5 22 2 5 2 4
998(9.81)
gd1
gd 2
gd 2
Also, h turbine

Solve by EES or by iteration. There are three solutions, two of which are positive and the
third is a meaningless negative value. The two valid (positive) solutions are:
(a ) Q 0.00437 m3 / s 15.7m3 / hr ; Re1 92,500, f1 0.0215 ; Re1 138,800, f1 0.0221
(b) Q 0.00250 m3 / s 9.0 m3 / hr ; Re1 52,900, f1 0.0232 ; Re1 79, 400, f1 0.0232 Ans.
[Thenegative(meaningless)solutionisQ=0.0069m3/hr.]Bothsolutions(a)and(b)
arevalidmathematically.Solution(b)ispreferredthesamepowerfor43%lesswater
flow,andtheturbinecaptures16.3moftheavailable20mhead.Solution(a)isalso
unrealistic,becausearealturbinespowerincreaseswithwaterflowrate.Turbine(a)
wouldgeneratemorethan400W.

232

Chapter 6Viscous Flow in Ducts

6.77ModifyProb.6.76intoaneconomicanalysis,asfollows.Letthe40mofwrought
3
ironpipehaveauniformdiameterd.LetthesteadywaterflowavailablebeQ30m h.
Thecostoftheturbineis$4perwattdeveloped,andthecostofthepipingis$75per
centimeterofdiameter.Thepowergeneratedmaybesoldfor$0.08perkilowatthour.
Findtheproperpipediameterforminimumpaybacktime,i.e.,minimumtimeforwhich
thepowersaleswillequaltheinitialcostofthesystem.
Solution:Withflowrateknown,weneedonlyguessadiameterandcomputepower
fromtheenergyequationsimilartoProb.6.76:
P gQh t , where h t 20m

V2
1 f
2g

L
8Q 2
L

20

d
d
2 gd 4

P
(24)(365)
1000

Then Cost $4 * P $75(100d) and Annualincome $0.08

TheMoodyfrictionfactoriscomputedfromRe 4 Q(d)and d 0.066d(mm).


Thepaybacktime,inyears,isthenthecostdividedbytheannualincome.Forexample,
Ifd0.1m,Re106000,f0.0200,ht19.48m,P1589.3W
Cost$7107Income$1,114yearPayback6.38years
Since the piping cost is very small ($1000), both cost and income are nearly
proportionaltopower,hencethepaybackwillbenearlythesame(6.38years)regardless
ofdiameter.Thereisanalmostinvisibleminimumatd7cm,Re151000,f0.0201,
ht17.0m,Cost$6078,Income$973,Payback6.25years.However,asdiameterd
decreases,wegeneratelesspowerandgainlittleinpaybacktime.

233

Solutions ManualFluid Mechanics, Seventh Edition

6.78InFig.P6.78theconnectingpipeis
commercialsteel6cmindiameter.Estimate
3
theflowrate,inm h,ifthefluidiswater
at20C.Whichwayistheflow?

Fig.P6.78

Solution:Forwater,take998kgm
and 0.001 kgms. For commercial
steel, take 0.046 mm, hence d
0.046600.000767.Withp1,V1,andV2
all 0, the energy equation between
surfaces(1)and(2)yields
0 0 z1

p2
200000
0 z 2 h f , or h f 15
5.43m (flowto left)
g
998(9.81)

Guessturbulentflow: h f f

L V2
50 V 2
f
5.43, or: fV 2 0.1278
d 2g
0.06 2(9.81)
1/2

m
0.1278
0.00767, guess ffullyrough 0.0184, V
2.64 , Re 158000

0.0184
d
s
m
fbetter 0.0204, Vbetter 2.50 , Re better 149700, f3rditeration 0.0205(converged)
s
Theiterationconvergesto
2

f0.0205,V2.49ms,Q(4)(0.06) (2.49)0.00705m s25m hAns.

6.79Agardenhoseisusedasthereturnlineinawaterfalldisplayatthemall.Inorder
toselect theproperpump,youneedtoknow thehosewallroughness,whichis not
suppliedbythemanufacturer.Youdeviseasimpleexperiment:attachthehosetothe
drainofanabovegroundpoolwhosesurfaceis3mabovethehoseoutlet.Youestimate
theminorlosscoefficientintheentranceregionas0.5,andthedrainvalvehasaminor
lossequivalentlengthof200diameterswhenfullyopen.Usingabucketandstopwatch,
3
youopenthevalveandmeasureaflowrateof2.0E4m /sforahoseofinsidediameter
1.5cmandlength10m.Estimatetheroughnessheightofthehoseinsidesurface.
Solution:FirstevaluatetheaveragevelocityinthehoseanditsReynoldsnumber:
V

Q
2.0 E4
m
Vd 998(1.13)(0.015)

1.13 , Red

16940(turbulent )
A ( /4)(0.015)2
s

0.001

234

Chapter 6Viscous Flow in Ducts

Writetheenergyequationfromsurface(point1)tooutlet(point2),assuminganenergy
correctionfactor1.05:
2
Leq V 2

p1 1V1
p
V2

z1 2 2 2 z2 h f hloss , where hloss K e f 2 2


g
2g
g
2g
d
2g

Theunknownisthefrictionfactor:
z1 z2
f

2 Ke

V /2 g
( L Leq )/d

3m
1.05 0.5
(1.13)2 /2(9.81)

0.0514
(10/0.015 200)

Forf0.0514andRed16940,theMoodychart(Eq.6.48)predicts/d0.0206.
Thereforetheestimatedhosewallroughnessis0.0206(1.5cm)0.031cmAns.

6.80Theheadversusflowratecharacteri
stics of acentrifugal pumpare shownin
Fig. P6.80. If this pump drives water at
20C through 120 m of 30cmdiameter
castiron pipe, what will be the resulting
3
flowrate,inm /s?

Fig.P6.80

Solution:Forwater,take998kg/m
and0.001kg/ms.Forcastiron,take
0.26 mm, hence /d 0.26/300
0.000867. The head loss must match the
pumphead:
h f
f

L V 2 8fLQ 2

h pump 80 20Q 2 , withQinm 3 /s


d 2g 2 gd 5

Evaluate h f

8f(120)Q 2

2 (9.81)(0.3)5

80 20Q 2 , or: Q 2

80

Guess f 0.02, Q

20 4080(0.02)

1/2

80
20 4080f

m3
4 Q
0.887
, Re
3.76E6
s
d

m3
0.000867, f
0.0191, Re
3.83E6, convergesto Q 0.905
better
better
d
s

Ans.

235

Solutions ManualFluid Mechanics, Seventh Edition

6.81ThepumpinFig.P6.80isusedtodelivergasolineat20Cthrough350mof
3
30cmdiametergalvanizedironpipe.Estimatetheresultingflowrate,inm /s.(Note
thatthepumpheadisnowinmetersofgasoline.)
3

Solution:Forgasoline,take 680kg/m and 2.92E4kg/ms.Forgalvanized


iron,take0.15mm,hence/d0.15/3000.0005.Headlossmatchespumphead:
hf

8fLQ 2
8f (350)Q 2
80

11901fQ 2 h pump 80 20Q 2 , Q 2


2
5
2
5
20 11901f
gd
(9.81)(0.3)
Guess frough 0.017, Q 0.600
Re better 5.93E6,

m3
,
s

0.0005, fbetter 0.0168


d
3

Thisconvergestof0.0168,Re5.96E6,Q0.603m /s.Ans.

P6.82
Fluid at 20C flows through a horizontal galvanized-iron pipe 20 m long and 8 cm
in diameter. The wall shear stress is 90 Pa. Calculate the flow rate in m3/h if the fluid is (a)
glycerin; and (b) water.
3

Solution: (a) For glycerin, take 1260 kg/m and 1.49 kg/ms. For galvanized iron, take
0.15 mm, hence /D 0.15/80 0.001875. But we are guessing this flow is laminar:

w,laminar 90 Pa

8 V
8(1.49) V

,
D
(0.08)

solve V 0.604

m
s

m3
m3
D V (0.08) 2 (0.604) 0.00304
10.9
4
4
s
h
VD (1260)(0.604)(0.08)
Check Re D

41 (yes, laminar)

1.49
Q

Ans.(a )

(b) For water, take 998 kg/m and 0.001 kg/ms. For galvanized iron, take 0.15
mm, hence /D 0.15/80 0.001875. Now we are guessing this flow is turbulent. At
that roughness, the minimum friction factor is 0.023, which we can use for a first estimate:
V 2
998V 2
m
w 90 Pa f
(0.023)
, or : V 5.6 , Re D 447, 000
8
8
s

236

Chapter 6Viscous Flow in Ducts

TheReynoldsnumberestimateiscertainlyhighenough,andintothefullyroughregion
oftheMoodychart.Iteratebrieflytothefinalresult,onlyslightlydifferent:

f 0.0235 ; Re D 443, 000 ; V 5.55

m
m3
m3
; Q 0.0279
100
Ans.(b)
s
s
h

6.83ForthesystemofFig.P6.55,letz80mandL185mofcastironpipe.What
3
isthepipediameterforwhichtheflowratewillbe7m /h?
3

Solution:Forwater,take998kg/m and0.001kg/ms.Forcastiron,take
0.26mm,butdisunknown.Theenergyequationissimply

Fig.P6.55

8fLQ 2 8f(185)(7/3600)2 5.78E5f

, or d 0.0591f 1/5
2 gd 5
2 (9.81)d 5
d5
4 Q

Guess f 0.03, d 0.0591(0.03)1/5 0.0293m, Re


84300,
0.00887
d
d
z 80m h f

Iterate:fbetter0.0372,dbetter0.0306m,Rebetter80700,/dbetter0.00850,etc.The
processconvergestof0.0367,d0.0305m.Ans.
3

6.84Itisdesiredtodeliver60m /hofwater(998kg/m ,0.001kg/ms)at20C


through a horizontal asphalted castiron pipe. Estimate the pipe diameter which will
causethepressuredroptobeexactly40kPaper100metersofpipelength.

237

Solutions ManualFluid Mechanics, Seventh Edition

Solution:Writeouttherelationbetweenpandfrictionfactor,takingL100m:
L 2
100 (998) 60/3600
p f
V f

d 2
d
2 ( /4)d 2

40,000 22.48

f
, or: d 5 0.00562 f
5
d

Knowing0.12mm,then/d0.00012/dandRed4Q/(d)21178/d.UseEES,
orguessf0.02anditerateuntiltheproperdiameterandfrictionfactorarefound.
Finalconvergence:f0.0216;Red204,000;d0.104m.Ans.

P6.85 ForthesysteminProb.P6.53,apump,whichdelivers15,000Wtothewater,is
usedatnighttorefilltheupperreservoir.Thepipediameterisincreasedfrom12cmto
providemoreflow.Iftheresultantflowrateis90m3/h,estimatethenewpipesize.
Solution: For water at 20C, Table A.3, = 998 kg/m3 and = 0.001 kg/m-s. Recall that
z = 60 m and L = 360 m. Since the pressures and velocities cancel, the energy equation
becomes
360m V 2
zlower zupper h f h pump , or : h p z h f 60m f
D 2(9.81)
where

1
f

2.0 log10 (

Re D f
VD
4 Q 4(998)(90 / 3600)
) , Re D

,
2.51

D
(0.001) D

2
m3
m3
D 90
0.025
, Power g Q h p (998)(9.81) Q h p 15, 000 W
4
h
s
Youcouldsolvebyiteration,guessingvaluesofDgreaterthan12cm,untilQ=90m3/h.
OryoucouldputtheaboveequationsintoEES,whichwillreporttheanswer:
Q V

Re D 169, 000 ; f 0.0161 ; Q 90 m3 / h

if

D 0.188 m

Ans.

Thusa57%increaseindiameteronlyproducesa13%increaseinflowrate.Evenwith
anindefinitelylargediameter,becauseofthe60meterelevationheadtofightagainst,Q
canneverbegreaterthan92m3/hifP=15kW.

238

Chapter 6Viscous Flow in Ducts

6.86SAE10oilat20Cflowsatanaveragevelocityof2m/sbetweentwosmooth
parallelhorizontalplates3cmapart.Estimate(a)thecenterlinevelocity,(b)theheadloss
permeter,and(c)thepressuredroppermeter.
3

Solution:ForSAE10oil,take870kg/m and0.104kg/ms.Thehalfdistance
betweenplatesiscalledh(seeFig.6.37).CheckDhandRe:
Dh

4A
VD h 870(2.0)(0.06)
4h 6cm, Re Dh

1004(laminar)
P

0.104
Then uCL umax

3
3
V (2.0) 3.0 m/s
2
2

Ans.(a)

Theheadlossandpressuredroppermeterfollowfromlaminartheory,Eq.(6.63):
p

3 VL 3(0.104)(2.0)(1.0)

2770Pa/m
h2
(0.015m)2
hf

p
2770

0.325m/m
g 870(9.81)

Ans.(c)

Ans.(b)

6.87Acommercialsteelannulus40ftlong,witha1inandb 12 in,connectstwo
3
reservoirswhichdifferinsurfaceheightby20ft.Computetheflowrateinft /sthrough
theannulusifthefluidiswaterat20C.
3

Solution:Forwater,take1.94slug/ft and2.09E5slug/fts.Forcommercial
steel,take0.00015ft.Computethehydraulicdiameteroftheannulus:
4A
Dh
2(a b) 1inch;
P
40
L V2
V2
h f 20ft f
f
, or: fV 2 2.683
1/12
2(32.2)
D h 2g
WecanmakeareasonableestimatebysimplyrelatingtheMoodycharttoDh,ratherthan
themorecomplicatedeffectivediametermethodofEq.(6.77).Thus

0.00015
ft

0.0018, Guess frough 0.023, V (2.683/0.023)1/2 10.8


Dh
1/12
s
Re

VDh 1.94(10.8)(1/12)
ft

83550, fbetter 0.0249, Vbetter 10.4

2.09E5
s

239

Solutions ManualFluid Mechanics, Seventh Edition

Thisconvergestof0.0250,V10.37ft/s,Q(a b )V0.17ft /s.Ans.

6.88Anoilcoolerconsistsofmultipleparallelplatepassages,asshowninFig.P6.88.
Theavailablepressuredropis6kPa,andthefluidisSAE10Woilat20C.Ifthedesired
3
totalflowrateis900m /h,estimatetheappropriatenumberofpassages.Theplatewalls
arehydraulicallysmooth.

Fig.P6.88
3

Solution:ForSAE10Woil,870kg/m and0.104kg/ms.Thepressuredrop
remains6kPanomatterhowmanypassagesthereare(ductsinparallel).Guesslaminar
flow,Eq.(6.63),
Qonepassage

bh3 p

3 L

wherehisthehalfthicknessbetweenplates.IfthereareNpassages,thenb50cmfor
3
alland h 0.5m/(2N).Wefind h and N suchthat NQ 900m /hforthefullsetof
passages.TheproblemisidealforEES,butonecaniteratewithacalculatoralso.We
3
findthat18passagesareonetoomanyQonlyequals835m /h.Thebettersolutionis:
N 17passages, QN 935m 3 /h, h 1.47cm, Re Dh 512(laminarflow)

6.89Anannulusofnarrowclearancecausesaverylargepressuredropandisusefulas
anaccuratemeasurementofviscosity.Ifasmoothannulus1mlongwitha50mmand
3
b49mmcarriesanoilflowat0.001m /s,whatistheoilviscosityifthepressuredrop
is250kPa?

Chapter 6Viscous Flow in Ducts

240

Solution:Assuminglaminarflow,useEq.(6.73)forthepressuredropandflowrate:

p 4 4 (a 2 b2 )2
Q
a b
, or,forthegivendata:
8 L
ln(a/b)
2
2 2
250000
4
4 {(0.05) (0.049) }
0.001m /s

(0.05) (0.049)
8 1m
ln(0.05/0.049)
3

Solvefor 0.0065kg/ms

Ans.

P6.90
A rectangular sheet-metal duct is 200 ft long and has a fixed height H = 6
inches. The width B, however, may vary from 6 to 36 inches. A blower provides a pressure
drop of 80 Pa of air at 20C and 1 atm. What is the optimum width B that will provide the
most airflow in ft3/s?
Solution:Forairat20Cand1atm,take=1.20kg/m3and=1.8E5kg/ms.The
pressuredropisrelatedtothehydraulicdiameteroftheduct.ConvertL=200ft=60.96
m.Forsheetmetal,fromTable6.1,theroughness=0.05mm.

p f

L 2
V ,
Dh 2

Solve for V

where Dh

2 p Dh

f L

2BH
4A

, f related to Re Dh and
P
BH
Dh

2 (80 Pa ) Dh
Dh
m
1.479
which gives V in
f (1.20)(60.96)
f
s

Theductarea A=2BH increaseswith B forafixed H,andsodoesthehydraulic


diameter. The Reynolds number (VDh/) also increases, hence the friction factor f
decreases.AllofthesefactorsmaketheflowrateQincreasewithH.Therefore,without
evenmakingcalculations,weconcludethat thewidest H (36inches)produces

themostflowrate.Ans.

WecancalculatetheactualflowrateforH=36inches=0.9144m:

241

Solutions ManualFluid Mechanics, Seventh Edition

H 36 in , Dh 0.261 m , Re Dh 95, 200 ,

0.000191 , f 0.01914,
Dh

m
m3
ft 3
, Q V B H 0.761
27
Ans.
s
s
s
HereisaplotofflowrateQversuswidthH.Itisalmostexactlylinear.
Giving V 5.46

30

Q,m3/s

25
20
15
10

Ductwidth

B,inches

0
0

10

15

20

25

30

35

40

6.91Heatexchangersoftenconsistofmanytriangularpassages.TypicalisFig.P6.91,
with L 60 cm and an isoscelestriangle cross section of side length a
2cmandincludedangle80.IftheaveragevelocityisV2m/sandthefluidisSAE
10oilat20C,estimatethepressuredrop.

Fig.P6.91

Chapter 6Viscous Flow in Ducts

242

Solution:ForSAE10oil,take 870kg/m and 0.104kg/ms.TheReynolds


numberbasedonsidelengthaisRe Va/335,sotheflowislaminar.Thebottom
sideofthetriangleis2(2cm)sin402.57cm.Calculatehydraulicdiameter:
A
Re Dh

1
4A
(2.57)(2 cos 40) 1.97cm 2 ; P 6.57cm; D h
1.20cm
2
P

VDh 870(2.0)(0.0120)

201; fromTable6.4, 40, fRe 52.9

0.104

Then f

52.9
L 2
0.6 870

2
0.263, p f
V (0.263)

(2)

201
Dh 2
0.012
2
23000Pa

Ans.

6.92Alargeroomusesafantodrawinatmosphericairat20 Cthrougha30cmby30cm
3
commercialsteelduct12mlong,asinFig.P6.92.Estimate(a)theairflowrateinm /hr
iftheroompressureis10Pavacuum;and(b)theroompressureiftheflowrateis1200
3
m /hr.Neglectminorlosses.

Fig.P6.92

Solution:Forair,take1.2kg/m and1.8E5kg/ms.Forcommercialsteel,
0.046 mm. For a square duct, Dh sidelength 30 cm, hence /d 0.046/300
0.000153.The(b)partiseasier,withflowrateknownwecanevaluatevelocity,Reynolds
number,andfrictionfactor:
V

Q 1200/3600
m
1.2(3.70)(0.3)

3.70 , ReDh
74100, thus f Moody 0.0198
A (0.3)(0.3)
s
1.8 E5

Thenthepressuredropfollowsimmediately:

243

Solutions ManualFluid Mechanics, Seventh Edition

p f

L 2
12 1.2

V 0.0198
(3.70)2 6.53Pa,

0.3 2
Dh 2
or: proom 6.5Pa (vacuum) Ans.(b)

(a)Ifp10Pa(vacuum)isknown,wemustiteratetofindfrictionfactor:
Q
12 1.2

V 2, V
,
0.3
2
(0.3)2

p 10 Pa f

1.2V (0.3)

f fcn
,
0.000153
1.8 E5 Dh

Afteriteration,theresultsconvergeto:
3

V4.69m/s;Red93800;f0.0190;Q0.422m /s1520m /hAns.(a)

P6.93InMoodysExample6.6,the6inchdiameter,200ftlongasphaltedcastironpipehasa
pressuredropofabout280lbf/ft2whentheaveragewatervelocityis6ft/s.Comparethistoan
annularcastironpipewithaninnerdiameterof6inandthesameannularaveragevelocityof6
ft/s.(a)Whatouterdiameterwouldcausetheflowtohavethesamepressuredropof280lbf/ft 2?
(b) How do the crosssection areas compare, and why? Use the hydraulic diameter
approximation.

Solution:RecalltheEx.6.6data,=0.0004ft.Forwaterat68F,take=1.94slug/ft3and
=2.09E5slug/ftsec.ThehydraulicdiameterofanannulusisDh=2(RoRi),whereRi=0.25
ft.Weknowthepressuredrop,hencetheheadlossis

hf

(6 ft / s ) 2
280 lbf / ft 2
L V2
200 ft
p
f

4.49 ft
Dh 2 g
2( Ro 0.25 ft ) 32.2 ft / s 2
g
62.4 lbf / ft 3

WedonotknowforRo.TheadditionalrelationistheMoodyfrictionfactorcorrelation:

1
f

2.0 log10 (

/ Dh
2.51

)
3.7
Re Dh f

where

Re Dh

VDh (1.94)(6.0)[2( Ro 0.25)]

2.09 E 5

244

Chapter 6Viscous Flow in Ducts

(a)For=0.0004ft,solvethesetwosimultaneously,usingEESorExcel,toobtain
f 0.0199 ; Re Dho 276,000 ; Ro 0.498 ft

Ans.( a )

(b)Theannulargapis0.4980.25=0.248ft,justaboutequaltotheinnerradius.However,
theannularareaisthreetimestheareaofMoodyspipe!Ans.(b)Theannularpipehasmuch
morewallareathanahollowpipe,morefriction,somorecrosssectionareaisneededtomatch
thepressuredrop.

P6.94
Air at 20C flows through a smooth duct of diameter 20 cm at an average
velocity of 5 m/s. It then flows into a smooth square duct of side length a. Find the
square duct size a for which the pressure drop per meter will be exactly the same as the
circular duct?
Solution: For air at 20C and 1 atm, take = 1.20 kg/m3 and = 1.8E-5 kg/m-s.
Compute the pressure drop in the circular duct:
Re D

VD
(1.2)(5)(0.2)

66, 700 ;

1.8E 5

p f

f smooth 0.0196

1 m 1.2kg / m3
L 2
m
Pa
V (0.0196)(
)(
)(5 ) 2 1.47
D 2
0.2 m
2
s
m

Thesquareductwillhaveslightlydifferentsize,Reynoldsnumber,andvelocity:

Vsquare a
(1.2)Vs a
4a 2
Dh
a ; Re Dh

4a

1.8 E 5

m3
D V (0.2) 2 (5) 0.157
Vs a 2
4
4
s
Thuseverythingcanbewrittenintermsofthesquareductsizea:
But Q

245

Solutions ManualFluid Mechanics, Seventh Edition

Re Dh

1.2(0.157 / a 2 ) a 10470
1 m 1.2 0.157 2
Pa
L 2

; p 1.47
f
Vs f
( )( 2 )
1.8 E 5
a
m
Dh 2
a 2
a

or : 1.47

0.0147 f
a

or : f 99.5 a5

Guessfequalto,say,0.02,findtheimprovedReynoldsnumberandf,finallyfinda:
Vs=4.70m/s;ReDh=57,350;f=0.0203;a=0.183mAns.
_______________________________________________________________________
P6.95
Although analytical solutions are available for laminar flow in many duct
shapes [34], what do we do about ducts of arbitrary shape? Bahrami et al. [57] propose
that a better approach to the pipe result, f Re = 64, is achieved by replacing the hydraulic
diameter Dh by A, where A is the area of the cross section. Test this idea for the isosceles
triangles of Table 6.4. If time is short, at least try 10, 50, and 80. What do you
conclude about this idea?
Solution:WecanseeforthetrianglesinTable6.4thatthevaluesoffReDharealllessthan
64,byasmuchas25%. IfwedenoteBahramisideaas DB = A,thenewReynolds
numberisbasedon

DB

DB
A
Dh
Dh
Dh
4A / P

P
4 A

) Dh

For an isosceles triangle of height h , P 2h sec 2h tan , A h 2 tan


sec tan
Thus DB (
) Dh
2 tan
SowemultiplythevaluesoffReDhbytheabovefactorandseehowwecloseitisto64:
, degrees
f ReB

10
73

20
63

30
61

40
62

50
65

60
72

70
85

80
116

246

Chapter 6Viscous Flow in Ducts

We see that intermediate angles, 20 to 50, work well, sharper angles not so good.
Bahrami et al. [57] make some suggestions to modify the idea for too-small or too-large
angles.

P6.96
A fuel cell [Ref. 59] consists of air (or oxygen) and hydrogen micro ducts,
separated by a membrane that promotes proton exchange for an electric current, as in Fig.
P6.96. Suppose that the air side, at 20C and approximately 1 atm, has five 1 mm by 1
mm ducts, each 1 m long. The total flow rate is 1.5E-4 kg/s. (a) Determine if the flow is
laminar or turbulent. (b) Estimate the pressure drop.
air flow
hydrogen flow
anode
membrane

1mmby1mmby1m

Fig. P6.96. Simplified diagram of an air-hydrogen fuel cell.


[Problem courtesy of Dr. Pezhman Shirvanian]
Solution: For air at 20C and 1 atm, take = 1.20 kg/m3 and = 1.8E-5 kg/m-s. The
hydraulic diameter of a square duct is easy, the side length a = 1 mm. The mass flow
through one duct is
1.5E 4 kg / s
kg
kg
0.3E 4
AV (1.20 3 )[(0.001m) 2 ]V
5
s
m
VDh 1.20(25.0)(0.001)
m
Solve for V 25.0 , hence Re Dh

1667 (laminar) Ans.( a)


s

0.000018
m&1duct

(b) We could go with the simply circular-duct approximation, f = 64/Re, but we have a
more exact laminar-flow result in Table 6.4 for a square duct:

247

f square

Solutions ManualFluid Mechanics, Seventh Edition

56.91
56.91

0.0341
Re Dh
1667

Then p f

1m
L 2
1.2kg / m3
m
V (0.0341)(
)(
)(25 ) 2 12, 800 Pa
D 2
0.001m
2
s

Ans.(b)

6.97Aheatexchangerconsistsofmultipleparallelplatepassages,asshowninFig.P6.97.
Theavailablepressuredropis2kPa,andthefluidiswaterat20C.Ifthedesiredtotal
3
flowrateis900m /h,estimatetheappropriatenumberofpassages.Theplatewallsare
hydraulicallysmooth.

Fig.P6.97
3

Solution:Forwater,998kg/m and0.001kg/ms.UnlikeProb.6.88,herewe
expectturbulentflow.IfthereareNpassages,thenb50cmforallNandthepassage
thicknessisH0.5m/N.ThehydraulicdiameterisDh2H.Thevelocityineachpassage
isrelatedtothepressuredropbyEq.(6.58):
p f

L 2
V
Dh 2

VDh

where f fsmooth fcn

Forthegivendata, 2000Pa f

2.0m 998kg/m 3 2
V
2(0.5m/N )
2

Select N,find H and V and Qtotal AV b V andcomparetothedesiredflowof900


3
3
m /h.Forexample, guess N 20,calculate f 0.0173and Qtotal 2165m /h.The
convergedresultis
Qtotal 908m 3 /h,

f 0.028,

Re Dh 14400, H 7.14mm, N 70passages

Ans.

248

Chapter 6Viscous Flow in Ducts

6.98A rectangular heat exchanger is to


be divided into smaller sections using
sheetsofcommercialsteel0.4mmthick,as
sketched in Fig. P6.98. The flow rate is
20kg/sofwaterat20C.Basicdimensions
are L 1m, W 20cm,and H 10cm.
What is the proper number of square
sectionsiftheoverallpressuredropistobe
nomorethan1600Pa?

Fig.P6.98

Solution:Forwaterat20C,take998kg/m and0.001kg/ms.Forcommercial
steel,0.046mm.Lettheshortside(10cm)bedividedintoJsquares.Thenthelong
2
(20cm)sidedividesinto2JsquaresandaltogetherthereareN2J squares.Denote
thesidelengthofthesquareasa,whichequals(10cm)/Jminusthewallthickness.The
hydraulicdiameterofasquareexactlyequalsitssidelength,D h a.Thetotalcross
sectionareaisA=Na2.Thenthepressuredroprelationbecomes
L 2
1.0 998
Q

p f
V f

Dh 2
a
2
Na

1600 Pa, where N 2J 2 and a

0.1
0.0004
J

Asafirstestimate,neglectthe0.4mmwallthickness,soa0.1/J.Thentherelationfor
pabovereducestofJ0.32.Sincef0.036forthisturbulentReynoldsnumber(Re
1E4)weestimatethatJ9andinfactthisisnotbadevenincludingwallthickness:
J 9, N 2(9)2 162, a
Re

0.1
20/998
m
0.0004 0.0107m, V

1.078
9
s
162(0.0107)2

Va 998(1.078)(0.0107)
0.046

11526,

0.00429, fMoody 0.0360

0.001
a 10.7
1.0 998

Then p (0.036)
(1.078)2 1950Pa
0.0107
2

SothewallthicknessincreasesVanddecreasesasopistoolarge.TryJ8:
m
J 8, N 128, a 0.0121m, V 1.069 ,
s

Re 12913,
0.0038, f 0.0347
a
Then p f(L/a)(/2)V 2 1636 Pa. Closeenough, J 8,N 128

Ans.

[IsupposeapracticalpersonwouldspecifyJ7,N98,tokeepp1600Pa.]

249

Solutions ManualFluid Mechanics, Seventh Edition

P6.99InSec.6.11itwasmentioned
thatRomanaqueductcustomersobtained
D2=5cm

extrawaterbyattachingadiffusertotheir
2m

D1=3cm,L=2m

pipeexits.Fig.P6.99showsasimulation:

V2

V1

asmoothinletpipe,withandwithouta15

15diffuser

diffuserexpandingtoa5cmdiameterexit.

Fig.P6.99

Thepipeentranceissharpedged.
Calculatetheflowrate(a)without,and(b)withthediffuser.

Solution:Forwaterat20C,take=998kg/m3and=0.001kg/ms.Theenergyequation
betweentheaqueductsurfaceandthepipeexityields

z surf

z2

V22
V2
V2
L
h f hm z 2 2 1 ( f
K entrance K diffuser )
2g
2g
2g
D1

(a)Withoutthediffuser,Kdiff=0,andV1=V2.Forasharpedge,takeKent=0.5.Weobtain

2m

V12
2m
(1 f
0.5) ,
2g
0.03m

with

f fcn(Re V1 D1 / )

Solve : Re 115,000 ; f 0.0175 ; V1 4.48 m / s ; Qwithout 0.00271 m 3 / s

Ans.( a )

Chapter 6Viscous Flow in Ducts

250

(b)Withthediffuser,fromFig.6.23,forD1/D2=3/5=0.6and2=15,readKdiffuser0.2.From
onedimensionalcontinuity,V2=V1(3/5)2=0.36V1.Theenergyequationbecomes

2m

(0.36V1 ) 2
V2
2m
1 (1 f
0.5 0.2)
2g
2g
0.03m

Solve : Re 134,000 ; f 0.0169 ; V1 3.84 m / s ; Qwith 0.00316 m 3 / s

Ans.(b)

Addingthediffuserincreasestheflowrateby17%.[NOTE:DontknowiftheRomansdid
this,butawellroundedentrance,Kent=0.05,wouldincreasetheflowratebyanother15%.]

P6.100
Water at 68F flows at 100 gal/min through a 2-inch-diameter commercial
steel pipe that is 150 ft long. According to Table 6.5, how many 90 regular screwed
elbows can be added to the system before the overall pressure drop exceeds 25 lbf/in2?
Solution: For water at 68F (20C), = 1.94 slug/ft3 and = 0.0000209 slug/ft-s. For
commercial steel, = 0.00015 ft, hence /D = 0.00015/(2/12) = 0.0009. Compute the
velocity, Reynolds number, and friction factor for a steady flow rate of 100 gal/min.
0.223 ft 3 s
gal
ft 3
Q
ft
0.223
;V

10.2
2
2
min
s
s
( / 4) D
( / 4)(2 /12 ft )
VD (1.94)(10.2)(2 /12)

158, 000 ;
0.0009 ; f Moody 0.0210

(0.0000209)
D

Q 100
Re D

Compute the pressure drop of the pipe alone:


p f

L 2
150 1.94
lbf
lbf
V (0.021)(
)(
)(10.2)2 1910 2 13.2 2
D 2
2 /12 2
ft
in

Thus we can add elbows until their total minor loss = 25.0 13.2 = 11.8 lbf/in2 = 1700
lbf/ft2. From Table 6.5, each elbow has a loss K = 0.95.
The pressure loss of each elbow is

251

Solutions ManualFluid Mechanics, Seventh Edition

p elbow K elbow

2
1.94
lbf
V (0.95) (
)(10.2) 2 96 2
2
2
ft

Thus the no. of elbows

total elbow loss 1700 lbf / ft 2

17 elbows max Ans.


loss per elbow
96 lbf / ft 2

NOTE:INPROBLEMS6.1006.110,MINORLOSSESAREINCLUDED.
6.101InFig.P6.101athickfilterisbeingtestedforlosses.Theflowrateinthepipeis
3
7m /min,andtheupstreampressureis120kPa.Thefluidisairat20C.Usingthewater
manometerreading,estimatethelosscoefficientKofthefilter.

Fig.P6.101
3

Solution:Theupstreamdensityisairp/(RT)120000/[287(293)]1.43kg/m .The
averagevelocityV(whichisusedtocorrelatelosscoefficient)followsfromtheflowrate:
Q
7/60m3 /s
V

14.85m/s
Apipe ( /4)(0.1m)2
Themanometermeasuresthepressuredropacrossthefilter:
pmano ( w a )ghmano (998 1.43kg/m 3 )(9.81m/s 2 )(0.04m ) 391Pa
ThispressureiscorrelatedasalosscoefficientusingEq.(6.78):
p filter
391Pa
K filter

2.5
2
(1/2) V
(1/2)(1.43kg/m3 )(14.85m/s)2

Ans.

252

Chapter 6Viscous Flow in Ducts

6.102A70percentefficientpumpdeliverswaterat20Cfromonereservoirtoanother
20fthigher,asinFig.P6.102.Thepipingsystemconsistsof60ftofgalvanizediron2in
pipe,areentrantentrance,twoscrewed90longradiuselbows,ascrewedopengatevalve,
andasharpexit.Whatistheinputpowerrequiredinhorsepowerwithandwithouta6well
3
designedconicalexpansionaddedtotheexit?Theflowrateis0.4ft /s.

Fig.P6.102

Solution:For water at 20C, take 1.94 slug/ft and 2.09E5 slug/fts. For
galvanized iron, 0.0005ft,whence /d 0.0005/(2/12ft) 0.003.Withoutthe6
cone,theminorlossesare:
K reentrant 1.0; K elbows 2(0.41); K gatevalve 0.16; K sharpexit 1.0
Evaluate V

Q
0.4
ft
Vd 1.94(18.3)(2/12)

18.3 ; Re

284000
2
A (2/12) /4
s

2.09E5

AtthisReandroughnessratio,wefindfromtheMoodychartthatf0.0266.Then
(a) h pump z

60
V2 L
(18.3)2

20

0.0266

1.0 0.82 0.16 1.0


2g d
2(32.2)

2/12

gQh p (62.4)(0.4)(85.6)

0.70
3052 550 5.55 hp Ans.(a)

or h pump 85.6ft, Power

(b)Ifwereplacethesharpexitbya6conicaldiffuser,fromFig.6.23,Kexit0.3.Then
h p 20

60
(18.3)2
1.0 .82 .16 0.3 81.95ft
0.0266

2(32.2)
2/12

then Power (62.4)(0.4)(81.95)/0.7 550 5.31hp (4%less)

Ans.(b)

253

Solutions ManualFluid Mechanics, Seventh Edition

6.103ThereservoirsinFig.P6.103areconnectedbycastironpipesjoinedabruptly,
withsharpedgedentranceandexit.Includingminorlosses,estimatetheflowofwaterat
20Cifthesurfaceofreservoir1is45fthigherthanthatofreservoir2.

Fig.P6.103
3

Solution:Forwaterat20C,take1.94slug/ft and2.09E5slug/fts.Leta
bethesmallpipeandbthelarger.Forwroughtiron, 0.00015ft,whence /da
0.0018and/db0.0009.Fromthecontinuityrelation,
Q Va

1
d a Vb d 2b or,since d b 2d a , weobtain Vb Va
4
4
4

For pipe a there are two minor losses: a sharp entrance, K1 0.5, and a sudden
2 2
expansion,Fig.6.22,Eq.(6.101),K2 [1 (1/2) ] 0.56.Forpipebthereisone
minorloss,thesubmergedexit,K 31.0.Theenergyequation,withequalpressuresat
(1)and(2)andnearzerovelocitiesat(1)and(2),yields
z h fa h ma h fb h mb

Va2 La
V b2
L
f

0.5

0.56

fb b 1.0
,
a

2g d a
2g
db

or,since Vb Va /4, z 45ft

Va2
120
1.0
240fa 1.06
fb

2(32.2)
16
16

wherefaandfbareseparatelyrelatedtodifferentvaluesofReand/d.Guesstostart:
fa fb 0.02: then Va 21.85ft/s, Re a 169000, /d a 0.0018, fa2 0.0239
Vb 5.46ft/s, Re b 84500, /d b 0.0009, fb2 0.0222
Convergesto: fa 0.024, fb 0.0224, Va 20.3ft/s,
Q Va Aa 0.111ft 3 /s. Ans.

254

Chapter 6Viscous Flow in Ducts

P6.104
Water at 20C flows through a smooth horizontal pipe at 12 m3/h. The pipe
diameter is 5 cm, and its length is 10 m. (a) Compute the pressure drop. (b) If a gate
valve is added, what gate opening h/D will reduce the flow rate by 50% for the same
pressure drop?
Solution: For water at 20C, Table A.3, = 998 kg/m3 and = 0.001 kg/m-s. (a) First
compute the Reynolds number from the known flow rate and then get the (smooth)
friction factor:
V

Q
(12 / 3600)
m
VD (998)(1.70)(0.05)

1.70
; Re D

84, 700
2
A ( / 4)(0.05)
s

0.001
1
Eq.(6.38) :
2.0 log(84700 f ) 0.8 yields f 0.0186
f

Then the pressure drop is given by


L 2
10m 998
p f
V (0.0186)(
)(
)(1.70) 2 5350 Pa
D 2
0.05m 2

Ans.(a )

(b) The partly closed gate valve cuts the flow rate in half to 6 m3/s, or V = 0.85 m/s.
This cuts the Reynolds number in half, to 42,350, which slightly increases the (smooth)
friction factor. From Eq. (6.38), the new friction factor is f = 0.0217. Add the K factor of
the valve to p:

2 L
998
10
V ( f K valve ) (
)(0.85)2 [(0.0217)(
) K valve ]
2
D
2
0.05
or 14.8 4.34 K valve ,
Solve for
K valve 10.5
Figure 6.18b shows that, for the gate valve, a minor loss coefficient K 10.5 is reached
for a gate opening h/D 0.35. Ans.(b)
p 5350 Pa

6.105ThesysteminFig.P6.105consistsof1200mof5cmcastironpipe,two45 and
four90flangedlongradiuselbows,afullyopenflangedglobevalve,andasharpexit
intoareservoir.Iftheelevationatpoint1is400m,whatgagepressureisrequiredat
3
point1todeliver0.005m /sofwaterat20Cintothereservoir?

255

Solutions ManualFluid Mechanics, Seventh Edition

Solution:Forwaterat20C,take998kg/m and0.001kg/ms.Forcastiron,
take0.26mm,hence/d0.0052.Withtheflowrateknown,wecancomputeV,Re:

Fig.P6.105

Q
0.005
m
998(2.55)(0.05)

2.55 ; Re
127000, fMoody 0.0315
2
A ( /4)(0.05)
s
0.001

Theminorlossesmaybelistedasfollows:
45 longradiuselbow: K 0.2; 90 longradiuselbow:K 0.3
Openflangedglobevalve: K 8.5; submergedexit:K 1.0
Thentheenergyequationbetween(1)and(2thereservoirsurface)yields
p1 V12

z 0 0 z 2 h f h m,
g 2g 1
(2.55)2

1200
or: p1 /(g) 500 400
0.0315
0.5 2(0.2) 4(0.3) 8.5 1 1

0.05
2(9.81)

100 253 353m, or: p1 (998)(9.81)(353) 3.46 MPa

Ans.

256

Chapter 6Viscous Flow in Ducts

6.106ThewaterpipeinFig.6.106slopes
upwardat30.Thepipeis1inchdiameter
and smooth. The flanged globe valve is
fully open. If the mercury manometer
showsa7inchdeflection,whatistheflow
rateincubicfeetpersec?

Fig.P6.106

Solution:For water at 20C, take


3
1.94slug/ft and2.09E5slug/fts.The
pipelengthandelevationchangeare
10ft
11.55ft; z 2 z1 10 tan 30 5.77ft, Open1 globevalve: K 13
cos30
Themanometerindicatesthetotalpressurechangebetween(1)and(2):
L

7
62.4(5.77) 819psf
12

p1 p2 (Merc w )gh w gz (13.6 1)(62.4)


Theenergyequationyields

p1 p2
V 2 11.55
819lbf/ft 2
z h f h m 5.77
f
13
g
2(32.2) 1/12
62.4lbf/ft 3

2(32.2)(7.35)
ft
or: V 2
. Guess f 0.02, V 5.48 , Re 42400, fnew 0.0217
(139f 13)
s
2

Rapidconvergencetof0.0217,V5.44ft/s,QV( /4)(1/12) 0.0296ft /s.Ans.


[NOTEthatthemanometerreadingof7inchesexactlybalancesthefrictionlosses,and
thehydrostaticpressurechangegzcancelsoutoftheenergyequation.]
6.107InFig.P6.107thepipeisgalvanizediron.Estimatethepercentageincreaseinthe
flowrate(a)ifthepipeentranceiscutoffflushwiththewalland(b)ifthebutterfly
valveisopenedwide.

Fig.P6.107

257

Solutions ManualFluid Mechanics, Seventh Edition

Solution:Forwaterat20C,take998kg/m and0.001kg/ms.Forgalvanized
iron,take0.15mm,hence/d0.003.Firstestablishminorlossesasshown:
Protrudingentrance(Fig.6.21a),

L
1.2, K 1;
d

Butterfly@30(Fig6.19)K8020
Theenergyequation,withp1p2,yields:
V2
V2
z
hf hm
2g
2g

L
V2
2

1.0 80 20 5m
1 f d K 2(9.81) 1 f 0.05

m
, Re 54300,
s
m
0.0284, Vnew 1.086
s

Guess f 0.02, V 1.09


fnew

0.003,
d

Thusthebaseflow,forourcomparison,isVo1.086m/s,Qo0.00213m /s.
Ifwecutofftheentranceflush,wereduceKent from1.0to 0.5;hardlyasignificant
reductioninviewofthehugebutterflyvalvelossKvalve80.Theenergyequationis
5m

V2
m
[1 40f 0.5 80 20], solve V 1.090 ,
2(9.81)
s
Q 0.00214

m3
(0.3%more) Ans.(a)
s

Ifweopenthebutterflywide,Kvalvedecreasesfrom80toonly0.3,ahugereduction:
V2
m
5m
[1 40f 1.0 0.3], solve V 5.4 ,
2(9.81)
s
Q 0.0106

m3
(5timesmore) Ans.(b)
s

Obviouslyopeningthevalvehasadominanteffectforthissystem.

6.108ThewaterpumpinFig.P6.108maintainsapressureof6.5psigatpoint1.There
isafilter,ahalfopendiskvalve,andtworegularscrewedelbows.Thereare80ftof
3
4inchdiametercommercialsteelpipe.(a)Iftheflowrateis0.4ft /s,whatistheloss
coefficientofthefilter?(b)Ifthediskvalveiswideopenand Kfilter 7,whatisthe
resultingflowrate?

258

Chapter 6Viscous Flow in Ducts

Fig.P6.108
3

Solution:For water, take 1.94 slug/ft and 2.09E5 slug/fts. The energy
equationiswrittenfrompoint1tothesurfaceofthetank:
p1 V12
p2 V22
V12 fL

z1

z2
[ K valve K filter 2 K elbow K exit ]
g 2g
g 2g
2g D
3

(a)Fromtheflowrate,V1Q/A(0.4ft /s)/[( /4)(4/12ft) ]4.58ft/s.Lookupminor


lossesandenterintotheenergyequation:
(6.5)(144)lbf/ft 2 (4.58ft/s)2

0
62.4lbf/ft 3
2(32.2ft/s2 )
0 0 9ft

(4.58)2
80ft
f
2.8 K filter 2(0.64) 1

2(32.2) (4/12ft)

We can solve for Kfilter if we evaluate f. Compute ReD (1.94)(4.58)(4/12)/(2.09E5)


141,700.Forcommercialsteel, /D 0.00015ft/0.333ft 0.00045and V12/2g =0.326ft.
FromtheMoodychart,f0.0193,andfL/D4.62.Theenergyequationabovebecomes:
15.0ft 0.326ft 9ft 0.326(4.62 2.8 K filter 1.28 1)ft,
Solve K filter 9.7

Ans.(a)

(b)IfKfilter7.0andVisunknown,wemustiterateforthevelocityandflowrate.The
energyequationbecomes,withthediskvalvewideopen(KValve0):

V2
V 2 80
15.0ft
9ft
f
0 7.0 1.28 1

2(32.2)
2(32.2) 1/3
Iterateto find

f 0.0189, Re D 169,000, V 5.49ft/s,


Q AV 0.48ft 3 /s

Ans.(b)

259

Solutions ManualFluid Mechanics, Seventh Edition

6.109InFig.P6.109thereare125ftof
2inpipe,75ftof6inpipe,and150ftof
3in pipe, all cast iron. There are three
90 elbows and an open globe valve, all
flanged.Iftheexitelevationiszero,what
horsepower is extracted by the turbine
3
whentheflowrateis0.16ft /sofwaterat
20C?

Fig.P6.109

Solution:Forwaterat20C,take1.94slug/ft and2.09E5slug/fts.Forcast
iron,0.00085ft.The2,6,and3pipeshave,respectively,
(a)L/d750,/d0.0051;(b)L/d150,/d0.0017;
(c)L/d600,/d0.0034
Theflowrateisknown,soeachvelocity,Reynoldsnumber,andfcanbecalculated:
Va

0.16
ft
1.94(7.33)(2/12)
7.33 ; Rea
113500, fa 0.0314
2
s
2.09E5
(2/12) /4

Also, Vb 0.82ft/s, Re b 37800, fc 0.0266; Vc 3.26, Re c 75600, fc 0.0287


Finally,theminorlosscoefficientsmaybetabulated:
sharp2entrance:K0.5;three290elbows:K3(0.95)
2suddenexpansion:K0.79;3openglobevalve:K6.3
Theturbineheadequalstheelevationdifferenceminuslossesandtheexitvelocityhead:
h t z h f h m Vc2 /(2g)
(7.33)2
100
[0.0314(750) 0.5 3(0.95) 0.79]
2(32.2)

(0.82)2
(3.26)2
(0.0266)(150)
[0.0287(600) 6.3 1] 72.8 ft
2(32.2)
2(32.2)

TheresultingturbinepowergQht(62.4)(0.16)(72.8)5501.32hp.Ans.

260

Chapter 6Viscous Flow in Ducts

6.110InFig.P6.110thepipeentranceis
3
sharpedged.Iftheflowrateis0.004m /s,
what power, in W, is extracted by the
turbine?

Fig.P6.110

Solution:For water at 20C, take


3
998kg/m and0.001kg/ms.Forcast
iron, 0.26 mm, hence /d 0.26/50 0.0052. The minor loss coefficients are
Entrance:K0.5;5cm(2)openglobevalve:K6.9.
Theflowrateisknown,hencewecancomputeV,Re,andf:
V

Q
0.004
m
998(2.04)(0.05)

2.04 , Re
102000, f 0.0316
2
A ( /4)(0.05)
s
0.001

Theturbineheadequalstheelevationdifferenceminuslossesandexitvelocityhead:
V2
(2.04)2

125
40
(0.0316)
0.5 6.9 1 21.5m

2g
2(9.81)
0.05

Power gQh t (998)(9.81)(0.004)(21.5) 840W Ans.

h t z h f h m

6.111For the parallelpipe system of


Fig.P6.111,eachpipeiscastiron,andthe
2
pressuredropp1p23lbf/in .Compute
thetotalflowratebetween1and2ifthe
fluidisSAE10oilat20C.

Fig.P6.111

Solution:ForSAE10oilat20C,take1.69slug/ft and0.00217slug/fts.For
castiron,0.00085ft.Convertp3psi432psfandguesslaminarflowineach:
? 128 L a Q a

pa

d a4

432

128(0.00217)(250)Q a
,
(3/12)4

ft 3 .
Check Re 300 (OK)
s
? 128 L b Q b
128(0.00217)(200)Q b
p b
432
,
4
db
(2/12)4
Qa 0.0763

ft 3
. Check Re 112(OK)
s
Thetotalflowrateis Q Q a Q b 0.0763 0.0188 0.095ft 3 /s.
Q b 0.0188

Ans.

261

Solutions ManualFluid Mechanics, Seventh Edition

6.112IfthetwopipesinFig.P6.111areinsteadlaidin series withthesametotal


pressuredropof3psi,whatwilltheflowratebe?ThefluidisSAE10oilat20C.
3

Solution:ForSAE10oilat20C,take 1.69slug/ft and 0.00217slug/fts.


Againguesslaminarflow.Now,insteadofpbeingthesame,QaQbQ:
pa p b 432psf

128 L a Q 128L b Q 128(0.00217) 250


200

4
4
4

da
db
/
(2/12)4
(312)

Solve for Q 0.0151ft 3 /s

Ans. Check Rea 60(OK)and Re b 90(OK)

Inseries,theflowrateissixtimeslessthanwhenthepipesareinparallel.
6.113The parallel galvanizediron pipe
system of Fig. P6.113 delivers water at
3
20Cwithatotalflowrateof0.036m /s.If
thepumpiswideopenandnotrunning,with
alosscoefficientK1.5,determine(a)the
flow rateineachpipeand(b)theoverall
pressuredrop.

Fig.P6.113

Solution:Forwaterat20C,take998kg/m and0.001kg/ms.Forgalvanized
iron,0.15mm.Assumeturbulentflow,withpthesameforeachleg:
h f1 f1

L1 V12
V2 L
h f2 h m2 2 f2 2 1.5 ,
d1 2g
2g
d2

and Q1 Q 2 ( /4)d12 V1 ( /4)d 22 V2 Q total 0.036m 3 /s


WhenthefrictionfactorsarecorrectlyfoundfromtheMoodychart,thesetwoequations
maybesolvedforthetwovelocities(orflowrates).Beginbyguessingf0.020:
V12
V22
55

60

(0.02)

(0.02)
1.5 , solvefor V1 1.10V2

0.05 2(9.81) 2(9.81)


0.04

then

m
m
(0.05)2 (1.10V2 ) (0.04)2 V2 0.036. Solve V2 10.54 , V1 11.59
4
4
s
s
Correct Re1 578000, f1 0.0264, Re 2 421000, f2 0.0282, repeat.

The2nditerationconverges:f10.0264,V111.69m/s,f20.0282,V210.37m/s,
3

Q1A1V10.023m /s,Q2A2V20.013m /s.Ans.(a)


Thepressuredropisthesameineitherleg:

262

Chapter 6Viscous Flow in Ducts

V22
L1 V12 L 2
p f1
f2
1.5
2.16E6 Pa
d1 2
d2
2

Ans.(b)

*P6.114 Ablowersuppliesstandardairtoaplenumthatfeedstwohorizontalsquaresheet

metalductswithsharpedgedentrances.Oneductis100ftlong,withacrosssection6inby6
in.Thesecondductis200ftlong.Eachductexhauststotheatmosphere.Whentheplenum
pressureis5.0lbf/ft2 gage,thevolumeflowinthelongerductisthreetimestheflowinthe
shorterduct.Estimatebothvolumeflowsandthecrosssectionsizeofthelongerduct.

Solution:Forstandardair,inBGunits,take=0.00238slug/ft3and=3.78E7slug/ft
sec.Forsheetmetal,take=0.00016ft.Theenergyequationforthiscaseis
p1
g

V12
p2
V2
z1
2 z 2 h f hentrance , or :
2g
g
2g
1
L
p
V 2 (1 f
K ent ) where K sharp edged 0.5
2
Dh

WehaveabbreviatedtheductvelocitytoV,withoutasubscript.Forasquareduct,thehydraulic
diameteristhesidelengthofthesquare.Firstcomputetheflowrateintheshortduct:

lbf 0.00238slug / ft3 100 ft

5 .0 2
{1 f 0.5) , f fcn(ReD , )
hD
2
0 .5 f t
ft
h
L 100 ft : Re D

87,000 ; f 0.0200 ; V 27.7 ft / s ; Qshort 6.92 ft 3 /s

TheReynoldsnumberfortheshortductis Re=(0.00238)V(0.5)/(3.78E7)=3148V,
and/Dh=0.00016ft/0.5ft=0.00032.Thesolutionis

263

Solutions ManualFluid Mechanics, Seventh Edition

Forthelongerduct,Re=(0.00238)VDh/(3.78E7),and/Dh=0.00016ft/Dh.Wedont
knowDhandmustsolvetomakeQlong=3Qshort.Thesolutionis
L 200 ft : Re D

150,000 ; f 0.0177 ; V 27.5 ft / s ; Qlong 20.75 ft 3 / s


Solve for

Dh,long 0.87 ft

Ans.

NOTE: It is an interesting numerical quirk that, for these duct parameters, the velocities
in each duct are almost identical, regardless of the magnitude of the
pressure drop.

6.115InFig.P6.115allpipesare8cmdiametercastiron.Determinetheflowrate
fromreservoir(1)ifvalveCis(a)closed;and(b)open,withKvalve0.5.

Fig.P6.115
3

Solution:Forwaterat20C,take998kg/m and0.001kg/ms.Forcastiron,
0.26mm,hence/d0.26/800.00325forallthreepipes.Notep1p2,V1V20.
Thesearelongpipes,butwemightwishtoaccountforminorlossesanyway:
sharpentranceatA:K10.5;linejunctionfromAtoB:K20.9(Table6.5)
branchjunctionfromAtoC:K31.3;twosubmergedexits:KBKC1.0

Chapter 6Viscous Flow in Ducts

264

IfvalveCisclosed,wehaveastraightseriespaththroughAandB,withthesameflow
rateQ,velocityV,andfrictionfactorfineach.Theenergyequationyields
z1 z 2 h fA h mA h fB h mB ,
V 2 100
50


or: 25m
f
0.5 0.9 f
1.0 , where f fcn Re,

2(9.81) 0.08
0.08
d

Guessfffullyrough0.027,thenV3.04m/s,Re998(3.04)(0.08)/(0.001)243000,
/d0.00325,thenf0.0273(converged).ThenthevelocitythroughAandBisV
3
2
3.03m/s,andQ( /4)(0.08) (3.03)0.0152m /s.Ans.(a).
IfvalveCisopen,wehaveparallelflowthroughBandC,withQ AQBQCand,with
dconstant,VAVBVC.ThetotalheadlossisthesameforpathsABandAC:
z1 z 2 h fA h mAB h fB h mB h fA h mAC h fC h mC ,
or: 25

VA2
100
VB2
50

0.5

0.9

f
1.0
A
B

2(9.81)
0.08
2(9.81)
0.08

VC2
VA2
100
70

fA
0.5 1.3
fC
1.0

2(9.81)
0.08
2(9.81)
0.08

plustheadditionalrelationVAVBVC.Guessfffullyrough0.027forallthreepipes
andbegin.Theinitialnumbersworkoutto
2g(25) 490.5 VA2 (1250fA 1.4) VB2 (625fB 1) VA2 (1250fA 1.8) VC2 (875fC 1)
Iff 0.027, solve(laboriously) VA 3.48m/s,VB 1.91m/s,VC 1.57m/s.
Compute Re A 278000, fA 0.0272, Re B 153000, fB 0.0276,
Re C 125000, fC 0.0278
Repeatonceforconvergence:VA3.46m/s,VB1.90m/s,VC1.56m/s.Theflow
3
2
ratefromreservoir(1)isQA(/4)(0.08) (3.46)0.0174m /s.(14%more)Ans.(b)

6.116For the seriesparallel system of Fig. P6.116, all pipes are 8cmdiameter
asphaltedcastiron.Ifthetotalpressuredropp1p2750kPa,findtheresultingflow
3
rateQm /hforwaterat20C.Neglectminorlosses.

265

Solutions ManualFluid Mechanics, Seventh Edition

Solution:Forwaterat20C,take998kg/m and0.001kg/ms.Forasphalted
castiron,0.12mm,hence/d0.12/800.0015forallthreepipes.Theheadlossis
thesamethroughACandBC:

Fig.P6.116

L V2
2
p
LV
L V2
h fA h fC h fB h fC f

g
d 2g
d 2g
d 2g A

L V2
f
d 2g
B

Sincedisthesame,VAVBVCandfA,fB,fCarefoundfromtheMoodychart.Cancel
gandintroducethegivendata:
750000
250 VA2
150 VC2
100 VB2
150 VC2
fA
fC
fB
fC
, VA VB VC
998
0.08 2
0.08 2
0.08 2
0.08 2
m
m
m
Guessfrough 0.022andsolvelaboriously: VA 2.09 , VB 3.31 , VC 5.40
s
s
s
NowcomputeReA167000,fA0.0230,ReB264000,fB0.0226,ReC431000,
andfC0.0222.Repeattheheadlossiterationandweconverge:V A2.06m/s,VB
3
2
3.29m/s,VC5.35m/s,Q( / 4)(0.08) (5.35)0.0269m /s.Ans.

6.117Ablowerdeliversairat3000m /htotheductcircuitinFig.P6.117.Eachductis
commercialsteelandofsquarecrosssection,withsidelengthsa1a320cmanda2
a412cm.Assumingsealevelairconditions,estimatethepowerrequirediftheblower
hasanefficiencyof75%.Neglectminorlosses.

266

Chapter 6Viscous Flow in Ducts

Solution:Forairtake 1.2kg/m and 1.8E5kg/ms.Establishconditionsin


eachduct:

Fig.P6.117

3000
m3
0.833m 3 /s
1.2(20.8)(0.2)
0.833
; V1&3
20.8m/s; Re1&3
278,000
2
3600
s
1.8 E5
(0.2m)
0.833m 3 /s
1.2(57.8)(0.12)
V2&4

57.8m/s;
Re

463,000
2&4
1.8 E5
(0.12m)2

Forcommercialsteel(Table6.1)0.046mm.Thenwecanfindthetwofrictionfactors:

1&3 0.046 0.00023; Re1&3 278000; Moodychart: f1&3 0.0166


D
200

2&4 0.046 0.000383; Re2&4 463000; Moodychart: f1&3 0.0170


D
120
2
L V 2
80 (1.2)(20.8)
Then p1&3 f
(0.0166)
1730Pa
0.2

2
D 2
1&3
2
L V 2
60 (1.2)(57.8)
and p2&4 f

(0.0170)
17050Pa

0.12
2
D 2
1&3

Thetotalpowerrequired,at75%efficiency,isthus:
Power

Qp (0.833m 3 /s)(1730 17050Pa)

20900W

0.75

Ans.

267

Solutions ManualFluid Mechanics, Seventh Edition

*6.118ForthepipingsystemofFig.P6.118, allpipesareconcretewitharoughness
of0.04inch.Neglecting minorlosses,computetheoverall pressuredropp 1 p2 in
2
3
lbf/in .Theflowrateis20ft /sofwaterat20C.
3

Solution:Forwaterat20C,take1.94slug/ft and2.09E5slug/fts.Sincethe
pipesarealldifferentmakealittletableoftheirrespectiveL/dand/d:

Fig.P6.118

(a)
(b)
(c)
(d)

L1000ft,
1500ft
800ft
1200ft

d12in,
8in
12in
15in

L/d1000,
2250
800
960

/d0.00333
0.00500
0.00333
0.00267

Withtheflowrateknown,wecanfindeverythinginpipe(a):
Va

Qa
20
ft
1.94(25.5)(1)

25.5 , Re a
2.36E6, fa 0.0270
2
Aa ( /4)(1ft)
s
2.09E5

Thenpipes(b,c,d)areinparallel,eachhavingthesameheadlossandwithflowrates
3
whichmustadduptothetotalof20ft /s:
8fb L b Q 2b
8fc L c Q c2
8fd L d Q d2
ft 3
h fb 2 5 h fc 2 5 h fd 2 5 , and Q b Q c Q d 20
s
g db
g dc
g dd
1/2

IntroduceLb,db,etc.tofindthatQc3.77Qb(fb/fc) andQd5.38Qb(fb/fd)

1/2

Thentheflowratesareiteratedfromtherelation
ft 3
Q 20
Q b [1 3.77(fb /fc )1/2 5.38(fb /fd )1/2 ]
s
Firstguess: fb fc fd : Q b 1.97ft 3 /s; Q c 7.43ft 3 /s; Q d 10.6ft 3 /s
ImprovebycomputingReb 349000,fb 0.0306,Rec 878000,fc 0.0271,Red
3
3
3
1002000,fd0.0255.RepeattofindQb1.835ft /s,Qc7.351ft /s,Qd10.814ft /s.

268

Chapter 6Viscous Flow in Ducts

Repeatoncemoreandquit:Qb 1.833ft /s,Qc 7.349ft /s,Qd 10.819ft /s,from


whichVb5.25ft/s,Vc9.36ft/s,Vd8.82ft/s.Thepressuredropis
p1 p2 pa p b fa

L a Va2
L Vb2
fb b
da 2
db 2

17000 1800 18800psf 131

lbf
in 2

Ans.

P6.119
For the system of Prob. P6.111, again let the fluid be SAE 10W oil at 20C,
with both pipes cast iron. If the flow rate in the 2-inch pipe (b) is 3.0 ft3/min, estimate the
flow rate in the 3-inch pipe (a), in ft3/min.

Solution: For SAE 10W oil at 20C, = 1.69 slug/ft3 and = 0.00219 slug/ft-s. The oil
is pretty viscous and the flow rate small, so we think ReDb will be laminar:
Re Db

Vb Db
4 Qb
4(1.69)(3.0 / 60)

295 2300

Db
(0.00219)(2 /12)

If we need it, Vb

4 Qb
Db2

4(3.0 / 60 ft 3 / s)
2

2.29

Yes, Laminar

ft
s

(2 /12 ft )
For laminar flow, compute the pressure drop pb from Eq. (6.12) and set it equal to pa:
pb

128 Lb Qb

128(0.00219)(200)(3 / 60)

1156 Pa
(2 /12) 4
128 La Qa
128(0.00219)(250)Qa
Parallel Pipes : pa pb 1156

Da4
(3 /12) 4

Db4

Solve for

ft 3
ft 3
Qa 0.2025
12.2
s
min

Check Rea = 4Q/Da) = 4(1.69)(0.2025)/[ 796

Ans.
Yes, laminar also.

269

Solutions ManualFluid Mechanics, Seventh Edition

6.120Threecastironpipesarelaidinparallelwiththesedimensions:
Pipe1:
Pipe2:
Pipe3:

L1800m
L2600m
L3900m

d112cm
d28cm
d310cm

Thetotalflowrateis200m /hofwaterat20C.Determine(a)theflowrateineachpipe;
and(b)thepressuredropacrossthesystem.
3

Solution:Forwaterat20C,take998kg/m and0.001kg/ms.Forcastiron,
0.26mm.Then,/d10.00217,/d20.00325,and/d30.0026.Theheadlossesare
thesameforeachpipe,andtheflowratesadd:
hf

8 f1L1Q12 8 f2 L2Q22 8 f3 L3Q32


200 m3

;
and
Q

1
2
3
3600 s
2 gd15
2 gd25
2 gd35

Substituteandcombine: Q1[1 0.418( f1/ f2 )1/2 0.599( f1/ f3 )1/2 ] 0.0556m 3 /s


WecouldeithergodirectlytoEESorbeginbyguessingf1f2f3,whichgivesQ1
3
3
3
0.0275 m /s, Q2 0.0115 m /s, and Q3 0.0165 m /s. This is very close! Further
iterationgives
Re1 298000, f1 0.0245; Re 2 177000, f2 0.0275; Re 3 208000, f3 0.0259
Q1 0.0281m 3 /s, Q2 0.0111m 3 /s, and Q3 0.0163m 3 /s

Ans.(a)

hf 51.4m, p ghf (998kg/m 3 )(9.81m/s2 )(51.4m) 503, 000Pa

6.121Consider the threereservoir


systemofFig.P6.121withthefollowing
data:
L195m
z125m

L2125m L3160m
z2115m z385m

All pipes are 28cmdiameter unfinished


concrete( 1mm).Computethesteady
flowrateinallpipesforwaterat20C.

Fig.P6.121

Ans.(b)

270

Chapter 6Viscous Flow in Ducts

Solution:Forwaterat20C,take998kg/m and0.001kg/ms.Allpipeshave
/d1/2800.00357.Lettheintersectionbea.Theheadlossataisdesired:
z1 ha f1

L V2
L1 V12
L V2
; z 2 h a f2 2 2 ; z 3 h a f3 3 3
d1 2g
d 2 2g
d3 2g

plustherequirementthatQ1Q2Q30or,forsamed,V1V2V30
WeguesshatheniterateeachfrictionfactortofindVandQandthencheckifQ0.
V12
m
95
, solve f1 0.02754, V1 10.25

0.28 2(9.81)
s
2
Similarly, 115 75 f2 (125/0.28) V2 /2(9.81)
gives f2 0.02755. V2 7.99

h a 75m: 25 75 ()50 f1

and 85 75 f3 (160/0.28) V32 /2(9.81)


m
, V 1.27
s
Repeatingforha80mgivesV110.75,V27.47,V32.49m/s,V0.79.
Interpolatetoha78m,givesV110.55m/s,V27.68m/s,V32.95m/s,or:
gives f3 0.02762, V3 3.53

Q10.65m /s,Q20.47m /s,Q30.18m /s.Ans.


6.122ModifyProb.6.121byreducingthediameterto 15cm,with1mm.Compute
theflowrateineachpipe.Theyallreduce,comparedtoProb.6.121,byafactorofabout
5.2.Canyouexplainthis?
Solution:Theroughnessratioincreasesto/d1/1500.00667,andallL/dsincrease.
Guessha75m:convergestof10.0333,f20.0333,f40.0334
andV16.82m/s,V25.32m/s,V32.34m/s,V0.85
Wefinallyobtainha78.2m,givingV17.04m/s,V25.10m/s,V31.94
m/s,
3

or:Q10.124m /s,Q20.090m /s,Q30.034m /s.Ans.


*6.123ModifyProb.6.121asfollows.Letz 3beunknownandfinditsvaluesuchthat
3
theflowrateinpipe3is0.2m /stowardthejunction.(Thisproblemisbestsuitedfor
computeriterationorEES.)

271

Solutions ManualFluid Mechanics, Seventh Edition

Solution:Forwaterat20C,take998kg/m and0.001kg/ms.Allpipeshave
/d1/2800.00357.Lettheintersectionbea.Theheadlossataisdesiredforeach
inordertochecktheflowrateinpipe3.
3

InProb.6.121,withz385m,wefoundQ3tobe0.18m /stowardthejunction,pretty
close.Werepeattheprocedurewithafewnewvaluesofz3,closingtoQ0eachtime:
Guessz3 85m:

h a 78.19m, Q1 0.6508,

Q 2 0.4718,

Q 3 0.1790m 3 /s

0.6657,

0.6657,

0.2099m 3 /s

90m:

80.65m,

Interpolate: h a 79.89, Q1 0.6611,


Q 2 0.4608, Q3 0.200 m 3 /s, z 3 88.4m

6.124The threereservoir system in


Fig. P6.124 delivers water at 20C. The
systemdataareasfollows:
D18in

D26in

D39in

L11800ft

L21200ft

L31600ft

Ans.

Fig.P6.124

Allpipesaregalvanizediron.Computethe
flowrateinallpipes.

Solution:Forwaterat20C,take 1.94slug/ft and 2.09E5slug/fts.For


galvanizediron,take0.0005ft.Thentheroughnessratiosare

/d1 0.00075 /d2 0.0010 /d3 0.00667


Lettheintersectionbea.Theheadlossataisdesired:
f1L1 V12
f2 L 2 V22
f3 L 3 V32
z1 h a
; z2 ha
; z3 ha
; plus Q1 Q 2 Q 3 0
d1 2g
d 2 2g
d 3 2g

WeguesshatheniterateeachfrictionfactortofindVandQandthencheckifQ0.

272

Chapter 6Viscous Flow in Ducts

f1 (1800)V12
Guess h a 50ft: 20 50 ( )30ft
,
(8/12)2(32.2)
ft
solve f1 0.0194, V1 6.09
s
Similarly,f2 0.0204,V2 8.11ft/sandofcourseV3 0. Get Q 0.54ft 3 /s

TryagainwithaslightlylowerhatoreduceQ1andincreaseQ2andQ3:
h a 48ft: convergesto Q1 2.05
Q3 0.76

ft 3
ft 3
, Q 2 1.62 ,
s
s

ft 3
, Q 0.33
s

Interpolateto
h a 49.12ft: Q1 2.09ft 3 /s, Q 2 1.61ft 3 /s, Q3 0.49ft 3 /s

6.125Suppose that the three castiron


pipesinProb.6.120areinsteadconnected
tomeetsmoothlyatapointB,asshownin
Fig. P6.125. The inlet pressures in each
pipeare:p1200kPa;p2160kPa;p3
100 kPa. The fluid is water at 20C.
Neglect minor losses. Estimate the flow
rateineachpipeandwhetheritistoward
orawayfrompointB.

Ans.

Fig.P6.125

Solution:Forwatertake 998kg/m
and0.001kg/ms.Thepressureatpoint
Bmustbeaknown(constant)valuewhich
makesthenetflowrateequaltozeroatjunctionB.Theflowclearlygoesfrom(1)toB,
andfromBto(3),butwearenotsureaboutpipe(2).Forcastiron(Table6.1), 0.26mm.
Eachpipehasaflowratebaseduponitspressuredrop:
p1 pB f1

L1 V12
;
D1 2

p2 pB f2

L2 V22
;
D2 2

pB p3 f3

L3 V32
D3 2

wherethe fsaredeterminedfromtheMoodychartforeachpipes /D andReD.The


2
correctvalueofpBmakestheflowrates Qi(/4)Di VibalanceatjunctionB.EESis
excellentforthistypeofiteration,andthefinalresultsbalanceforpB166.7kPa:

273

Solutions ManualFluid Mechanics, Seventh Edition

f1 0.0260; Re1 74300; /D1 0.00217; Q1 0.00701m 3 /s (towardB)


f2 0.0321; Re 2 18900; /D2 0.00325; Q2 0.00119m 3 /s (awayfromB) Ans.
f3 0.0270; Re3 74000; /D3 0.00260; Q3 0.00582m 3 /s (awayfromB)
*6.126ModifyProb.6.124asfollows.Letalldatabethesameexceptthatpipe1is
fittedwithabutterflyvalve(Fig.6.19b).Estimatethepropervalveopeningangle(in
3
degrees)fortheflowratethroughpipe1tobereducedto1.5ft /stowardreservoir1.
(ThisproblemrequiresiterationandisbestsuitedtoadigitalcomputerorEES.)
3

Solution:Forwaterat20C,take 1.94slug/ft and 2.09E4slug/fts.For


galvanizediron,take0.0005ft.Thentheroughnessratiosare

/d1 0.00075 /d2 0.0010 /d3 0.00667


ForabutterflyvalvelosscoefficientK(tobefound).LetthejunctionbeJ.Thehead
lossatJisdesiredandthentobeiteratedtogivetheproperflowrateinpipe(1):
z1 h J

V12
2g

L
V22 L
f

K
;
z

2
J
f
d 1
2g d

;
2

V32 L
z3 h J
f ; and Q1 Q 2 Q 3 0
2g d 3
Weknowz120ft,z2100ft,andz350ft.FromProb.6.124,whereK0,theflow
3
ratewas2.09ft /stowardreservoir1.NowguessafinitevalueofKandrepeat:
K 40: convergesto h J 50.0, Q1 1.59ft 3 /s, Q 2 1.59ft 3 /s; Q3 0
K 50: convergesto h J 50.03ft, Q1 1.513 Q 2 1.591 Q 3 0.078
K 52: gives h J 50.04ft, Q1 1.500 ft 3 /s Q 2 1.591 Q 3 0.091 Ans.
FromFig.6.19b,abutterflyvalvecoefficientK52occursatopening35.Ans.
*6.127InthefivepipehorizontalnetworkofFig.P6.127,assumethatallpipeshavea
3
frictionfactorf0.025.Forthegiveninletandexitflowrateof2ft /sofwaterat20C,
2
determinetheflowrateanddirectioninallpipes.IfpA120lbf/in gage,determinethe
pressuresatpointsB,C,andD.

274

Chapter 6Viscous Flow in Ducts

Solution:Forwaterat20C,take1.94slug/ft and2.09E5slug/fts.Eachpipe
hasaheadlosswhichisknownexceptforthesquareoftheflowrate:

Fig.P6.127

PipeAC: h f

8fLQ 2
2 gd 5

AC
AC 8(0.025)(3000)Q
K ACQ 2AC ,
2
5

(32.2)(6/12)

where K AC 60.42

ft 3
Similarly, K AB 19.12, K BC 13.26, K CD 19.12, K BD 1933. Qin
s

Therearetwotriangularclosedloops,andthetotalheadlossmustbezeroforeach.
UsingtheflowdirectionsassumedonthefigureP6.127above,wehave
LoopABC: 19.12Q 2AB 13.26Q 2BC 60.42Q 2AC 0
2
LoopBCD: 13.26Q 2BC 19.12Q CD
1933.0Q 2BD 0

Andtherearethreeindependentjunctionswhichhavezeronetflowrate:
JunctionA: Q AB Q AC 2.0; B: Q AB Q BC Q BD ; C: Q AC Q BC Q CD
Thesearefivealgebraicequationstobesolvedforthefiveflowrates.Theanswersare:
Q AB 1.19, Q AC 0.81, Q BC 0.99, Q CD 1.80, Q BD 0.20

ft 3
s

Ans.(a)

ThepressuresfollowbystartingatA(120psi)andsubtractingoffthefrictionlosses:
p B p A gK ABQ 2AB 120 144 62.4(19.12)(1.19)2
p B 15590psf 144 108

lbf
in 2

275

Solutions ManualFluid Mechanics, Seventh Edition

Similarly, p C 103psi and p D 76psi

Ans.(b)

*6.128ModifyProb.6.127aboveasfollows:LettheinletflowatAandtheexitflow
atDbeunknown.LetpApB100psi.Computetheflowrateinallfivepipes.
Solution:OurheadlosscoefficientsKfromaboveareallthesame.HeadlossABis
known,pluswehavetwoloopequationsandtwojunctionequations:
p A p B 100 144

231ft K ABQ 2AB 19.12Q 2AB , or Q AB 3.47ft 3 /s


g
62.4
Twoloops: 231 13.26Q 2BC 60.42Q 2AC 0
2
13.26Q 2BC 19.12Q CD
1933.0Q 2BD 0

Twojunctions:QAB3.47QBCQBD;QACQBCQCD
ThesolutionsareinexactlythesameratioasthelowerflowratesinProb.6.127:
ft 3
ft 3
ft 3
, Q BC 2.90
, Q BD 0.58
,
s
s
s
ft 3
ft 3
5.28
, Q AC 2.38
Ans.
s
s

Q AB 3.47
Q CD

6.129In Fig.P6.129 all four horizontal


castironpipesare45mlongand8cmin
diameterandmeetatjunctiona,delivering
waterat20C.Thepressuresareknownat
fourpointsasshown:
p1950kPa
p3675kPa

p2350kPa
p4100kPa

Neglectingminorlosses,determinetheflow
rateineachpipe.

Fig.P6.129

276

Chapter 6Viscous Flow in Ducts

Solution:Forwaterat20C,take998kg/m and0.001kg/ms.Allpipesare
castiron,with /d 0.26/80 0.00325.AllpipeshaveL/d 45/0.08 562.5.One
solution method is to guess the junction pressure p a, iterate to calculate the friction
factorsandflowrates,andchecktoseeifthenetjunctionflowiszero:
Guess pa 500kPa: h fl

950000 500000
8f L Q 2
45.96m 12 1 51 1.135E6f1Q12
998(9.81)
gd1

thenguess f1 0.02, Q1 0.045m 3 /s, Re1 4 Q1/(d1 ) 715000, f1new 0.0269


convergesto f1 0.0270, Q1 0.0388m 3 /s
m3
(awayfroma), Q 3 0.0241, Q 4 0.0365
s
Q 0.00403, sowehaveguessedpa alittlelow.

Iteratealsoto Q 2 0.0223

Tryingpa530kPagivesQ0.00296,henceiteratetopa517kPa:
m3
m3
Q1 0.0380
(towarda), Q 2 0.0236
,
s
s
m3
m3
Q3 0.0229
, Q 4 0.0373
Ans.
s
s
*6.130InFig.P6.130lengthsABandBD
are 2000 and 1500 ft, respectively. The
frictionfactoris0.022everywhere,and pA
2
90lbf/in gage.Allpipeshaveadiameter
of6in.Forwaterat20C,determinethe
flowrateinallpipesandthepressuresat
pointsB,C,andD.

Fig.P6.130

Solution:Forwaterat20C,take1.94slug/ft and2.09E5slug/fts.Eachpipe
hasaheadlosswhichisknownexceptforthesquareoftheflowrate:
8fLQ 2 8(0.022)(1500)Q 2AC
PipeAC: h f 2 5
K AC Q 2AC , where K AC 26.58
2
5
gd
(32.2)(6/12)
Similarly,KABKCD35.44,KBD26.58,andKBC44.30.
The solution is similar to Prob. 6.127, except that (1) the Ks are different; and
(2)junctionsBandChaveadditionalflowleavingthenetwork.Thebasicflowrelationsare:
LoopABC: 35.44Q2AB 44.3Q 2BC 26.58Q 2AC 0

277

Solutions ManualFluid Mechanics, Seventh Edition


2
LoopBCD: 44.3Q 2BC 35.44QCD
26.58Q 2BD 0

JunctionsA,B,C: Q AB Q AC 2.0;
Q AB Q BC Q BD 1.0; Q AC Q BC Q CD 0.5
InthiseraofPCequationsolverssuchasEES,itisprobablynotnecessarytodwell
uponanysolutionmethods.Forhandwork,onemightguessQAB,thentheotherfourare
obtainedinsequencefromtheaboverelations,plusacheckontheoriginalguessfor
QAB. The assumed arrows are shown above. It turns out that we have guessed the
directionincorrectlyonQBCabove,buttheothersareOK.Thefinalresultsare:
Q AB 0.949ft 3 /s (towardB); Q AC 1.051ft 3 /s (towardC)
Q BC 0.239 (towardB); Q CD 0.312 (towardD); Q BD 0.188 (toD)

Ans.(a)

ThepressuresstartatA,fromwhichwesubtractthefrictionlossesineachpipe:
pB pA gK ABQ 2AB 90 144 62.4(35.44)(0.949)2 10969psf 144 76psi
Similarly,weobtain p C 11127psf 77 psi; p D 10911 psf 76 psi

Ans.(b)

6.131Awatertunneltestsectionhasa1mdiameterandflowpropertiesV20m/s,p
100kPa,andT20C.Theboundarylayerblockageattheendofthesectionis9percent.
Ifaconicaldiffuseristobeaddedattheendofthesectiontoachievemaximumpressure
recovery,whatshoulditsangle,length,exitdiameter,andexitpressurebe?
3

Solution:Forwaterat20C,take998kg/m and0.001kg/ms.TheReynolds
numberisveryhigh,Re Vd/(998)(20)(1)/(0.001)muchhigherthanthe
diffuserdatainFig.6.28b(Re1.2E5).Butwhatcanwedo(?)Letsuseitanyway:
Bt 0.09, read C p,max 0.71 atL/d 25, 2 4, AR 8:
Then cone 2, L 25d 25 m, Dexit d(8)1/2 2.8 m
Cp 0.71

pe p t
p 100000
e
, or: pexit 242000 Pa
2
(1/2) Vt
(1/2)(998)(20)2

Ans.(a)
Ans.(b)

6.132ForProb.6.131,supposewearelimitedbyspacetoatotaldiffuserlengthof
10 meters. What should be the diffuser angle, exit diameter, and exit pressure for
maximumrecovery?

278

Chapter 6Viscous Flow in Ducts

Solution:WearelimitedtoL/D10.0.FromFig.6.28b,readCp,max0.62atAR4
and26.Ans.Theexitdiameterandpressureare
De d AR (1.0)(4.0)1/2 2.0 m

Ans.

Cp,max 0.62 (pe 100000)/[(1/2)(998)(20)2 ], or: p exit 224000 Pa

Ans.

6.133Awindtunneltestsectionis3ftsquarewithflowproperties V 150ft/s, p 15
2
lbf/in absolute,andT68F.Boundarylayerblockageattheendofthetestsectionis8
percent. Findtheangle,length,exitheight,andexitpressureofaflatwalleddiffuser
addedontothesectiontoachievemaximumpressurerecovery.
3

Solution:Forairat20Cand15psi,take0.00238slug/ft and3.76E7slug/fts.
TheReynoldsnumberisratherhigh,Re Vd/(0.00238)(150)(3)/(3.76E7)2.9E6;
muchhigherthanthediffuserdatainFig.6.28a(Re2.8E5).Butwhatcanwedo(?)
Letsuseitanyway:
Bt 0.08, readCp,max 0.70 at L/W1 17, 2 9.5, AR 3.75:
Then best 4.75, L 17W1 51ft, W2 (AR)W1 3.75(3) 11ft
Cp 0.70

pe p t
p e 15 144
lbf

, or: pexit 2180 2


2
2
(1/2)V1 (1/2)(0.00238)(150)
ft

Ans.
Ans.

6.134ForProb.6.133above,supposewearelimitedbyspacetoatotaldiffuserlength
of30ft.Whatshouldthediffuserangle,exitheight,andexitpressurebeformaximum
recovery?
Solution:WearelimitedtoL/W110.0.FromFig.6.28a,readCp,max0.645atAR
2.8and210.Ans.Theexitheightandpressureare
Wl,e (AR)W1 (2.8)(3.0) 8.4 ft

Ans.

Cp,max 0.645 [pe (15)144)]/[(1/2)(0.00238)(150)2 ], or pe 2180

lbf
ft 2

Ans.

6.135Anairplaneusesapitotstatictubeasavelocimeter.Themeasurements,with
theiruncertainties,areastatictemperatureof(113)C,astaticpressureof602kPa,

279

Solutions ManualFluid Mechanics, Seventh Edition

andapressuredifference(pops)320060Pa.(a)Estimatetheairplanesvelocityand
itsuncertainty.(b)Isacompressibilitycorrectionneeded?
2 2

Solution:Theairdensityis p/(RT) (60000Pa)/[(287m /s K)(262K)] 0.798


3
kg/m .
(a)EstimatethevelocityfromtheincompressiblePitotformula,Eq.(6.97):
V

2 p

2 p
2(3200Pa )
m

90
3
p/( RT )
s
0.798kg/m

The overall uncertainty involves pressure difference, absolute pressure, and absolute
temperature:
1/2

1/2

2
2
2
2
2
2
V 1 p 1 p
1 60 2
3
1 T


0.020

V
2 3200
2 T
60
262
2 p 2 p
Theuncertaintyinvelocityis2%,thereforeourfinalestimateisV902m/sAns.
(a)
1/2
1/2
ChecktheMachnumber.Thespeedofsoundisa(kRT) [1.4(287)(262)] 324m/s.
Therefore

MaV/a90/3240.280.3.Nocompressibilitycorrectionisneeded.Ans.(b)

6.136Forthepitotstaticpressurearrange
mentofFig.P6.136,themanometerfluid
is(colored)waterat20C.Estimate(a)the
centerline velocity, (b) the pipe volume
flow, and (c) the (smooth) wall shear
stress.

Fig.P6.136

280

Chapter 6Viscous Flow in Ducts

Solution:Forairat20Cand1atm,take1.2kg/m and1.8E5kg/ms.For
3
waterat20C,take998kg/m and0.001kg/ms.Themanometerreads
po p ( water air )gh (998 1.2)(9.81)(0.040) 391Pa
Therefore VCL [2 p/ ]1/2 [2(391)/1.2]1/2 25.5m/s

Ans.(a)

WecanestimatethefrictionfactorandthencomputeaveragevelocityfromEq.(6.43):
Guess Vavg 0.85VCL 21.7

m
Vd 1.2(21.7)(0.08)
, then Re d

115,700
s

1.8E5

Then fsmooth 0.0175, Vbetter

25.5
m
21.69 (converged)
s
[1 1.33

Thusthevolumeflowis Q ( /4)(0.08)2 (21.69) 0.109 m 3 /s. Ans.(b)


f
0.0175
Finally, w V 2
(1.2)(21.69)2 1.23 Pa
8
8
6.137For the 20C water flow of
Fig. P6.137, use the pitotstatic arrange
menttoestimate(a)thecenterlinevelocity
and (b) the volume flow in the 5in
diameter smooth pipe. (c) What error in
flowrateiscausedbyneglectingthe1ft
elevationdifference?

Ans.(c)

Fig.P6.137

Solution:For water at 20C, take


3
1.94slug/ft and2.09E5slug/fts.For
themanometerreadingh2inches,
poB p A (SG merc 1)(g)water h
water g(1ft) butfromtheenergyequation,
p A p B water gh fAB water g(1ft) ThereforepoB p B (SG 1)gh mano gh fAB
wherefrictionloss h fAB f(L/d)(V 2 /2g)
Thusthepitottubereadingequalsthemanometerreading(ofabout130psf)plusthe
frictionlossbetweenAandB(whichisonlyabout3psf),sothereisonlyasmallerror:
(SG 1)gh (13.56 1)(62.4)(2/12) 130.6psf, VCL

2p

1/2

2(130.6)

1.94

1/2

281

Solutions ManualFluid Mechanics, Seventh Edition

or VCL 11.6

ft
ft
1.94(9.9)(5/12)
, so Vavg 0.85VCL 9.9 , Re
381500,
s
s
2.09E5

so fsmooth 0.0138, or pfriction f(L/d)V 2 /2 3.2lbf/ft 2


If we now correct the pitot tube reading to p pitot 130.6 3.2 133.8 psf, we may
iterateandconvergerapidlytothefinalestimate:
f 0.01375, VCL

ft
ft 3
ft
11.75 ; Q 1.39
; Vavg 10.17
s
s
s

Ans.(a,b)

TheerrorcomparedtoourearlierestimateV 9.91ft/sisabout 2.6% Ans.(c)

6.138Anengineerwhotookcollegefluid
mechanicsonapassfailbasishasplaced
thestaticpressureholefarupstreamofthe
stagnation probe, as in Fig. P6.138, thus
contaminating the pitot measurement
ridiculouslywithpipefrictionlosses.Ifthe
pipeflowisairat20Cand1atmandthe
manometerfluidisMeriamredoil(SG
0.827),estimatetheaircenterlinevelocity
forthegivenmanometerreadingof16cm.
Assumeasmoothwalledtube.

Fig.P6.138

Solution:Forair at20Cand 1atm,take 1.2kg/m and 1.8E5kg/ms.


Becauseofthehighfrictionlossover10metersoflength,themanometeractuallyshows
poBlessthanpA,whichisabitweirdbutcorrect:
p A poB ( mano air )gh [0.827(998) 1.2](9.81)(0.16) 1294Pa
Meanwhile, p A p B gh f f

L V2
fL V 2
2
, or p oB p B
1294 VCL
d 2
d 2
2

10 1.2

1.2 V
Guess f 0.02, V 0.85VCL , whence 0.02
V 2 1294

0.06 2
2 0.85
Solvefor V 33.3

m
1.2(33.3)(0.06)
, Re d
133000, fbetter 0.0170,
s
1.8E5

V VCL /[1 1.33 f] 0.852VCL , repeattoconvergence


Finallyconverges, f 0.0164, V 39.87m/s, VCL V/0.8546 46.65m/s. Ans.

282

Chapter 6Viscous Flow in Ducts

6.139Professor Walter Tunnel must


measurevelocityinawatertunnel.Dueto
budgetary restrictions, he cannot afford a
pitotstatictube,soheinsertsatotalhead
probe and a statichead probe, as shown,
bothinthemainstreamawayfromthewall
boundary layers. The two probes are
connected to a manometer. (a) Write an
expressionfortunnelvelocityVintermsof
theparametersinthefigure.(b)Isitcritical
that h1 bemeasured accurately? (c)How
doespart(a)differfromapitotstatictube
formula?

Fig.P6.139

Solution:WriteBernoullifromtotalheadinlet(1)tostaticheadinlet(2):
po w gz1 ps

w 2
2( po ps w gh1 )
V w gz2 , Solve V
2
w

Combinethiswithhydrostaticsthroughthemanometer:
ps w gh2 m gh3 po w gh1 w gh2 w gh3 , cancelout w gh2
or: po ps w gh1 ( m w )gh3
IntroducethisintotheexpressionforVabove,forthefinalresult:
Vtunnel

2( m w )gh3
w

Ans.(a)

Thisisexactlythesameasapitotstatictubeh1isnotimportant.Ans.(b,c)
3

6.140Keroseneat20Cflowsat18m /hina5cmdiameterpipe.Ifa2cmdiameter
thinplateorificewithcornertapsisinstalled,whatwillthemeasuredpressuredropbe,in
Pa?
3

Solution:Forkeroseneat20C,take 804kg/m and 1.92E3kg/ms.The


orificebetaratiois2/50.4.ThepipevelocityandReynoldsnumberare:
V

Q
18/3600
m
804(2.55)(0.05)

2.55 , Re
53300
2
A ( /4)(0.05)
s
1.92E3

FromEqs.(6.112)and(6.113a)[cornertaps],estimateC d0.6030.Thentheorifice

283

Solutions ManualFluid Mechanics, Seventh Edition

pressuredropformulapredicts
Q

18

2 p
0.6030 (0.02)2
, solve p 273kPa
3600
4
804[1 (0.4)4 ]

Ans.

*6.141Gasolineat20Cflowsat105m /hina10cmdiameterpipe.Wewishtometer
theflowwithathinplateorificeandadifferentialpressuretransducerthatreadsbestat
about55kPa.Whatistheproperratiofortheorifice?
3

Solution:Forgasolineat20C,take 680kg/m and 2.92E4kg/ms.This


problemissimilartoExample6.21inthetext,butwedonthavetobesoprecisebecause
wedontknowtheexactgeometry:cornertaps, D: 12 D taps,etc.Thepipevelocityis
V1

Q
105/3600
m
680(3.71)(0.1)

3.71 , Re D
865000
2
A1 ( /4)(0.1)
s
2.92E4

FromFig.6.41,whichisreasonableforallorificegeometries,readCd0.61.Then
Vthroat

3.71m/s
2(55000)
Cd
, or
2

680(1 4 )
Solvefor

2
0.478
(1 4 )1/2

0.66 Ans.

CheckingbackwithFig.6.41,weseethatthisisaboutright,sonofurtheriterationis
neededforthislevelofaccuracy.

6.142The shower head in Fig. P6.142


delivers water at 50C. An orificetype
flow reducer is to be installed. The up
stream pressure is constant at 400 kPa.
Whatflowrate,ingal/min,resultswithout
thereducer?Whatreducerorificediameter
woulddecreasetheflowby40percent?
Solution:For water at 50C, take
3
988 kg/m and 0.548E3 kg/ms.
Furtherassumethattheshowerheadisa
poordiffuser,sothepressureintheheadis

Fig.P6.142

284

Chapter 6Viscous Flow in Ducts

alsoabout400kPa.Assumetheoutsidepressureissealevelstandard,101kPa.From
Fig.6.41foratypicalorifice,estimateC d0.61.Then,with0forthesmallholes,
eachholedeliversaflowrateof
Q1hole Cd A hole

2 p
2(400000 101000)

0.61 (0.0015)2
,
4
4
(1 )
988(1 0 4 )

or Q1hole 2.65E5m 3 /s and Q total 45Q1hole 0.00119

m3
gal
19

s
min

Thisisalargeflowratealotofexpensivehotwater.Checkingback,theinletpipefor
thisflowratehasReD183000,soCd0.60wouldbeslightlybetterandarepeatofthe
3
calculationwouldgiveQnoreducer0.00117m /s18.6gal/min.Ans.
3

A40%reductionwouldgiveQ0.6(0.00117)7.04E4m /s451.57E5m /sfor


eachhole,whichcorrespondstoapressuredrop
2 p

Q1hole 1.57E5 0.60 (0.0015)2
, or p 108000Pa
4
988
or p insidehead 101 108 209kPa,thereducermustdroptheinletpressuretothis.
2(400000 209000)

Q 7.04E4 0.61 (0.015 )2

988(1 4 )
4

1/2

, or

Solvefor 0.56, d reducer 0.56(1.5) 0.84cm

2
0.332
(1 4 )1/2
Ans.

6.143A10cmdiametersmoothpipecontainsanorificeplatewithD: 12 D tapsand
0.5.Themeasuredorificepressuredropis75kPaforwaterflowat20C.Estimatethe
3
flowrate,inm /h.Whatisthenonrecoverableheadloss?
3

Solution:Forwaterat20C,take 998kg/m and 0.001kg/ms.Weknow


everything intheorifice relation, Eq.(6.104),exceptC d,which wecanestimate (as
0.61):
Q Cd A t

2p

2(75000)
Cd (0.05)2
0.0249Cd
4
4
(1 )
998[1 (0.5)4 ]

Guess Cd 0.61, Q 0.0152

m3
4Q
, Re D
193000, Cd (Eq.6.112) 0.605
s
D

285

Solutions ManualFluid Mechanics, Seventh Edition

Thisisconverged:Q0.0249(0.605)0.0150m /s54m /h.Ans.(a)


(b)FromFig.6.44,thenonrecoverableheadlosscoefficientisK1.8,basedonVt:
Q
0.0150
m
Vt

7.66
,
A t (0.025)2
s
p loss K

2
998
Vt 1.8
(7.66)2 53000Pa

2
2

Ans.(b)

6.144Waterat20Cflowsthroughtheorificeinthefigure,whichismonitoredbya
3
mercury manometer. If d 3 cm, (a) what is h when the flow is 20 m h; and
(b)whatisQwhenh58cm?
Solution:(a)EvaluateVQA2.83m/sandReD VD141,000,0.6,thus
Cd0.613.

Fig.P6.144

20

2 p

2(13550 998)(9.81)h
Cd d 2
(0.613) (0.03)2
4
3600
4
4
(1 )
998(1 0.6 4 )

wherewehaveintroducedthemanometerformulap(mercurywater)gh.
Solvefor: h 0.58m 58 cm

Ans.(a)

Solvethisproblemwhenh58cmisknownandQistheunknown.Well,wecansee
thatthenumbersarethesameaspart(a),andthesolutionis
Solvefor: Q 0.00556m 3 /s 20m 3 /h
.

Ans.(b)

286

Chapter 6Viscous Flow in Ducts

6.145The1mdiametertankinFig.P6.145isinitiallyfilledwithgasolineat20C.
Thereisa2cmdiameterorificeinthebottom.Iftheorificeissuddenlyopened,estimate
thetimeforthefluidlevelh(t)todropfrom2.0to1.6meters.
3

Solution:Forgasolineat20C,take680kg/m and 2.92E4kg/ms.The

Fig.P6.145

orificesimulatescornertapswith0,so,fromEq.(6.112),Cd0.596.Fromtheenergy
equation,thepressuredropacrosstheorificeispgh(t),or
2gh

0.596 (0.02)2 2(9.81)h 0.000829 h
4
4
(1 )
d
dh

dh
Butalso Q ( tank ) A tank
(1.0m)2
dt
dt
4
dt

Q Cd A t

SettheQsequal,separatethevariables,andintegratetofindthedrainingtime:
1.6

2.0

final
dh
2[ 2 1.6 ]
0.001056 dt, or t final
283s 4.7min
0.001056
h
0

Ans.

6.146Apipeconnectingtworeservoirs,asinFig.P6.146,containsathinplateorifice.
For water flow at 20C, estimate (a) the volume flow through the pipe and (b)the
pressuredropacrosstheorificeplate.

287

Solutions ManualFluid Mechanics, Seventh Edition

Solution:Forwaterat20C,take 998kg/m and 0.001kg/ms.Theenergy


equationshouldincludetheorificeheadlossandtheentranceandexitlosses:

Fig.P6.146

z 20m
V2

V2 L

0.6
, where K entr 0.5, K exit 1.0, K orifice
1.5(Fig.6.44)
f K

2g d

2(9.81)(20)
392.4

; guess f 0.02, V 3.02m/s


[f(100/0.05) 0.5 1.0 1.5] 2000f 3.0
Iterateto fsmooth 0.0162, V 3.33m/s
3

ThefinalReVD/166000,andQ(/4)(0.05) (3.33)0.00653m /sAns.(a)


(b)Thepressuredropacrosstheorificeisgivenbytheorificeformula:
ReD166000,0.6,Cd0.609(Fig.6.41):

1/2

2
2p
2p

Q 0.00653 Cd A t

0.609
(0.03)

4
4
(1 )
998(1 0.6 )
p 100 kPa Ans. (b)

*6.147Air flows through a 6cm


diameter smooth pipe which has a 2 m
long perforated section containing 500
holes (diameter1mm),asinFig.P6.147.
Pressure outside the pipe is sealevel
standardair.Ifp1105kPaandQ1110
3
m /h,estimatep2andQ2,assumingthatthe
holesareapproximatedbythinplateorifices.
Hint:Amomentumcontrolvolumemaybe
veryuseful.

Fig.P6.147

1/2

288

Chapter 6Viscous Flow in Ducts

Solution:Forairat20Cand105kPa,take 1.25kg/m and 1.8E5kg/ms.


UsetheentranceflowratetoestimatethewallshearstressfromtheMoodychart:
V1

Q1
110/3600
m
1.25(10.8)(0.06)

10.8
,
Re

45000, fsmooth 0.0214


1
A ( /4)(0.06)2
s
1.8E5
f
0.0214
then wall V 2
(1.25)(10.8)2 0.390Pa
8
8

Furtherassumethatthepressuredoesnotchangetoomuch,soporifice105000101350
3650Pa.Thentheflowratefromtheorificesis,approximately,

2(3650)

0, Cd 0.61: Q 500Cd A t (2 p/)1/2 500(0.61) (0.001)2


4
1.25
110
or: Q 0.0183 m 3 /s, so Q 2
0.0183 0.01225m 3 /s
3600

1/2

ThenV2Q2/A20.01225/[(/4)(0.06) ]4.33m/s.Acontrolvolumeenclosingthe
pipewallsandsections(1)and(2)yieldsthexmomentumequation:
&2 V2 m
&1V1 AV22 AV12 , dividebyA:
Fx p1A p2 A w DL m
(0.06)(2.0)
1.25(4.33)2 1.25(10.8)2 52 23 146 71Pa
2
( /4)(0.06)

p1 p2 0.390

Thusp210500071105kPaalsoandaboveiscorrect:Q20.0123m /s.Ans.
3

6.148Asmoothpipecontainingethanolat20Cflowsat7m /hthroughaBernoulli
obstruction,asinFig.P6.148.Threepiezometertubesareinstalled,asshown.Ifthe
obstructionisathinplateorifice,estimatethepiezometerlevels(a)h2and(b)h3.

Fig.P6.148

289

Solutions ManualFluid Mechanics, Seventh Edition

Solution:Forethanolat20C,take 789kg/m and 0.0012kg/ms.Withthe


flowrateknown,wecancomputeReynoldsnumberandfrictionfactor,etc.:
V

Q
7/3600
m
789(0.99)(0.05)

0.99
;
Re

32600, fsmooth 0.0230


D
A ( /4)(0.05)2
s
0.0012

FromFig.6.44,at0.6,K1.5.Thentheheadlossacrosstheorificeis
h h 2 h1 K

{0.99/(0.6)2}2
Vt2
(1.5)
0.58m, hence h 2 1.58 m
2g
2(9.81)

Thenthepiezometerchangebetween(2)and(3)isduetoMoodyfrictionloss:
h3 h2 hf f

2
L V2
5 (0.99)
(0.023)
0.12m,
0.05
2(9.81)
d 2g

or h3 1.58 0.12 1.7 m

Ans.(b)

Ans.(a)

290

Chapter 6Viscous Flow in Ducts

6.149Inalaboratoryexperiment,airat20Cflowsfromalargetankthrougha2cm
diametersmoothpipeintoasealevelatmosphere,asinFig.P6.149.Theflowismetered
byalongradiusnozzleof1cmdiameter,usingamanometerwithMeriamredoil(SG
0.827).Thepipeis8mlong.Themeasurementsoftankpressureandmanometerheight
areasfollows:

Fig.P6.149

ptank,Pa(gage): 60

320

1200

2050

2470

3500

4900

hmano,mm:

38

160

295

380

575

820

Usethisdatatocalculatetheflowrates Q andReynoldsnumbersReD andmakeaplotof


measuredflowrateversustankpressure.Istheflowlaminarorturbulent?Comparethedata
withtheoreticalresultsobtainedfromtheMoodychart,includingminorlosses.Discuss.
3

Solution:Forairtake1.2kg/m and0.000015kg/ms.Withnoelevationchangeand
negligibletankvelocity,theenergyequationwouldyield
ptank patm

V 2
L

, K ent 0.5 and K noz 0.7


1 f K entrance K nozzle

2
D

Sincepisgiven,wecanusethisexpressionplustheMoodycharttopredictVandQ
AVandcomparewiththeflownozzlemeasurements.Theflownozzleformulais:
Vthroat Cd

2 pmano
(1 4 )

where p (oil air )gh, Cd fromFig.6.42 and 0.5

ThefrictionfactorisgivenbythesmoothpipeMoodyformula,Eq.(6.48)for 0.The
resultsmaybetabulatedasfollows,andtheplotonthenextpageshowsexcellent(too
good?)agreementwiththeory.

291

Solutions ManualFluid Mechanics, Seventh Edition

ptank,Pa:

60

320

1200

2050

2470

3500

4900

V,m/s(nozzledata):

2.32

5.82

11.9

16.1

18.2

22.3

26.4

2.39

6.22

12.9

17.6

19.9

24.5

29.1

Q,m /h(theory):

2.31

6.25

13.3

18.0

20.0

24.2

28.9

fMoody:

0.0444

0.0331 0.0271 0.0252 0.0245 0.0234 0.0225

Q,m /h(nozzledata):

6.150Gasolineat20Cflowsat0.06m /sthrougha15cmpipeandismeteredbya
9cmdiameterlongradiusflownozzle(Fig.6.40a).Whatistheexpectedpressuredrop
acrossthenozzle?
Solution:Forgasolineat20C,take680kg/mand2.92E4kg/ms.Calculate
thepipevelocityandReynoldsnumber:
V

Q
0.06
m
680(3.40)(0.15)

3.40 , Re D
1.19E6
2
A ( /4)(0.15)
s
2.92E4

TheISOcorrelationfordischarge(Eq.6.114)isusedtoestimatethepressuredrop:

292

Chapter 6Viscous Flow in Ducts

106
Cd 0.9965 0.00653

Re D

1/2

106 (0.6)

0.9965 0.00653

1.19E6

1/2

0.9919

2 p

Then Q 0.06 (0.9919) (0.09)2
,
4
680(1 0.6 4 )
Solve p 27000 Pa

Ans.

P6.151 Anengineerneedstomonitoraflowof20Cgasolineatabout25025gal/min
througha4indiametersmoothpipe.Shecanuseanorificeplate,alongradiusflownozzle,or
aventurinozzle,allwith2indiameterthroats.Theonlydifferentialpressuregageavailableis
accurateintherange6to10lbf/in2.Disregardingflowlosses,whichdeviceisbest?

Solution:Forgasolineat20C,take=680kg/m3and=2.92E4kg/ms.Wearegiven

=2/4=0.5. Theflowrateisintherange0.0142< Q <0.0174m3/s. ThepipeReynolds


numberisintherangeReD=460,00010%.InSIunits,thethroatdiameteris0.0508m,andits
areais(/4)(0.0508m)2=0.00203m2.OurbasicobstructionformulaisEq.(6.104):

Q C d At

2 p

(1 4 )

C d (0.00203m 2 )

2 p
(680kg / m 3 ){1 (0.5) 4 }

0.0158 0.0016

ItremainsonlytodetermineCdforthethreedevicesandthencalculatep.Theresultsare:
Orificeplate,D:1/2Dtaps:Cd0.605,p=6.2to9.3lbf/in2Ans.
Longradiusflownozzle:Cd0.99,p=2.3to3.5lbf/in2
Venturinozzle:Cd0.977,p=2.4to3.6lbf/in2
Only the orifice plate, with its high losses, is compatible with the available pressure gage.

m3
s

293

Solutions ManualFluid Mechanics, Seventh Edition

6.152Keroseneat20Cflowsat20m /hinan8cmdiameterpipe.Theflowistobe
meteredbyanISA1932flownozzlesothatthepressuredropis7kPa.Whatisthe
propernozzlediameter?
3

Solution: Forkeroseneat20 C,take 804kg/m and1.92E3kg/ms.We


cannotcalculatethedischargecoefficientexactlybecausewedontknow ,sojust
estimateCd:
2(7000)
20 m 3

2
Guess Cd 0.99, then Q 0.99 (0.08 )

4
804(1 4 ) 3600 s

2
or:
0.268, solve 0.508,
(1 4 )1/2
Re D

4(804)(20/3600)
37000
(1.92E3)(0.08)

NowcomputeabetterCdfromtheISAnozzlecorrelation,Eq.(6.115):
Cd 0.99 0.2262

4.1

(0.000215 0.001125 0.00249

4.7

106
)
Re D

1.15

Iterateoncetoobtainabetter0.515,d0.515(8cm)4.12cmAns.

0.9647

294

Chapter 6Viscous Flow in Ducts

6.153Two water tanks, each with base


2
area of 1 ft , are connected by a 0.5in
diameterlongradiusnozzleasinFig.P6.153.
Ifh1ftasshownfort0,estimatethe
timeforh(t)todropto0.25ft.

Fig.P6.153

Solution:For water at 20C, take


3
1.94slug/ft and2.09E5slug/fts.For
alongradiusnozzlewith0,guessCd
0.98andKloss 0.9fromFig.6.44.The
elevation difference h must balance the
head losses in the nozzle and submerged
exit:
z h loss

Vt2
Vt2

K
(0.9nozzle 1.0 exit ) h, solve Vt 5.82 h
2g
2(32.2)

1/2

hence Q Vt
4 12

1
dh
dh
0.00794 h A tank
0.5
2
dt
dt

Theboldfacefactor 1/2 accountsforthefactthat,asthelefttankfallsbydh,theright


tankrisesbythesameamount,hencedhdtchangestwiceasfastasforonetankalone.
Wecanseparateandintegrateandfindthetimeforhtodropfrom1ftto0.25ft:
1.0

0.25

dh
h

0.0159

t final

dt, or: t final

1 0.25
0.0159

63s

Ans.

P6.154
Gasoline at 20C flows through a 6-cm-diameter pipe. It is metered by a
modern venturi nozzle with d = 4 cm. The measured pressure drop is 6.5 kPa. Estimate
the flow rate in gallons per minute.
Solution: For gasoline at 20C, from Table A.3, = 680 kg/m3 and = 2.92E-4 kg/m-s.
We are not sure of the Reynolds number, so assume for the present that Fig. 6.43 is valid,
in which case, from Eq. (6.116),
0.04 4.5
Cd ,venturi 0.9858 0.196 4.5 0.9858 0.196 (
)
0.954
0.06
Then, from Eq. (6.104), the flow rate is

295

Solutions ManualFluid Mechanics, Seventh Edition

Q Cd At

2 (8500 Pa )

m3
gal
2
0.954( )(0.04m)
0.00669
106
Ans.
4
4
4
s
min
(1 )
680[1 (4 / 6) ]
2 p

4Q
4(680)(0.00669)

330, 000 OK, within range.


D (2.92E 4)(0.06)
________________________________________________________________________
Check Re D

P6.155
It is desired to meter methanol at 20C flowing through a 5-inch-diameter
pipe. The expected flow rate is about 300 gal/min. Two flowmeters are available: a
venturi nozzle and a thin-plate orifice, each with d = 2 in. The differential pressure gage
on hand is most accurate at about 12-15 lbs/in2. Which meter is better for this job?
Solution: For methanol at 20C, from Table A.3, = 791 kg/m3 = 1.535 slug/ft3 and =
5.98E-4 kg/m-s = 1.25E-5 slug/ft-s. Compute the average velocity in the pipe and find the
discharge coefficients for each meter, for = (2 in)/(5 in) = 0.4:
gal
ft 3 5
ft
ft
0.668
( ft )2 Vavg ,solve for V 4.90
or Vt 30.6
min
s
4 12
s
s
VD (1.535)(4.90)(5 /12)
Orifice : Re D

251, 000, 0.4, Fig .6.41: Cd 0.601

(0.0000125)
Venturi nozzle : 1.5E5 Re D 2E6, OK , 0.4, Fig .6.43 : Cd 0.983
Q 300

Find the expected pressure drops from Eq. (6.104):


Vt 30.6

ft
Cd
s

2 p

(1 4 )

Venturi nozzle :

Cd

Cd 0.983 ,

Thin - plate orifice : Cd 0.601 ,

2 p
(1.535)[1 (0.4)4 ]
solve
solve

1.156 Cd

p 725
p 1940

lbf
2

ft
lbf

ft 2

5
14

p
lbf
in 2
lbf
in 2

The orifice plate is the better choice, for accuracy, although the head loss is much larger.

296

Chapter 6Viscous Flow in Ducts

6.156Ethanolat20CflowsdownthroughamodernventurinozzleasinFig.P6.156.If
themercurymanometerreadingis4in,asshown,estimatetheflowrate,ingal/min.
3

Solution:Forethanolat20C,take1.53slug/ft and2.51E5slug/fts.Given
0.5,thedischargecoefficientis
4.5

Cd0.98580.196(0.5) 0.9771

Fig.P6.156

The9inchdisplacementofmanometertapsdoesnotaffectthepressuredropreading,
becausebothlegsarefilledwithethanol.ThereforeweproceeddirectlytopandQ:
pnozzle ( merc eth )gh (26.3 1.53)(32.2)(4/12) 266lbf/ft 2

2 p
Hence Q Cd A t
4
(1 )

1/2

3

4 12

0.9771

2(266)
ft 3

0.924
s
1.53(1 0.54 )

Ans.

6.157ModifyProb.6.156ifthefluidis airat20C,enteringtheventuriatapressureof
18psia.Shouldacompressibilitycorrectionbeused?
3

Solution:Forairat20Cand18psi,take0.00286slug/ft and3.76E7slug/fts.
Withstillequalto0.5,Cdstillequals0.9771aspreviouspage.Themanometerreadingis
pnozzle (26.3 0.00286)(32.2)(4/12) 282lbf/ft 2 ,
3
whence Q 0.9771
4 12

2(282)
ft 3
22.0
s
0.00286(1 0.54 )

Ans.

297

Solutions ManualFluid Mechanics, Seventh Edition

Fromthisresult,thethroatvelocityV tQ/At448ft/s,quitehigh,theMachnumberin
thethroatisapproximatelyMa 0.4,a (small)compressibilitycorrectionmightbe
expected. [Makingaonedimensionalsubsonicflowcorrection,usingthemethodsof
3
Chap.9,resultsinathroatvolumeflowestimateofQ22.8ft /s,about4%higher.]

6.158Waterat20Cflowsinalonghorizontalcommercialsteel6cmdiameterpipe
thatcontainsaclassicalHerschelventuriwitha4cmthroat.Theventuriisconnectedto
3
amercurymanometerwhosereadingis h40cm.Estimate(a)theflowrate,inm /h,
and (b) the total pressure difference between points 50 cm upstream and 50 cm
downstreamoftheventuri.
3

Solution:Forwaterat20C,take998kg/m and0.001kg/ms.Forcommercial
steel,0.046mm,hence/d0.046/600.000767.Firstestimatetheflowrate:
p ( m w )gh (13560 998)(9.81)(0.40) 49293Pa
2(49293)
m3

2
(0.04)

0.0137

s
998[1 (4/6)4 ]
4

Guess Cd 0.985, Q (0.985)

Check Re D

4Q
291000
D

AtthisReynoldsnumber,weseefromFig.6.42thatCd doesindeed 0.985forthe


3
3
Herschelventuri.Therefore,indeed,Q0.0137m /s49m /h.Ans.(a)
(b)50cmupstreamand50cmdownstreamarefarenoughthatthepressurerecovers
fromitsthroatvalue,andthetotal pisthesumofMoodypipelossandventurihead
2
loss.Firstworkoutthepipevelocity,VQ/A(0.0137)/[(/4)(0.06) ]4.85m/s.Then
Re D 291000,

0.000767, then fMoody 0.0196; Fig.6.44: K venturi 0.2


d

Then p p Moody p venturi


998(4.85)2

V 2 L
f K
2 d

1.0
0.2 6200 Pa
0.0196 0.06

Ans.(b)

6.159Amodernventurinozzleistestedinalaboratoryflowwithwaterat20 C.The
pipediameter is5.5cm,andthe venturithroatdiameter is3.5cm.Theflowrateis
measuredbyaweightankandthepressuredropbyawatermercurymanometer.The
massflowrateandmanometerreadingsareasfollows:

298

Chapter 6Viscous Flow in Ducts

m&, kg/s:

0.95

1.98

2.99

5.06

8.15

h,mm:

3.7

15.9

36.2

102.4

264.4

UsethesedatatoplotacalibrationcurveofventuridischargecoefficientversusReynolds
number.Comparewiththeacceptedcorrelation,Eq.(6.116).
3

Solution:Forwaterat20C,take998kg/m and0.001kg/ms.Thegivendata
ofmassflowandmanometerheightcanreadilybeconvertedtodischargecoefficientand
Reynoldsnumber:
Q

&
&
m
2(13.56 1)w (9.81)h
m(kg/s)

Cd (0.035)2
, or: Cd
4
4
998
16.485 h meters
w [1 (3.5/5.5) ]
ReD

&
4m&
4m
&

23150m(kg/s)
D (0.001)(0.055)

Thedatacanthenbeconvertedandtabulatedasfollows:
h,m:

0.037

0.0159

0.0362

0.1024

0.2644

Cd:

0.947

0.953

0.953

0.959

0.962

ReD:
22000
46000
69000
117000
189000
Thesedataareplottedinthegraphbelow,similartoFig.6.42ofthetext:

TheycloselyresembletheclassicalHerschelventuri,butthisdataisactuallyfora
modernventuri,forwhichweonlyknowthevalueofCdfor1.5E5ReD2E5:

299

Solutions ManualFluid Mechanics, Seventh Edition

3.5
Eq.(6.116) Cd 0.9858 0.196

5.5

4.5

0.960

ThetwodatapointsnearthisReynoldsnumberrangearequitecloseto0.9600.002.

P6.160 An instrument popular in the beverage


industry is the target flowmeter in Fig. P6.x.

Flow

A small flat disk is mounted in the center of the


pipe, supported by a strong but thin rod.

Fig.P6.x

(a) Explain how the flowmeter works.


(b) If the bending moment M of the rod is measured at the wall,
derive a formula for the estimated velocity of the flow.
(c) List a few advantages and disadvantages of such an instrument.

Solution: (a) The flow creates a drag force F on the disk, approximately proportional to V2.
(b) The bending moment is the drag force times the pipe radius. Thus the formula
M FR ,

2
V Adisk
2
2M
Solve for V
CD A disk R

where

F CD

Ans.(b)

We are neglecting the drag of the thin rod, especially if it is streamlined. In Chapter 7,
Fig. 7.16, we learn that the drag coefficient of a disk is about 1.2 over a wide range of
Reynolds number.
(c) Advantages: low cost; disk easy to keep clean; works for a wide variety of fluids; can
measure flow in either direction; useful in moderately unsteady flows. Disadvantages:
needs calibration because the formula is too simplified; very poor accuracy at low
velocities, due to the square-root relationship; heavy flows can break the rod; gummy
flows can coat the disk and change the calibration; the drag force causes a pipe head loss.

Chapter 6Viscous Flow in Ducts

P6.161

300

An instrument popular in the

water supply industry, sketched in Fig. P6.161,


is the single jet water meter.
(a) How does it work?
(b) What do you think a typical calibration
curve would look like?
(c) Can you cite further details, for example, reliability,

Fig.P6.161

head loss, cost [58]?

Solution: (a) The single-jet meter is similar in principle to the standard turbine meter, Fig.
6.32 of the text, except that, instead of flowing axially through the turbine, it creates an inplane jet that strikes the turbine blades and turns them. There are also multi-jet meters.
(b) Just as in a standard turbine meter, the single-jet meter turns the turbine at a rate nearly
proportional to the flow rate. The manufacturer states a K factor of an equation for turning
rate versus volume flow rate Q, in the form = K Q. At very low flow rates, the
Reynolds number is very low, and the manufacturer includes an error curve (a few per cent
deviation) for low rates.
(c) The single-jet meter is quite reliable and prized for its accuracy at low flow rates. It is not
used for large pipe sizes because, due to its offset design, it would have to be huge.
_______________________________________________________________________
_

301

Solutions ManualFluid Mechanics, Seventh Edition

6.162Air flows at high speed through a Herschel venturi monitored by a mercury


manometer,asshowninFig.P6.161.Theupstreamconditionsare150kPaand80C.Ifh
37cm,estimatethemassflowinkg/s.[HINT:Theflowiscompressible.]
Solution:Theupstreamdensityis 1 p1/(RT) (150000)/[287(273 80)] 1.48
3
kg/m .Thecluehighspeedmeansthatwehadbetterusethe compressible venturi
formula,Eq.(6.117):

Fig.P6.162

2 1 ( p1 p2 )
1 4

m& CdYAt

where 4/6 forthisnozzle.

Thepressuredifferenceismeasuredbythemercurymanometer:
p1 p2 ( merc air ) gh (13550 1.48kg/m3 )(9.81m/s2 )(0.37m) 49200Pa
Thepressureratioisthus(15049.2)/1500.67and,for2/3,wereadY0.76from
Fig. 6.45.FromFig.6.43estimate Cd 0.985.The(compressible)venturiformulathus
predicts:

(0.04 m) 2
4

m& 0.985(0.76)

2(1.48)(49200)
kg
0.40
4
s
1 (2/3)

Ans.

*6.163ModifyProb.6.162asfollows.Findthemanometerreading h forwhichthe
mass flow through the venturi is approximately 0.4 kg/s. [ HINT: The flow is
compressible.]

Chapter 6Viscous Flow in Ducts

302

Solution:This is, in fact, the answer to Prob. 6.162, but who knew? The present
problemisintendedasaniterationexercise,preferablywithEES.Weknowtheupstream
pressureanddensityandthedischargecoefficient,butwemustiterateforYandp2inthe
basicformula:
m& CdYAt

2 1 ( p1 p2 )
0.40kg/s
1 4

Theanswershouldbeh0.37m,Y0.76,andCd0.985,asinProb.6.162,butthe
problemisextremelysensitivetothevalueofh.A10%changeinhcausesonlya2%
changeinmassflow.TheactualanswertoProb.6.161wasamassflowof0.402kg/s.
EESreportsthat,formassflowexactlyequalto0.400kg/s,therequiredmanometerheight
ish0.361m.Ans.

FUNDAMENTALS OF ENGINEERING EXAM PROBLEMS: Answers


FE6.1Inflow through astraight, smoothpipe, thediameter Reynolds number for
transitiontoturbulenceisgenerallytakentobe
(a)1500(b)2300(c)4000(d)250,000(e)500,000
3
FE6.2Forflowofwaterat20Cthroughastraight,smoothpipeat0.06m /h,thepipe
diameterforwhichtransitiontoturbulenceoccursisapproximately
(a)1.0cm(b)1.5cm(c)2.0cm(d)2.5cm(e)3.0cm
FE6.3Forflowofoil(0.1kg/(ms),SG0.9)throughalong,straight,smooth5cm
3
diameterpipeat14m /h,thepressuredroppermeterisapproximately
(a)2200Pa(b)2500Pa(c)10,000Pa(d)160Pa(e)2800Pa
FE6.4ForflowofwaterataReynoldsnumberof1.03E6througha5cmdiameter
pipeofroughnessheight0.5mm,theapproximateMoodyfrictionfactoris
(a)0.012(b)0.018(c)0.038(d)0.049(e)0.102
FE6.5Minorlossesthroughvalves,fittings,bends,contractions etc.arecommonly
modeledasproportionalto
(a)totalhead(b)statichead(c)velocityhead(d)pressuredrop(e)velocity
FE6.6Asmooth8cmdiameterpipe,200mlong,connectstworeservoirs,containing
waterat20C,oneofwhichhasasurfaceelevationof700mandtheotherwithits
surfaceelevationat560m.Ifminorlossesareneglected,theexpectedflowratethrough
thepipeis
3
3
3
3
3
(a)0.048m /h(b)2.87m /h(c)134m /h(d)172m

/h(e)385m /h
3
FE6.7If,inProb.FE6.6thepipeisroughandtheactualflowrateis90m /hr,thenthe
expectedaverageroughnessheightofthepipeisapproximately

303

Solutions ManualFluid Mechanics, Seventh Edition

(a)1.0mm(b)1.25mm(c)1.5mm(d)1.75mm(e)2.0mm
FE6.8SupposeinProb.FE6.6thetworeservoirsareconnected,notbyapipe,butby
asharpedgedorificeofdiameter8cm.Thentheexpectedflowrateisapproximately
3
3
3
3
3
(a)90m /h(b)579m

/h(c)748m /h(d)949m /h(e)1048m /h


FE6.9Oil( 0.1kg/(ms),SG 0.9)flowsthrougha50mlongsmooth8cm
diameter pipe. The maximum pressure drop for which laminar flow is expected is
approximately
(a)30kPa(b)40kPa(c)50kPa(d)60kPa(e)70kPa
FE6.10Airat20Candapproximately1atmflowsthroughasmooth30cmsquare
ductat1500cubicfeetperminute.Theexpectedpressuredroppermeterofductlength
is
(a)1.0Pa(b)2.0Pa(c)3.0Pa(d)4.0Pa(e)5.0Pa
FE6.11Waterat20Cflowsat3cubicmetersperhourthroughasharpedged3cm
diameterorificeina6cmdiameterpipe.Estimatetheexpectedpressuredropacrossthe
orifice.
(a)440Pa(b)680Pa(c)875Pa(d)1750Pa(e)1870Pa
FE6.12Waterflowsthroughastraight10cmdiameterpipeatadiameterReynolds
numberof250,000.Ifthepiperoughnessis0.06mm,whatistheapproximateMoody
frictionfactor?
(a)0.015(b)0.017(c)0.019(d)0.026(e)0.032
FE 6.13What is the hydraulic diameter ofa rectangular airventilation duct whose
crosssectionis1meterby25cm?
(a)25cm(b)40cm(c)50cm(d)75cm(e)100cm
FE6.14Waterat20Cflowsthroughapipeat300gal/minwithafrictionheadlossof
45ft.Whatisthepowerrequiredtodrivethisflow?
(a)0.16kW(b)1.88kW(c)2.54kW(d)3.41kW(e)4.24kW
FE6.15Waterat20Cflowsat200gal/minthroughapipe150mlongand8cmin
diameter.Ifthefrictionheadlossis12m,whatistheMoodyfrictionfactor?
(a)0.010(b)0.015(c)0.020(d)0.025(e)0.030

304

Chapter 6Viscous Flow in Ducts

COMPREHENSIVE PROBLEMS
C6.1Apitotstaticprobewillbeusedtomeasurethevelocitydistributioninawater
tunnelat20C.ThetwopressurelinesfromtheprobewillbeconnectedtoaUtube
manometerwhichusesaliquidofspecificgravity1.7.Themaximumvelocityexpected
in the water tunnel is 2.3 m/s. Your job is to select an appropriate Utube from a
manufacturerwhichsuppliesmanometersofheights8,12,16,24and36inches.Thecost
increasessignificantlywithmanometerheight.Whichoftheseshouldyoupurchase?
Solution:The pitotstatic tube formula relates velocity to the difference between
stagnationpressurepoandstaticpressurepsinthewaterflow:
po ps

1
kg
wV 2 , where w 998 3
2
m

and Vmax 2.3

m
s

Meanwhile,themanometerreadinghrelatesthispressuredifferencetothetwofluids:
po ps (mano w )gh w (SGmano 1)gh
Solvefor hmax

2
Vmax
(2.3)2

0.385m 15.2 in
2g(SGmano 1) 2(9.81)(1.7 1)

Itwouldthereforebemosteconomicalto buythe16inchmanometer.Butbecareful
whenyouuseit:abitofoverpressurewillpopthemanometerfluidoutofthetube!

C6.2Apumpdeliversasteadyflowofwater(,)fromalargetanktotwoother
higherelevationtanks,asshown.Thesamepipeofdiameterdandroughnessisused
throughout.Allminorlosses exceptthroughthevalve areneglected,andthepartially
closedvalvehasalosscoefficientKvalve.Turbulentflowmaybeassumedwithallkinetic
energy flux correction coefficients equal to 1.06. The pump net head H is a known
functionofQAandhencealsoofVAQA/Apipe,forexample, H a bVA2 , whereaand
bareconstants.SubscriptJreferstothejunctionpointattheteewherebranchAsplits
intoBandC.PipelengthL CismuchlongerthanLB.Itisdesiredtopredictthepressure
atJ,thethreepipevelocitiesandfrictionfactors,andthepumphead.Thusthereare8
variables:H,VA,VB,VC,fA,fB,fC,pJ.Writedowntheeightequationsneededtoresolve
thisproblem,but donotsolve,sinceanelaborateiterationprocedure,oranequation
solversuchasEES,wouldberequired.

305

Solutions ManualFluid Mechanics, Seventh Edition

Solution:First,equation(1)isclearlythepumpperformance:
H a bVA2

3Moody factors:

f A fcn VA ,

f B fcn VB ,

fC fcn VC ,

Conservationofmass(constantarea)atthejunctionJ:VAVBVC
Finally,therearethreeindependentsteadyflowenergyequations:

( J )to (2):

(2)
(3)
(4)
(5)

L A VA2
L V2
fB B B
d 2g
d 2g

(6)

L V2
V2
L A VA2
fC C C K valve C
d 2g
d 2g
2g

(7)

(1)to (2): z1 z2 H f A
(1)to (3): z1 z3 H f A

(1)

pJ
p
L V2
zJ atm z2 f B B B
g
g
d 2g

Fig.PC6.2

(8)

306

Chapter 6Viscous Flow in Ducts

C6.3The water slide in the figure is to be installed in a swimming pool. The


3
manufacturerrecommendsacontinuouswaterflowof1.39E3m /s(about22gal/min)
downtheslidetoensurethatcustomers donotburntheirbottoms.An80%efficient
pumpundertheslide,submerged1mbelowthewatersurface,feedsa5mlong,4cm
diameterhose,ofroughness0.008cm,totheslide.Thehosedischargesthewateratthe
topoftheslide,4mabovethewatersurface,asafreejet.Ignoreminorlossesand
assume1.06.Findthebrakehorsepowerneededtodrivethepump.

FigC6.3

Solution:For water take 998 kg/m and 0.001 kg/ms. Write the steady-flow energy
equation from the water surface (1) to the outlet (2) at the top of the slide:

pa 1V12
pa 2V22
1.39 E3
m

z1

z2 h f h pump , where V2
1.106
2
g
2g
g
2g
s
(0.02)
Solve for h pump

V22
L
( z2 z1 )
2 f
2g
d

307

Solutions ManualFluid Mechanics, Seventh Edition

Work out Red Vd/ (998)(1.106)(0.04)/0.001 44200, /d 0.008/4 0.002,


whencefMoody0.0268.Usethesenumberstoevaluatethepumpheadabove:
h pump (5.0 1.0)
whence BHPrequired

(1.106)2
2(9.81)

5.0
4.27 m,
1.06 0.0268 0.04

gQh pump 998(9.81)(1.39E3)(4.27)

73 watts Ans.

0.8

C6.4Supposeyoubuildahouseoutintheboonies,whereyouneedtorunapipeto
thenearestwatersupply,whichfortunatelyisabout1kmabovetheelevationofyour
house.Thegagepressureatthewatersupplyis1MPa.Yourequireaminimum of
3gal/minwhenyourendofthepipeisopentotheatmosphere.Tominimizecost,you
wanttobuythesmallestpossiblediameterpipewithanextremelysmoothsurface.
(a)Findthetotalheadlossfrompipeinlettoexit,neglectingminorlosses.
(b)Whichismoreimportanttothisproblem,theheadlossduetoelevationdifference,or
theheadlossduetopressuredropinthepipe?
(c)Findtheminimumrequiredpipediameter.
3

Solution:Convert3.0gal/minto1.89E4m /s.Let1betheinletand2betheoutlet
andwritethesteadyflowenergyequation:

Fig.C6.4

p1gage

1V12

z1

p2 gage

2V22
z2 h f
2g

g
2g
g
p1gage
1E 6kPa
1000m
1000 102 1102m
or: h f z1 z2
g
998(9.81)

Ans.(a)

(b)Thus,elevationdropof1000mismoreimportanttoheadlossthanp/g102m.

308

Chapter 6Viscous Flow in Ducts

(c)Tofindtheminimumdiameter,iteratebetweenflowrateandtheMoodychart:
hf f

L V2
, L 6000m,
d 2g

2.51
1
Q
2 log
,
, V
d 2 /4
f
Re f

m3
Vd
Q 1.89E4
, Re
s
v
2
Wearegivenhf1102mandwater1.005E6m /s.Wecaniterate,ifnecessary,oruse
EES,whichcanswiftlyarriveatthefinalresult:
fsmooth 0.0266; Re 17924; V 1.346m/s; d min 0.0134m

Ans.(c)

C6.5Waterat20Cflows,atthesameflowrateQ9.4E4m /s,throughtwoducts,
onearoundpipe,andoneanannulus,asshown.ThecrosssectionareaAofeachductis
identical,andeachhaswallsofcommercialsteel.Botharethesamelength.Inthecross
sectionsshown,R15mmanda25mm.
(a) Calculate the correct radius b for the
annulus.
Fig.C6.5
(b)Compareheadlossperunitlengthfor
the two ducts, first using the hydraulic
diameter and second using the effective
diameterconcept.
(c)Ifthelossesaredifferent,why?Which
ductismoreefficient?Why?
Solution:(a)Settheareasequal:
A R 2 (a 2 b2 ), or: b a 2 R 2 (25)2 (15)2 20mm

Ans.(a)

(b)Findtheroundpipeheadloss,assuming1.005E6m /s:
Q 9.4 E4m 3 /s
m
(1.33)(0.030)
V
1.33 ; Re
39700;
2
A (0.015m)
s
1.005E6

0.00153, f Moody 0.0261


d
2

Thushf/L(f/d)(V /2g)(0.0261/0.03)(1.33 )/2/9.810.0785(round)Ans.(b)

Annulus:Dh4A/P2(ab)20mm,sameV1.33m/s:

309

Solutions
VD ManualFluid
Mechanics, Seventh Edition

Re Dh

26500,

Dh

0.0023, f Moody 0.0291,

f V 2
0.131 ( annulus) Ans.(b)
Dh 2 g

h f /L

Effectivediameterconcept:b/a0.8,Table6.3:Deff0.667Dh13.3mm.Then

Re Deff 17700,
0.00345, f Moody 0.0327,
Deff
hf

f V2

0.147 (annulusDeff ) Ans.(b)


L Dh 2 g

NOTE:EverythinghereusesDeffexcepthf,whichbydefinitionusesDh.
Weseethattheannulushasabout85%moreheadlossthantheroundpipe,forthesamearea
andflowrate!Thisisbecausetheannulushasmorewallarea,thusmorefriction.Ans.(c)

C6.6JohnLaufer(NACATech.Rep.1174,1954)gavevelocitydatafor20Cairflowin
asmooth24.7cmdiameterpipeatRe5E5:
u/uCL:

1.0

0.997 0.988 0.959 0.908 0.847 0.818 0.771 0.690

r/R:

0.0

0.102 0.206 0.412 0.617 0.784 0.846 0.907 0.963

The centerline velocity uCL was 30.5 m/s. Determine (a) the average velocity by
numerical integration and (b) the wall shear stress from the loglaw approximation.
ComparewiththeMoodychartandwithEq.(6.43).
3

Solution: Forairat20C,take 1.2kg/m and 0.00018kg/ms.Theaverage


velocityisdefinedbythe(dimensionless)integral
1
V
R2

V
u
r
u(2 r)dr, or:

2 d, where
uCL 0 uCL
R

Prepareaspreadsheetwiththedataandcarryouttheintegrationbythetrapezoidalrule:
1

u
2 d [(u/uc )2 2 (u/uc )1 1 ](2 1 ) [(u/u c )3 3 (u/u c )2 2 ](3 2 ) L
uc

Theintegralisevaluatedonthespreadsheetbelow.TheresultisV/uCL0.8356,
orV(0.8356)(30.5)25.5m/s.Ans.(a)
Thewallshearstressisestimatedbyfittingtheloglaw(6.28)toeachdatapoint:
Foreach(u,y),

u 1 yu*
ln
B, and B 5.0
u*

310

Chapter 6Viscous Flow in Ducts

Weknowforairandaregivenuandyfromthedata,hencewecansolveforu*.The
2
2
spreadsheetgivesu*1.1m/s1%,orwu* (1.2)(1.1) 1.45Pa.Ans.(b)
y/R

r/R

u/uCL

u/uCL2 r /Rdr/R

u*

1.000
0.898
0.794
0.588
0.383
0.216
0.154
0.093
0.037
0.000

0.000
0.102
0.206
0.412
0.617
0.784
0.846
0.907
0.963
1.000

1.000
0.997
0.988
0.959
0.908
0.847
0.818
0.771
0.690
0.000

.0000
.0104
.0421
.1654
.3613
.5657
.6498
.7347
.8111
.8356

1.126
1.128
1.126
1.112
1.099
1.101
1.098
1.097

WemakesimilarestimatesfromtheMoodychartbyevaluatingReandfanditerating:
1.2(25)(0.247)
412000, fsmooth 0.0136
0.00018
uCL /[1 1.3 f] 26.5, whence Re 436000, fbetter 0.0135

GuessV 25m/s, thenRe


Vbetter

ThisconvergestoV26.5m/sAns.and w(f/8)V 1.42Pa.Ans.


C6.7Consider energy exchange in fullydeveloped laminar flow between parallel
plates,asinEq.(6.63).LetthepressuredropoveralengthLbep.Calculatetherateof
workdonebythispressuredroponthefluidintheregion(0xL,hyh)and
compare with the integrated energy dissipated due to the viscous function from
Eq.(4.50)overthissameregion.Thetwoshouldbeequal.Explainwhythisisso.Can
yourelatetheviscousdragforceandthewallshearstresstothisenergyresult?
Solution:FromEq.(6.63),thevelocityprofilebetweentheplatesisparabolic:
3
y 2
u V 1 2
2
h

h 2 p
where V
istheaveragevelocity
3 L

Letthewidthoftheflowbedenotedbyb.Theworkdonebypressuredroppis:
6 LbV 2
3 LV
W&pressure pVA
(
V
)(2
hb
)

h
h2

311

Solutions ManualFluid Mechanics, Seventh Edition

Meanwhile,fromEq.(4.50),theviscousdissipationfunctionforthisfullydevelopedflowis:
2

u
9 V 2 y 2
3Vy

h4
h
y
Integratethistogetthetotaldissipatedenergyovertheentireflowregionofdimensions
Lbybby2h:
h
9V 2 y 2
6 LbV 2
&
Edissipated Lb
dy
W&pressure ! Ans.
4
h
h

Thetwoenergytermsareequal.Thereisnoworkdonebythewallshearstresses(where
u0),sothepressureworkisentirelyabsorbedbyviscousdissipationwithintheflow
field.Ans.

C6.8Thistexthaspresentedthetraditionalcorrelationsforturbulentsmoothwallfriction
factor,Eq.(6.38),andthelawofthewall,Eq.(6.28).Recently,groupsatPrincetonandOregon
[56]havemadenewfrictionmeasurementsandsuggestthefollowingsmoothwallfrictionlaw:
1
f

1.930 log10 ( Re D

f ) 0.537

Inearlierwork,theyalsoreportthatbettervaluesfortheconstantsandBintheloglaw,
Eq.(6.28),are0.4210.002and B5.620.08.(a)Calculateafewvaluesoffinthe
range1E4ReD1E8andseehowthetwoformulasdiffer.(b)ReadRef.56andbrieflycheck
thefivepapersinitsbibliography.Reporttotheclassonthegeneralresultsofthiswork.

Solution: The two formulas are practically identical except as the Reynolds number is
very high or very low. The new formula was fit to new, and extensive, friction data in
Ref. 56 and can thus be said to be slightly more accurate. Here is a table of calculations.

Chapter 6Viscous Flow in Ducts

ReD
3000
10000
30000
100000
300000
1000000
3.0E+06
1.0E+07
3.0E+07
1.0E+08

fPrandtl
0.04353
0.03089
0.02349
0.01799
0.01447
0.01165
0.009722
0.008104
0.006949
0.005941

f Ref.56

0.04251
0.0305
0.02344
0.01811
0.01464
0.01186
0.009938
0.008316
0.007153
0.006134

312

Difference
-2.41%
-1.10%
-0.18%
0.62%
1.22%
1.76%
2.17%
2.56%
2.86%
3.15%

They differ by no more than 3%.

C6.9
A pipeline has been proposed to carry natural gas 1715 miles from Alaskas
North Slope to Calgary, Alberta, Canada. The (smooth) pipe diameter will be 52 inches.
The gas will be at high pressure, averaging 2500 lbs/in2. (a) Why? The proposed flow rate
is 4 billion cubic feet per day at sea-level conditions. (b) What volume flow rate, at 20C,
would carry the same mass at the high pressure? (c) If natural gas is assumed to be
methane (CH4), what is the total pressure drop? (d) If each pumping station can deliver
12,000 hp to the flow, how many stations are needed?
Solution: From Table A.4, for CH4, R = 518 m2/(s2-K) and = 1.03E-5 kg/m-s. Sea-level
density is o = p/RT = 101350/[518(288)] = 0.679 kg/m3. The proposed mass flow rate is

ol (4.0E9 ft 3 )(0.3048m / ft )3 1.13E8 m3 ;


Q ol / time (1.13E8 m3 ) /(24 3600 s ) 1311 m3 / s
m& o Q (0.679 kg / m3 )(1311 m3 / s) 890 kg / s
(a) The high pressure means that the gas is compacted about 170 times and thus much
less volume needs to be pumped. Ans.(a)
(b) At 2500 lbs/in2 = 1.72E7 Pa and 20C, the average gas density would be

313

Solutions ManualFluid Mechanics, Seventh Edition

p
1.72E7 Pa
kg

114 3
2
2
RT
(518 m / s K )(293K )
m

To match the mass flows, we would have

kg
kg
m3
m&pipeline (114 3 ) Q pipeline , hence Q pipeline 7.84
Ans.(b)
s
s
m
(c) For the pressure drop, first find the Moody (smooth-wall) friction factor:
m&standard 890

7.84 m3 / s
Q
m
VD (114)(5.73)(1.32)

5.73 ; Re pipeline

8.35E7
2
A
s

0.0000103
( / 4)(1.32m)

V pipeline
1
f

2.0 log(

Re D f
)
2.51

yields

f smooth 0.00608

Convert 1715 miles = 2760 km. Then the Darcy formula gives the (horizontal) pressure drop:

ptotal f

L 2
2760000m 114
lbf
V (0.00608)(
)(
)(5.73) 2 2.37E7 Pa 3440 2 Ans.(c )
D 2
1.32m
2
in

(d) Total horsepower = Q p = (7.84)(2.37E7) = 1.86E8 W 745.7 = 249,000 hp total.


If we divide this by 12,000 hp per pump, we get 20.76 or 21 pump stations
minimum. Ans. (d)
Each station must increase the gas pressure by 3440/21 = 164 psi, only 7% of the average
pressure in the pipeline.

You might also like